You are on page 1of 202

Basic Mathematics

1 & Measurements
Section I
Single Correct Option
1 p1
1, K = mv2 p2 =
2 9
 ∆K  ∆m ∆v 100
∴   = +2 ∴ Percentage increase in pressure =
 K  max m v 9
∴ Maximum error = 2% + 2 (3%) = 111
.
In the estimate of, Option (a) is correct.
p2
kinetic energy ( K ) = 8% 5. K =
2m
Option (c) is correct.
m m ∴ Error in the measurement of kinetic
2. d = = 3 energy (K)
V l
= 2 × 100%

 ∆d × 100 = 
∆m ∆l
× 100 + 3  × 100
= 200%
 
 d  max  m   l  Option (d) is correct.
= 4% + 3(3%) 6. 3400 = 3.400 × 103
= 13% ∴ Number of significant figures = 2
Option (d) is correct. Option (d) is correct.
F F
3. p = = 7. A = 3.124 m × 3.002 m
A L2
= 9.378 / 248 m 2
∴ Permissible error in pressure ( p)
= 9.378 m 2
= 4% + 2 (2%)
Option (a) is correct.
= 8%
GM
Option (a) is correct. 8. g= 2
R
4. p1V1 = p2 V2 constant
=

V
p2 = p1 1 R2
V2 1 2
K = Iω
V1 2
= p1 1 2
V1 − 10% of V1 =  MR2  ω
p1 2 5 
=
90% = constant × R2
Basic Mathematics & Measurements | 3
tI
∴ Decrease in R (radius) by 2% world −
(∆ V ) ε 0β
increase g by 4% and decrease K 16. Q = I α e (given)
(rotational kinetic energy) by 4%. We know that
Option (b) is correct. Q = It
9. Heat (H) = 22 Rt −
tI
(∆V ) ε 0β
∴ t=α e
∴ Maximum error in measuring heat (H)
= 2 (2%) + 1% + 1% ⇒ [α ] = [ t ]
 tI 
= 6% and [β ] =  
Option (b) is correct. ( ∆V ) ε 0 

10. V = lbt ⇒  β  =  tI  =  I   1 
α  ( ∆V ) ε 0 t  ( ∆V )   ε 0 
= 12 × 6 × 2.45
1 1
= 176.4 =
[Resistance]  ε 0 
= 1764
. × 102 cm 3
β  = 1 1
= 2 × 10 cm 2
2 or
α  [ML2 T –3A –2 ] [M–1L–3T 4 A 2 ]
Option (b) is correct. 1
P = –1 = [velocity]
11. I = [L T]
4 πr2 1
i.e., Ir2 = constant =
[µ 0 ε 0 ]1/ 2
i.e., if r is increased by 2% the intensity
Option (a) is correct.
will decrease by 4%.
pq
Option (d) is correct. 17. a = 2 3
r s
12. Option (b) is correct.
 ∆a × 100 1  ∆p  1  ∆q 
4 =  × 100 +  × 100
13. V = πr 3  a  max 2  p  2 q 
3
 ∆r   ∆s
∆V ∆r +2  × 100 + 3  × 100
=3  r   s 
V r
1 1
= 3(1%) = (1%) + (3%) +2 (0.5%) + 3(0.33%)
2 2
= 3%
= 0.5% + 1.5% +1% + 1%
Option (c) is correct.
= 4%
14. a 3 = 6a2 (given) Option (c) is correct.
∴ a=6 18. Least count of main scale
⇒ V = 63 = 216 m 3 =
2 mm
= 0.5 mm
Option (b) is correct. 4
l least count of main scale
15. g = 4 π2 2 Least count =
T 50
 ∆g  ∆l ∆T = 0.1 mm
∴ × 100 = × 100 + 2  × 100
 g  max l  T  Zero error = − 30 × 0.01 mm
 1 mm  015
. = −0.3 mm
= × 100 + 2  × 10
 100 m   2003  (–ive sign, zero of circular scale is lying
above observed reading of plate thick)
= 01
. % + 01. % = 0.2 %
= 2 MSR + 20 CSR
Option (a) is correct.
4 | Mechanics-1
. mm) + (20 × 0.01 mm)
= (2 × 05 = observed reading − zero error
= 1 mm + 0.2 mm = 1.2 mm + 0.3 mm
= 1.2 mm. = 1.5 mm
Plate thickness (corrected reading) Option (d) is correct.
More than One Correct Options
1. Maximum percentage error in x :
= (% error in a) + 2 (% error in b) +3 (% error in c)
= 15%
Assertion and Reason
1. Least count of screw gauge Thus, assertion is true.
Pitch From the above relation we conclude that
=
Number of divisions of circular scale least count of screw gauge is inversely
Less the value of pitch, less will be least proportional to the number of divisions of
count of screw gauge leading to len circular scale.
uncertainty that is more accuracy in the Thus reason is false.
measurement.
Option (c) is correct.
Match the Columns
GM1M2 2
1. (a) F =  qvB sin θ 
r2 = 
 qB 
∴ GM1M2 = Fr2
= v2 sin2 θ
i. e., [GM1M2 ] = [MLT –2 ][L2 ]
 F2  –1 2 2 –2
= [ML3T –2 ] ∴  q2 B2  = [LT ] = [L T ]
 
∴ (a) → (q)
3 RT 3 pV 3 work ∴ (c) → ( r)
(b) = = GM e
M n nM (d) g=
R2e
 3 RT  ML2 T −2 
∴ = = [L2 T −2 ] GM e
 M   M  = gRe
 
Re
∴ (b) → (r)
2 GM e  –2
F2  F ⇒  R  = [LT ][L]
(c) 2 2 =    e 
qB  qB
= [L2 T –2 ]
∴ (d) → ( r)
2 Units & Dimensions
Vectors
Section I
Single Correct Option
Force 4. φ = Li
1. Pressure ( p) =
Area ∴ [φ ] = [ L ][ i]
[MLT −2 ]
∴ [ p] = = [ML2 T −2 A −2 ] [ A ]
[L2 ]
= [ML2 T −2 A −1 ]
= [ML−1T −2 ] Option (a) is correct.
Option (d) is correct.
5. Linear impulse ( I ) = F ⋅ ∆t
2. W = I 2 Rt [I] = [ML T −2 ] [T]
[ML2 T −2 ] [ML2 T −2 A −2 ]
∴ [R] = = …(i) = [ML T −1 ]
[A 2 T] [T ]
Option (c) is correct.
dI
V =L and W = Vq mm
dt 6. F = G 1 2 2
d
W dt
∴ L= [ F ][ d2 ] [ML T −2 ] [L2 ]
q dI [G] = =
[ m1 m2 ] [M2 ]
[ML2 T −2 ][T]
[L] = 2
= [ML2 T −2 A −2 ] = [M−1 L3 T −2 ]
[A T]
Option (c) is correct.
Using Eq. (i)
µ i ⋅i
[ L] 7. F= 0 1 2
[R] = 4π d
[T ]
[ML T −2 ][L]
L ∴ [ µ 0] = = [ML2 T −2 A −2 ]
i.e., [T ] =   [A 2 ]
 R 
Option (c) is correct.
Option (c) is correct.
[L]
3. F = 6π ηav 8. [ k] = = [T]
[L T −1 ]
[F ]
∴ [ η] = Option (c) is correct.
[ av ]
[ML T −2 ] [ML T −2 ]
= 9. [ a ] = = [ML T −3 ]
[L L T −1 ] [T]
[ML T −2 ]
= [ML−1 T −1 ] [ b] = = [ML T −4 ]
[ T2]
Option (d) is correct.
Option (c) is correct.
6 | Mechanics-1
10. E = hν 17. q = CV
[ML2 T −2 ] and V = iR
∴ [ h] = = [ML2 T −1 ]
[T −1 ] ∴ q = iCR
nh i t = iCR
Angular momentum ( J ) =
2π ⇒ [CR] = [ t ] = [M0 L0 T A 0 ]
[ J ] = [ h] = [ML2 T −1 ] Option (a) is correct.
Option (b) is correct. 1 q1 q2
18. F =
11. [Energy] = [ML2 T −2 ] 4 πε 0 r2
= [M][L T −1 ] 2 ∴ Unit of ε 0 = Newton-metre2 /coulomb2 .
∴ [Mass] = [ Ev −2 ] Option (b) is correct.
Option (c) is correct. nh
19. Angular momentum ( J ) =
1 Energy 2π
12. ε 0 E2 = Energy density =
2 Volume I = Σ mr2
2
1
 ε E =
2 [ML T −2 ] h 2π J / n mvr
∴ 0 3 ∴ = =
2  [L ] I Σmr 2
Σ mr2
−1
= [ML−1 T −2 ]  h = [L T ] = [T −1 ]
Option (b) is correct.  I   L 
 
13. [ a ] = [T 2 ] = Frequency
[T 2 ] Option (a) is correct.
[ b] =
[L ][ML−1 T −2 ] b
20. v = at +
∴  a  = [M T −2 ] t+c
 b 
[ c] = [ T ]
Option (b) is correct.  b 
t + = [v ]
14. Velocity gradient =
dv
 c
dx
or [ b] = [L T −1 ] [T] = [L]
[L T −1 ]
[Velocity gradient] = [ at ] = [v ] = [L T −1 ]
[L]
⇒ [a] = [L T −2 ]
= [T −1 ]
Option (a) is correct.
= [M0 L0 T −1 ] 2π
21. y = A sin  ( c t − x)
Option (a) is correct.  λ 
[Force] = [ML T −2 ] 2π 2π x
15. = A sin  c t −
[F]  λ λ 
∴ [Mass] =
[L T −2 ] 2πx
= θ (angle)
λ
= [F L−1 T 2 ]
∴ [ x ] = [ λ ] = [L ]
Option (a) is correct.
Further, y = A sin θ
16. Coefficient of friction (µ)
∴ [ A ] = [ y ] = [L ]
Limitting frictional force
= Option (a) is correct.
Normal force
22. [ X ] = [M−1 L−3 T 3 A 2 ]
∴ [µ] = [M0 L0 T 0 ] [T A 2 ]
=
Option (b) is correct. [ML2 T −2 ]
Units & Dimensions Vectors 7
2 1
[ t] [ i ] = (3) (5)  
=  2
[ Work ]
∴ X is resistance. [QW = i2 Rt] = 7.5
→ ^ Option (b) is correct.
23. F = 2 ^i − 3 ^j + 4 k → → →
→ ^
30. A + B = C
^ ^
r =3i+2j+3k → → → → → →
∴ ( A + B) ⋅ ( A + B) = C⋅ C
 ^ ^ ^
i j k → → → → → → → →
→ → →   or A ⋅ A + 2 A ⋅ B + B⋅ B = C⋅ C
∴ τ = r × F = 3 2 3 
→ →
 2 −3 4  or A2 + 2 A ⋅ B + B2 = C2
  → →
→ ^ or A ⋅B = 0
or τ = 17 ^i − 6 ^j − 13 k
→ →
or |A ||B|cos θ = 0
24. (0.5)2 + (0.8)2 + ( c)2 = 1
or cos θ = 0
or 0.25 + 0.64 + c2 = 1 π
or c2 = 1 − 0.89 or θ=
2
c = 0.11 Option (d) is correct.
Option (b) is correct. 31. Magnetic field intensity.
→ → → →
25. |A + B|= |A − B| Option (d) is correct.
→ → → → → → → → → → →
( A + B) ⋅ ( A + B) = ( A − B) ⋅ ( A − B) 32. P + Q = R
→ → → → → → → → → →
A2 + B2 + 2 A ⋅ B = A2 + B2 − 2 A ⋅ B (P + Q) ⋅ (P + Q) = R⋅ R
→ → → →
i.e., A⋅ B = 0 P2 + Q2 + 2 P⋅ Q = R2
→ → → →
∴ Angle between A and B = 90° 122 + 52 + 2 P⋅ Q = 132
→ → → → → →
26. ( A + B) ⋅ ( A − B) = 0 P⋅ Q = 0
→ → → → → → → → → → π
A ⋅ A + B⋅ A − B⋅ B − A ⋅ B = 0 ∴ Angle between P and Q =
2
A2 − B2 = 0 Option (b) is correct.
A=±B 33. Option (b) is correct.
→ →
→ → →
|A |= |B| 34. P + Q + R = 0
Option (d) is correct. → → →
→ →
∴ P+Q= −R
27. Work ( = F⋅ s ) is a scalar quantity. → → → → → →
or (P + Q) ⋅ (P + Q) = ( − R) ⋅ ( − R)
Option (d) is correct.
→ → → → → → → →
→ or P⋅ P + Q⋅ Q + 2 P⋅ Q = R⋅ R
28. Speed = |v|
→ →
Option (d) is correct. or P2 + Q2 + 2 P⋅ Q = R2 …(i)
→ →
29. |A |= 3, |B|= 5 and angle between A and B
→ →
Let Q2 = P2 and R = P 2
is 60°. Thus, Eq. (i) takes the form
→ → → → P2 + P2 + 2 PQ cos θ = 2 P2
∴ A ⋅ B = |A ||B|cos 60°
or 2 PQcos θ = 0
8 | Mechanics-1
or cos θ = 0 Q sin θ ( − Q sin θ)

or θ = 90° P + Q cos θ P − Q cos θ
=
→ → Q sin θ ( − Q sin θ)
∴ Angle between P and Q is 90° 1− ⋅
( P + Q cos θ) ( P − Q cos θ)
→ → →
P+Q+R=0 2 PQ sin θ
= 2


P+R= −Q
→ → P + Q2 cos 2θ
→ →
→ → → → → → This implies that angle between P + Q and
or (P + R) ⋅ (P + R ) = ( − Q) ⋅ ( − Q)
→ →
or P2 + R2 + 2 PR cos φ = Q2 P − Q will vary from 0 to π.
or 2 PR cos φ = Q2 − P2 − R2 Option (b) is correct.
or 2 PR cos φ = − R2 36. R2 = P2 + Q2 + 2 PQ cos θ
or 2P cos φ = − R for R = P = Q
or 2 P cos φ = − P 2 P2 = P2 + P2 + 2 PP cos θ
1 1
or cos φ = − or cos θ = −
2 2
∴ φ = 135° or θ = 120°
→ → Option (b) is correct.
∴ Angle between P and R is 135°.
→ →
37. W = F⋅ s
135°
→ = (3 ^i + 4 ^j) ⋅ (3 ^i + 4 ^j)
R →
P = 25 J
90° Option (b) is correct.

135° Q → → ^ ^ ^ ^ ^ ^
38. P⋅ Q = ( a i + a j + 3 k) ⋅ ( a i − 2 j − k)
Option (a) is correct. = a2 − 2a − 3
→ → → → → → → →
35. Angle ( φ) between P + Q and P − Q For P ⊥ Q, P⋅ Q = 0



→ +Q
i.e., a2 − 2a − 3 = 0
Q
θ P or ( a − 3)( a + 1) = 0
φ →
P ⇒ a=3
→ → φ'
–Q P
– Q→ Other value is − ive.
Option (d) is correct.
Q sin θ 39. If a vector makes angles α, β and γ with
tan φ =
P + Q cos θ the co-ordinate axes, then
→ → → cos2 α + cos2 β + cos2 γ = 1
Angle φ′ between P − Q and P
2 2 2
3 9  6 36  2  4
tan φ′ =
Q sin ( π + θ) Now,   = ,  = ,  =
7 49  7  49  7  49
P + Q cos ( π + θ)
9 36 4
− Q sin θ and + + =1
= 49 49 49
P − Q cos θ
∴ Option (a) is correct.
tan φ − tan φ′
tan[ φ + ( −φ′ )] = → ^
40. A = 4 i − 3 j and

B = 8 ^i + 8 $j
1 + tan φ tan φ′
→ → →
∴ A + B = C = 12 ^i + 5 ^j
Units & Dimensions Vectors 9

^ C 12 ^i + 5 ^j ∴ B=2N
C= =
|C| Option (c) is correct.
122 + 52
→ ^ ^
12 ^ 5 ^ 46. Angle between A = 2 i + 3 j
= i+ j
13 13 and

B = ^i + ^j
Option (b) is correct. → →
→ ^ → ^ A ⋅B 2+3
41. A = 2 i + 3 $j − 2 k, B = 5 i + n$j + k,
^ ^ θ= =

|A ||B|

2 + 32 ⋅ 2
2
→ ^
C = − i + 2$j + 3 k
^
5
→ → → =
∴ Vectors A , B, and C will be coplanar if 13 ⋅ 2
5
their scalar triple product is zero i.e., =
→ → → 26
(A × C ) ⋅ B = 0 →
Component of A along ^i + ^j
$ 
 i$ $j k
→ 5
  ^
2 3 −2 ⋅ (5 ^i + n ^j + k) = 0 C= (2 ^i + 3 ^j )

 
 26
−1 2 3 → 5
  |C|=
^ ^ 2
or (13 ^i − 4 ^j + 7 k) ⋅ (5 ^i + n ^j + k) = 0
Option (a) is correct.
or 65 − 4 n + 7 = 0 47. R2 = (3 P)2 + (2 P)2 + 2 × 3 P × 2 P cos θ
or n = 18 or R2 = 13 P2 + 12 P2 cos θ …(i)
Option (a) is correct. Further
42. Option (a) is correct. (2R)2 = (6 P)2 + (2 P)2 + 2 × 6 P × 2 P cos θ
→ → → →
43. (a + b) × (a − b) or 4 R2 = 40 P2 + 24 P2 cos θ …(ii)
→ → → → → → → → Dividing Eq. (ii) by Eq. (i),
=a ×a +b×a −a ×b−b×b
10 P2 + 6 P2 cos θ = 13 P2 + 12 P2 cos θ
→ → → →
=0−a × b−a × b−0 or 6 cos θ = − P
→ → → → or θ = 120°
= − 2(a × b) = 2( b × a )
Option (b) is correct.
Option (a) is correct. Q sin α
→ ^ 48. tan θ =
44. A = 3 ^i + 4 ^j + 5 k P + Q cos α
→ ^
B = 3 ^i + 4 ^j − 5 k
→ →
( A ⋅ B) R
cos θ = Q = 2P
→ →
|A ||B| θ = 90°
9 + 16 − 25
= 2 P
3 + 42 + 52
As θ = 90°, tan α = ∞
=0
∴ P + Q cos α = 0
⇒ θ = 90° P
i.e., cos α = −
Option (c) is correct. Q
45. A + B=7 P
=−
A − B=3 2P
10 | Mechanics-1
1 or − P × 16 = − 96
=−
2
P = + 6N
∴ α = 120°
∴ Q = 10 N
Option (a) is correct.
→ → Option (a) is correct.
49. A ⋅ B = 0 → → → →
51. |A × B|= 3 ( A ⋅ B)

A → → → →
→ → ⇒ |A ||B|sin θ = 3 |A | |B|cos θ
B ×C
⇒ tan θ = 3
⇒ θ = 60°
→ → → → → →
→ → ∴|A × B|2 = |A |2 × |B|2 + 2|A ||B|cos θ
B C
= A2 + B2 + 2 AB cos 60°
→ →
⇒ A⊥B …(i) = A2 + B2 + AB
→ → → →
A ⋅C = 0 |A × B|= [ A2 + B2 + AB] 1/ 2
→ → → → →
⇒ A⊥C …(ii) 52. C is perpendicular to both A and B
→ → →
From Eq. (i) and Eq. (ii), we conclude that C=A×B

A is perpendicular to the plane containing
→ →
B and C.

This implies that A is perpendicular to
→ → → →
B ⋅ C. A B
→ →
Option (c) is correct. ∴ C⋅ A = 0
50. P2 + Q2 + 2 PQ cos α = R2 → → →
or (A × B ) ⋅ A = 0
2 2 2
or P + Q + 2 PQ cos α = 8
Option (d) is correct.
or P2 + Q2 + 2 PQ + 2 PQ cos α − 2 PQ = 64 ^ ^ ^ ^ ^ ^
53. (2 i + 3 j + 8 k) ⋅ ( −4 i + 4 j + α k) = 0
or ( P + Q)2 + 2 PQ (cos α − 1) = 64
⇒ − 8 + 12 + 8α = 0
or (16)2 + 2 PQ (cos α − 1) = 64 1
∴ α=−
or 2 PQ(cos α − 1) = − 192 2
or PQ cos α − PQ = − 96 …(i) Option (c) is correct.
→ → → →
Q sin α 54. P+Q+R= 0
tan θ = =∞ (as
P + Q cos α →
P
θ = 90°) γ β
α →
∴ P + Q cos α = 0 → R
Q
Q cos α = − P …(ii)
→ → →
Using Eq. (ii) and Eq. (i), |P| |Q| |R|
If, = =
sin α sin β sin γ
P ( − P) − PQ = − 96
or − P ( P + Q) = − 96
Units & Dimensions Vectors 11
→ → →
55. OA + AB = OB 56. Using answer to questions no. 35, as angle
→ → → →
B (–2,6,4) between A + B and A − B is 90°

A2 + B2 cos 2θ = 0
Z
or A2 = − B2 cos 2θ
π Qθ = π 
or A2 = − B2 cos 2  
O  2  2
Y
or A2 = − B2 cos π
A (0, 3, –1)
X
or A2 = B2
→ → →
∴ AB = OB − OA ⇒ A=B
^ ^
= ( −2 i + 6 ^j + 4 k) − (0 ^i + 3 ^j − k)
^
Option (a) is correct.
^
= − 2 ^i + 3 ^j + 5 k
Option (c) is correct.
Match the Columns
→ → → → → → → → → → → →
1. (a) |A × B|= |A ⋅ B| ⇒ ( A + B) ⋅ ( A + B) = ( A − B) ⋅ ( A − B)
→ → → → → → → → → → → →
or |A ||B|sin θ = ± |A ||B|cos θ or A ⋅ A + B⋅ A + B⋅ A + B⋅ B
→ → → → → → → →
or tan θ = ± 1 = A ⋅ A − B⋅ A − A ⋅ B + B⋅ B
π 3π
⇒ θ= , → →
2 4 or 4 A⋅ B = 0
→ →
Thus, (a) → (r) (s). ⇒ A⊥B
→ → → →
(b) A × B = B × A (given) Thus, (c) → (q).
→ → → → → → →
or A × B = − (B × A ) (d) A + B = C
→ → → → → → → → → →
or |A ||B|sin θ = − |A ||B|sin θ or ( A + B) ⋅ ( A + B) = C⋅ C
→ → → → → → → → → →
or sin θ = − sin θ or A ⋅ A + B⋅ A + B⋅ A + B⋅ B = C⋅ C
or 2 sin θ = 0 → →
or A2 + 2 A ⋅ B + B2 = C2
⇒ θ = 0 rad
→ →
Thus, (b) → (p). or 2 A ⋅ B = 0 (Q A2 + B2 = C2 )
→ → → →
(c) |A × B|= |A × B| ⇒

A⊥B

→ → → →
or |A + B|2 = |A − B|2 Thus, (d) → (q).

Section II
Subjective Questions
(2 × 1011)(105 dyne) [ωt ] = [M0L0 T 0 ] ∴ [ω ] = [ T −1 ]
1. 2 × 1011 N/m 2 =
(104 cm 2 ) [θ ] = [M0L0 T 0 ]
= 2 × 1012 dyne/cm 2 E J
4. h = = = J-s
(72)(10−5 N) ν per sec
2. 72 dyne/cm = = 0072
. N/m
(10−2 m ) [ h] = [ML2 T –2 ][T] = [ML2 T –1 ]
3. [ a ] = [ y] = [ L] 5. [ b] = [ x2 ] = [ L2 ]
12 | Mechanics-1
 b  = [ p] → → →
 at  14. A + B = R (say) = 3 i$ + $j
→ → →
 b  L2  A − B = S (say) = i$ + 5$j
∴ [a] =   =  2 –3 
= [M–1T 2 ]
tp
   TML T  → →
Angle between R and S
1 2  1
6. St = ut + at  − u( t − 1) + a( t − 1)2 

R⋅S
→ →
3+5 8 4
 2   2  cos θ = = = =
1 2 1 RS 9 + 1 1 + 25 260 65
= u.1 − a(1) + at(1) = u + at − a
2 2
a 4 
= u + (2t − 1) ∴ θ = cos −1  
2  65 
Here t in second. Hence the given 15. Their dot product should be zero.
equation seems to be dimensionally $ should be
16. Ratio of coefficients of $i, $j and k
incorrect. But it is correct because 1 is
hidden. same.
7. LHS is dimensionless. While RHS has the 17. No solution is required.
dimensions [L−1 ]. → →

A ⋅B

18. Component of A along B = A cos θ =


8. LHS is dimensionless. Hence n = 0. B
→ → →
9. Just write the dimension of different $
19. A + B = R = 5$i − $j + k
physical quantities. →
10. E = km x n y a z . $ =R
R
R
Here k = a dimensionless constant → → →
$
20. A × B = C (say) = − 3 i$ + 8 $j + 2k
∴ [ E ] = [ m ]x [ n] y [ a ] z
∴ [ML T ] = [M]x [T –1 ] y [L] z
2 –2 → →
Now A ⋅ C = −6 + 8 − 2 = 0
∴ x = 1, y = 2 and z = 2 → →
x y z ∴ A⊥C
11. F = km v r
→ →
(k = a dimensionless constant) B⋅C = 0+8 −8 = 0
[ F ] = [ m ] [v ] y [ r] z
x


B⊥C

∴ [MLT –2 ] = [M]x [LT –1 ] y [L] z → →


Solving we get, 21. Area of parallelogram = |A × B|
x = 1, y = 2 and z = − 1 22. → →
R B sin θ
kmv2 B ⇒
∴ F=
r
θ →
12. (a) [ d] = [ F ]x [ L ] y [ T ] z A A
B cos θ
–3 –2 x y z
∴ [ML ] = [MLT ] [L] [T]
B
Equating the powers we get, B sin θ = R =
2
x = 1, y = − 4, z = 2
∴ θ = 30°
∴ [ d] = [FL–4 T 2 ] → →
∴ Angle between A and B = 180 − θ = 150°
Similarly other parts can be solved.
→ → →
23. A + B = ( 4 $i + 6$j ) + B = 10 $i + 9 $j
→ →
A ⋅B 6−2+8
13. cos θ = = →
AB 9 + 1 + 4 4 + 4 + 16 ∴ B = (6$i + 3 $j)
12 3
= =
336 21
Units & Dimensions Vectors 13
24. 25. 3P

A 4P
→ → 2P
C B
B C P

A →
R = P $i + 2 P $j − 3 P $i − 4 P $j
Applying sine law, we have = ( − 2 P i$ − 2 P $j )
a b
= 26. R2 = P2 + Q2 + 2 PQ cos θ
sin (180 − A) sin (180 − B)
c S2 = P2 + Q2 − 2 PQ cos θ
=
sin (180 − C) ∴ R2 + S2 = 2 ( P2 + Q2 )
a b c
or = =
sin A sin B sin C
3 Motion in One Dimension

Introductory Exercise 3.1


π
1. Suppose a particle is moving with = 2.0 cm × rad s −1
constant velocity v (along the axis of x). 30
π
Displacement of particle in time t1 = vt1 = cms −1
15
Displacement of particle in time t2 = vt2
As the tip would be moving with constant
∴ Displacement of the particle in the time speed.
interval ∆t ( = t2 − t1) π
Average speed = cms −1
= vt2 − vt1 15
= v ( t2 − t1) In 15 s the second hand would rotate
∴ Average velocity in the time interval through 90° i.e., the displacement of its tip
v ( t2 − t1) will be r 2.
∆t ′ =
( t2 − t1) ∴ Modulus of average velocity of the tip of
second hand in 15 s.
=v
r 2
Now, as the particle is moving with =
15
constant velocity (i.e., with constant speed
2 2
in a given direction) its velocity and speed = cms −1
at any instant will obviously be v. 15
Ans. True. 4. (a) Yes. By changing direction of motion,
2. As the stone would be free to acceleration
there will be change in velocity and so
under earth’s gravity it acceleration will acceleration.
be g. (b) (i) No. In curved path there will always
be acceleration. (As explained in the
3. A second hand takes 1 min i.e., 60s to
previous answer no. 3)
complete one rotation (i.e., rotation by an
angle of 2π rad). (ii) Yes. In projectile motion the path of
the particle is a curved one while
∴ Angular speed of second hand acceleration of the particle remains
2π rad constant.
=
60 s
(iii) Yes. In curved path the acceleration
π will always be there. Even if the path
= rad s −1
30 is circular with constant speed the
Linear speed of its tip = radius × angular direction of the acceleration of the
particle would every time be changing.
speed
Motion in One Dimension | 15
Circumference 4v 2
5. (a) Time speed = =
Speed 2πr
2π × 4 cm v
= = 8π = 25.13 s
1 cm / s 2 2 v2
=
(b) As particle is moving with constant π r
speed of 1 cm/s, its average speed in 2 2 (1)2
=
any time interval will be 1 cm/s. π 4
r 2
|Average velocity|= = 0.23 cm s −2
T/4
6. Distance = Speed × time
4r 2 2 2
= = speed D1 = v1t1
 2πr  π
 
 speed  D2 = v2 t2
2 2 D1 + D2 v1t1 + v2 t2
= cms −1 Average speed = =
π t1 + t2 t1 + t2
= 0.9 cms −1 ( 4 × 2) + (6 × 3)
=
v 2 2+3
|Average acceleration|=
T/4
= 5.2 ms −1
(where v = speed)

Introductory Exercise 3.2


1. Acceleration (due to gravity). 4. v = t 3/ 4 (given)
1 ds
2. st = u + at − a is physically correct as it = t 3/ 4 …(i)
2 dt
3
gives the displacement of the particle in tth +1
3/ 4 t4
second (or any time unit). ∴ s= ∫t dt = +c
3
st = Displacement in t seconds +1
4
− displacement in ( t − 1) seconds 4 7 /4
or s= t +c
1 1 7
= ut + at2  − u ( t − 1) + a( t − 1)2 
 2 
   2  i.e., s ∝ t7 /4
Differentiating Eq. (i) w.r.t. time t,
1 3
st = u + at – a d2 s 3 4 − 1
2 = t
dt2 4
Displacement
⇒ a ∝ t −1/ 4
Velocity Acceleration
5. Displacement (s) of the particle
1
Therefore, the given equation is s = ( 40 × 6) + ( − 10) 62
dimensionally incorrect. 2
3. Yes. When a particle executing simple = 240 − 180
harmonic motion returns from maximum = 60 m (in the upward direction)
amplitude position to its mean position Distance covered ( D) by the particle
the value of its acceleration decreases Time to attain maximum height
while speed increases.
16 | Mechanics-1
40 ∴ Ans. True.
= = 4s < 6 s
10 1 2
9. 125 = 0 ⋅ t + gt
It implies that particle has come back 2
after attaining maximum height (h) given ⇒ t = 25 s
by 125 m
u2 Average velocity = (downwards)
h= 5s
2g
= 25 m/s (downwards)
( 40)2
= = 80 m 10. v = 10 + 5t − t 2
…(i
2 × 10
)
∴ D = 80 + (80 − 60) dv
∴ a=
= 5 − 2t
= 100 m dt
6. v = 40 − 10t At t =2s
dx a=5−2×2
∴ = 40 − 10t
dt = 1 m/s2
or dx = ( 40 − 10t) dt From Eq. (i),
or x = ∫ ( 40 − 10t) dt dx
= 10 + 5t − t2
or x = 40t − 5t2 + c dt
As at t = 0 the value of x is zero. ∴ x = ∫ (10 + 5t − t2 ) dt
c=0 5t2 t 3
or x = 10t + − +c
∴ x = 40t − 5t2 2 3
For x to be 60 m. As, at t = 0 the value of x is zero
60 = 40t − 5t2 c=0
5 t3
or t2 − 8 t + 12 = 0 ∴ x = 10t + t2 −
2 3
∴ t = 2 s or 6 s
Displacement in time t Thus, at t = 3 s
7. Average velocity = 5 33
t x = (10 × 3) + (3)2 −
1 2 2 3
ut + at
= 2 = 30 + 22.5 − 9
t = 43.5 m
1
= u + at →
11. u = 2 i m/s
^
2

8. v2 = v1 + at a = (2 cos 60° ^i + 2 sin 60° ^j ) m/s2
∴ at = v2 − v1
= (1 ^i + 3 ^j) m/s2
Displacement in time t y
Average velocity =
t
1 2
v1t + at
2 a = 2 m/s2
=
t 60°
1 x
= v1 + at u = 2 m/s
2 → → →
v2 − v1 v=u+at
= v1 +
2
v1 + v2 = 2 ^i + (1 ^i + 3 ^j) 2
=
2 = 4 ^i + 2 3 ^j
Motion in One Dimension | 17

|v|= 42 + 12 = 2 7 m/s Part II Yes. As explained below.

→ → 1→ v = 2 ^i + 2t ^j implies that initial velocity of
s = u t + a t2
2 the particle is 2 ^i m/s2 and the acceleration
1 is 2$j m/s2
= (2 i) 2 + (1 ^i + 3 ^j) 22
^
2 → 1
∴ s (at t = 1 s) = (2 ^i × 1) + (2 ^j) 12
= 4 ^i + 2 ^i + 2 3 ^j 2
= (2 ^i + ^j) m
= 6 ^i + 2 3 ^j
→ 13. x = 2t and y = t2
∴ |s |= 36 + 12 2
x
∴ y =  
=4 3m  2

12. Part I v = (2 ^i + 2t ^j)m/s …(i) or, x2 = 4 y
→ (The above is the equation to trajectory)
dv
= 2 ^j x = 2t
dt
dx →
→ ∴ = 2 i.e., v x = 2 ^i
a = 2 ^j m/s2 dt
From Eq. (i), y = t2
→ dy →
ds ∴ = 2t i.e., v y + 2t ^j
= (2 ^i + 2t ^j ) dt
dt → → →
→ Thus, v = vx + v y
∴ s = ∫ (2 ^i + 2t ^j ) dt
→ = (2 ^i + 2t ^j) m/s
s = 2t ^i + t2 ^j + c
→
→ dv
Taking initial displacement to be zero. a= = 2 ^j m/s2
→ dt
s (at t = 1 s) = (2 ^i + ^j) m

Introductory Exercise 3.3


1. At t = t1
v
s

α t2 β β
t1 t

θ
0 t1 t2 φ t
Acceleration at t = t1 :
v = tan θ at1 = tan α
As θ < 90°, vt1 is + ive. As α < 90°, a t1 is + ive constant.
At t = t2 Acceleration at t = t2
vt 2 = tan φ at 2 = tan β
As φ > 90°, vt 2 is − ive. As β < 90°, at 2 is + ive constant.
Corresponding v-t graph will be 2. Let the particle strike ground at time t
velocity of particle when it touches ground
18 | Mechanics-1
1 Corresponding velocity-time will be
would be gt. KE of particle will be mg2 t2
2
Velocity (m/s)
i.e., KE ∝ t2 . While going up the velocity
will get − ive but the KE will remain. KE
will reduce to zero at time 2 t when the
particle reaches its initial position.

4 8 t (s)
KE
h 2
4. = tan θ =
( t − 2) (2 − 1)
2t time ⇒ h = 2 ( t − 2)
t = 2h
g 2
a (m/s )
2
1 1 2h
KE = mg2 t2 = mg2
2 2 g
= mgh θ (t–2)
1 2 θ t
3. Speed of ball (just before making first
collision with floor)
h
= 2gh = 2 × 10 × 80
= 40 m/s
Time taken to reach ground
2h 2 × 80
= = =4s Particle will attain its initial velocity i.e.,
g 10 net increase in velocity of the particle will
Speed of ball (just after first collision with be zero when,
floor) area under a-t graph = 0
40 (1 + 2) × 2 ( − h) ( t − 2)
= = 20 m/s + =0
2 2 2
Time to attain maximum height or 3 − ( t − 2)2 = 0
−20
t= = 2s or ( t − 2)2 = 3
−10
or t−2= ± 3
∴ Time for the return journey to floor = 2 s.
or t=2± 3
Speed (m/s) Ans . At time t = 2 + 3 s
(t = 2 − 3 not possible).

4 8 t (s)

Introductory Exercise 3.4


1. Relative acceleration of A w.r.t. B ∴ Relative displacement (i.e., distance
a AB = ( + g) − ( + g) = 0 between A and B) would be
1
2. Velocity of A w.r.t. B = v A − vB s = (v A − vB ) t + a AB t2
2
Motion in One Dimension | 19
or s = (v A − vB ) t sin 30° sin θ sin (180° − 30° + θ)
= =
tan θ = (vA–vB)
150 t 20t 500 × 103
1 sin θ
=
s 300 20
1
θ = sin −1  
 15
θ 1 sin (30° + θ) 15
1
0 t
Now = 3
θ
300 t 500 × 10
224
3. In figure, u = speed of boat 5000
or = sin 30° cos θ + cos 30° sin θ
v = speed of river flow 3t
B C 5000 1 224 3 1
= + ⋅
Boat sailing Actual 3t 2 15 2 15
direction path of boat 5000
or = 0.5577
u 3t
d = 400 m
5000
⇒ t=
A v 3 × 0.5577
Time to cross river = 2989 s
AB  BC AC  = 50 min
= = =  10 m
u  v u + v2 
2 5.
 A B
400 m
=
10 m/s a A = 1 m/s2 , aB = 2 m/s2
v A = 3 m/s, vB = 1 m/s
= 40 s
v Acceleration of A w.r.t. B = 1 − 2 = − 1 m/s2
BC = AB
u
2 m/s Velocity of A w.r.t. B = 3 − 1 = 2 m/s
= × ( 400 m )
10 m/s Initial displacement of A w.r.t. B = − 10 m
= 80 m At time relative displacement of A w.r.t. B
1
4. Let C be the point along which pilot s = − 10 + 2 t + ( −1) t2
2
should head the plane.
B or s = − 10 + 2t − 0.5t2
North For s to be minimum
Drifting due
km

ds
to wind =0
5 00

dt
or 2 − (0.5 × 2t) = 0
i.e., t = 2s
smin = − 10 + (2 × 2) − 0.5 × (2)2
30°

θ Speed = 150 m/s ∴


= − 10 + 4 − 2
A East = −8m
Minimum distance between A and
Wind/speed = 20 m/s B = 8 m.
Apply sine formula in ∆ ABC
20 | Mechanics-1
AIEEE Corner
Subjective Questions (Level 1)
1. (a) D = v1t1 + v2 t2 Net displacement
(b) Average velocity =
km km 1  Time taken
= 60 × 1 h + 80 × h
 h   h 2  − 60 m
=
= 100 km 6.1 s
(b) Average speed = − 9.8 m/s
100 km = 9.8 m/s (downwards)
= = 66.67 km/h.
1.5 h Σ vt
4. Average velocity =
2. (a) Displacement in first two seconds Σt
1 vt0 + 2 vt0 + 3vT
= ( 4) (2) + (6) 22 2.5v =
2 t0 + t0 + T
= 20 m ∴ 5t0 + 2.5T = 3 t0 + 3 T
20 m
∴ Average velocity = = 10 m/s or T = 4 t0
2s
5. (a) Average acceleration
(b) Displacement in first four seconds Final velocity − initial velocity
1 =
= ( 4) ( 4) + (6) 42 = 64 m ( 4 + 8) s
2
0−0
∴ Displacement in the time interval =
12
t = 2 s to t = 4 s
= 0 m/s2
= 64 − 20 v
(c) a = tan θ = max
= 44 m 4
44 m
∴ Average velocity = = 22 m/s.
2s v
3. Let the particle takes t time to reach vmax
ground.
θ
+20 m/s h
4s 8s 12s time

60 m vmax
∴ 4= (Q a = 4 m/s2 )
4
1 i.e., vmax = 16 m/s
∴ − 64 = 20 t + ( − 10) t2 Displacement of particle in 12 seconds
2
i.e., 5t2 − 20t − 64 = 0 = Area under v-t graph
v
t = 6.1 s = 12 × max
2
If the particle goes h meter above tower 12 × 16
before coming down =
2
0 = (20)2 + 2 ( −10) h = 96 m
⇒ h = 20 m Average velocity
Total distance moved Displacement (96 m) at 12 s
(a) Average speed = =
Time taken Time (12 s)
(20 + 20 + 60)
= = 16.4 m/s = + 8 m/s
6.1
Motion in One Dimension | 21
(b) As the particle did not return back 7. Position vector at t = 0 s
distance travelled in 12 s →
r1 = (1 ^i + 2 ^j) m
= Displacement at 12 s
Position vector at t = 4 s
∴ Average speed = 8 m/s.

6. (a) Radius ( R) of circle =
21
m r2 = (6 ^i + 4 ^j) m
22
(a) Displacement from t = 0 s to t = 4 s
∴ Circumference of circle = 2πR → →
22 21 = ( r 2 − r1 )
=2× × m
7 22 = (6 ^i + 4 ^j) − (1 ^i + 2 ^j)
= 6m
Speed (v) of particle = 1 m/s = (5 ^i + 2 ^j) m
∴ Distance moved by particle in 2 s = 2 m (5 ^i + 2 ^j) m
∴ Average velocity =
Thus, angle through which the particle 4s
moved
2 2π . ^i + 05
= (125 . ^j) m/s
= × 2π = = 120°
6 3 (b) Average acceleration
Magnitude of Average velocity Final velocity − Initial velocity
Magnitude of displacement =
= 4s
Time ( = 2 s)
→ (2 ^i + 10 ^j) − ( 4 ^i + 6 ^j)
vA =
4
A(t = 0 s)
−2 ^i + 4 ^j
=
B 120° 4
O
(t = 2s) . ^i + ^j) m/s2
= ( −05

vB (c) We cannot find the average speed as
the actual path followed by the particle
AB 2R sin 60°
= = is not known.
2 2
Uniform acceleration
3
=R (a) One dimensional motion
2
21 3 21 3 8. If at time t the vertical displacement
= = m/s between A and B is 10 m
22 2 44
1 2 1
(b) Magnitude of average acceleration gt − g( t − 1)2 = 10
2 2
→ →
vB − v A 
= 
 or t2 − ( t − 1)2 = 2
2s
  or t2 − ( t2 − 2t + 1) = 2
120° or 2t = 3
2 v sin
= 2 t = 1.5 s
2
→ →
9. The two bodies will meet if
[Q |v A |= |v B |= v = 1 m/s) Displacement of
Displacement of
= v sin 60° first after attaining = second before
3 attaining highest
= m/s2 highest point point
2
22 | Mechanics-1
1 2 1 1
v0 t − gt = v0 ( t − t0 ) − g( t − t0 )2 − s = ( at0 ) t + ( − a) t2
2 2 2
1 2 1 1
or 0 = − v0 t0 − g( t0 − 2t0 t) ∴ − at20 = ( at0 ) t − at2
2 2 2
1 2 or 2
− t0 = 2t0 t − t 2
or gt0 t = gt0 + v0 t0
2 or t − 2t0 t − t20 = 0
2
1
gt0 + v0
or t= 2 2t0 ± ( −2t0 )2 − 41( − t20 )
t=
g 2
t0 v0 2t0 ± 2t0 2
= + =
2 g 2
1
10. 5 = gt2 = t0 + t0 2 (− ive sign being
2 absurd)
⇒ t = 1s = (1 + 2) t0
5m Time interval = 2141
. t0
T
A 1s From the begining of the motion the point
O 25 m mass will return to the initial position
t second for A
after time 3.141t0 .
H W B u=0
12. 52 = u2 + 2 ( −10)15
E (t–1) second for B
⇒ u2 = 325
R v=0
A B
5 m/s
For A H
1 2 15m u
H = 0. t + gt
2
1 2
or H= gt …(i) (a) For H
2
For B 02 = u2 + 2( −10) H
1 i.e., 20H = 325
g( t − 1)2
H − 25 = …(ii)
2 or . m
H = 1625
1 2 1
or gt − g( t − 1)2 = 25 (b) For t
2 2
0 = 325 + ( −10) t
[Substituting value of H from Eq. (i)] 325
1 ∴ t=
g[ t2 − ( t − 1)2 ] = 25 10
2
= 1. 8 s
t2 − ( t2 − 2t + 1) = 5 a u a
13. At rest 15 m/s
2t − 1 = 5
⇒ t=3s x 60 m
Substituting t = 3 s in Eq. (i) 6.0 s
1 152 = u2 + 2a × 60
H = × 10 × 32 = 45 m (a) …(i)
2
and 15 = u + a × 6 …(ii)
1 2
11. s = at0 Forward motion Substituting the value of 6a from Eq. (ii)
2
in Eq. (i)
v = at0 Backward motion
225 = u2 + 20 (15 − u)
Motion in One Dimension | 23
2
i.e., u − 20 u + 75 = 0 14. a=+x a = –y

(u − 15)(u − 5) = 0
Starts Stops
∴ u = 5 m/s A t1 t2 B
4 km B
(15 m/s being not possible)
s1 s2
(b) Using Eq. (ii) 4 min
5
a = m / s2 Journey A to P
3
(c) u2 = 02 + 2ax vmax = 0 + xt1 …(i)
u2 (5)2 and v2max = 2xs1 …(ii)
i.e., x= = v
2a 2 × 5 ⇒ t1 = max
3 x
= 7.5 m Journey P to B
1 0 = vmax + ( − y) t2 …(iii)
(d) s = at2
2 2
and vmax = 2 ys2 …(iv)
1 5 v
= × × t2 ⇒ t2 = max
2 3 y
5 vmax vmax
= t2 …(iii) ∴ + = t1 + t2 = 4
6 x y
t(s ) 0 1 3 6 9 12 1 1 
v (m/s) 0 5/6 7.5 30 67.5 120 or vmax  +  = 4 …(v)
 x y
120 s (m) From Eq. (ii) and Eq. (iv)
v2 v2
90 s1 + s2 = max + max
2x 2y
v2max  1 1 
60 or 4= + …(vi)
2  x y 
30 Dividing Eq. (vi) by Eq. (v)
15 vmax = 2
t (s) Substituting the value of vmax in Eq. (v)
3 6 9 12 1 1
+ =2 (Proved)
Differentiating Eq. (iii) w.r.t. time t x y
5 15. Let acceleration of the particle be a using
v= t
3 2m
0 4 +ive
t(s ) 0 3 6 9 12 t=0s
t = 10 s t = 6 s x-axis
v (m/s) 0 5 10 15 20 v=0

v (m/s) v = u + at
0 = u + a6
20 u
∴ a=−
15
6
(a) At t = 10 s, s = − 2 m
10 1
−2 = u × 10 + a × 102
2
5
or −2 = ( −6a) 10 + 50a
3 6 9 12 or −10a = − 2
t (s)
or a = 0.2 m/s 2
24 | Mechanics-1
→ →
(b) v(at t = 10 s) = u + a 10 u = 0 m/s
= − 6a + 10a →
a 2 = − 4 ^j m/s2 (t = 2 s to t = 4 s) t2 = 2 s
= 4a
. m/s
= 08 (a) Velocity
→ → →
(c) Two or three dimensional motion v1 = u + a 1 t1
→ →
→ F 10 N north = 0 + (2 ^i) 2
16. a= =
m 2 kg
= 4 ^i
2
= 5 m/s , north → → → →
^ 2
v2 = v1 + (a 1 + a 2 )t2
= 5 j m/s
→ = 4 ^i + (2 ^i − 4 ^j ) 2
u = 10 m/s, east →
or v2 = (8 ^i − 8 ^j ) m/s
^
= 10 i m/s
(b) Co-ordinate of particle
→ → →
using v = u + a t → → → 1
s 1 = s 0 + u t1 + →a 1 t12
→ 2
v = 10 ^i + (5 ^j × 2) 1 ^ 2
= (2 ^i + 4 ^j) + (0)(2) + (2 i) 2
= 10 i + 10 j^ ^ 2
→ = 2 ^i + 4 ^j + 4 ^i
|v|= 10 2 m/s
→ = 6 ^i + 4 ^j
v = 10 2, north-east
→ → → 1
→ → 1→ s 2 = s 1 + v1 t2 + ( a1 + a2 ) t22
using s = u t + a t2 2
2
1
1 = 6 ^i + 4 ^j + ( 4 ^i) 2 + (2 ^i − 4 ^j) 22
= (10 i × 2) + (5 ^j)22
^
2
2
= 6 ^i + 4 ^j + 8 ^i + 4 ^i − 8 ^j
= (20 ^i + 10 ^j) m

|s|= 202 + 102 = 18 ^i − 4 ^j
Co-ordinate of the particle
= 10 5 m
20 [18 m , − 4 m ]
cotθ = =2 → → →
10 ^ ^
18. u = (2 i − 4 j) m/s, s 0 = 0 m
θ = cot−1 2 →
North
a = ( 4 ^i + ^j) m/s2
(a) Velocity
→ → →
v=u+at

s
^j = (2 ^i − 4 ^j) + ( 4 ^i + ^j)2
θ
O ^ East = (10 ^i − 2 ^j ) m/s
i
(b) Co-ordinates of the particle

s = 10 5 m at cot−1(2) from east to north. → → → 1→
s = s 0 + u t + a t2
→ ^ ^
2
17. s 0 = (2 i + 4 j) m → 1

= 0 + (2 i − 4 ^j) + ( 4 ^i + $j )22
^

a 1 = 2 ^i m/s2 (t = 0 s to t = 2 s) t1 = 2 s 2
Motion in One Dimension | 25
= 10 ^i − 2 ^j . ^i + 0.4 ^j
01
=
∴ Co-ordinates of particle would be 002
.
[10 m , − 2 m ] = 5 ^i + 20 ^j m/s

u = 8 ^j m/s

19. |a av|= 52 + 202

a = ( 4 ^i + 2 ^j) m/s2 = 206. m/s
20

s0 = 0
→ tan θ =
5

s = 29 ^i + n ^j i.e., θ = tan1( 4)
→ → → 1→ 2 Non uniform acceleration.
(a) s = s 0 + u t + at
2 21. x = 2 + t2 + 2t 3
→ 1 ^ (a) At t = 0, x = 2 m
29 ^i + n ^j = 0 + (8 ^j) t + ( 4 i + 2 ^j) t2
2 dx
(b) = 2t + 6t2
29 ^i + n ^j = 8 ^j t + 2 ^i t2 + ^j t2 dt
 dx 
Comparing the coefficients of ^i and ^j   = 0 m/s
 dt  t = 0 s
29 = 2t2 …(i)
d2 x
and n = 8 t + t2 …(ii) (c) = 2 + 12t
dt2
29
⇒ t= = 3.807 s ( a) t = 2 s = 2 + (12 × 2) = 26 m/s2
2
dv
Substituting value of t in Eq. (ii) 22. a= −v
dx
n = 8 × 3.807 + (3.807)2
= − (10) × 3
= 44.95
= − 30 m/s2
29
(b) Speed at t = s 23. s = t 3 − 9 t2 − 15t
2
ds
→ → → ∴ = 3 t2 − 18 t − 15
v=
v=u+at dt
dv
= 8 ^j + ( 4 ^i + 2 ^j)(3.807) ie, a= = 6t − 18
dt
= (2 × 3.807) ^i + {( 4 × 3.807) + 8 } ^j Acceleration (a) in the interval 0 ≤ t ≤ 10 s
will be maximum at t = 10 s
= (7.614 ^i + 23.228 ^j) m/s

a(at t = 10 s) = (6 × 10) − 18
speed = |v|= (7.614)2 + (23.228)2 = 42 m/s2
= 24.44 m / s 24. a = 3 − 2t

20. s 0 = 5 i m
^ dv
= 3 − 2t

dt
at t = 002 . ^i + 0.4 ^j) m
. s, s = (51
or ∫ dv = ∫ (3 − 2t) dt
→ →
→ s − s0 or v = 3 t − t2 + c
Average velocity v av =
002
. or v = 3 t − t2 + v0 [as at t = 0, v = v0 ]
^ ^ ^
(51
. i + 0.4 j) − (5 i) or (3 t − t2 + v0 ) dt
=
002
.
∫ ds = ∫
3 t2 t 3
or s= − + v0 t
2 3
26 | Mechanics-1
(a) Displacement at = Displacement at Now, at t = 2 s, v = 6 m/s
t = 0s t = 5s 62
∴ = ( 4 × 2) + C
3.52 53 2
0= − + v0 5
2 3 Thus, C = 10.
⇒ v0 = 5/6 = 0833
. s v2
i.e., = 4 t + 10 …(ii)
(b) v = 3 t − t2 + v0 2
Velocity at t ( = 500
. s) ∴ at t = 3 s,
= 3.5 − 52 + 0833
. v2
= ( 4 × 3) + 10
= 15 − 25 + 833 2
= − 9.167 m/s v2 = 44
v = 44 = 2 11 m/s
25. v = 3 t2 − 6t
2 Substituting above found value of v in Eq.
∴ ∫ ds = ∫3(3 t
− 6t) dt
(i),
or s = t − 3 t2 + c
2 11 a = 4
or s = t 3 − 3 t2 = t2 ( t − 3) 2
(3.5)2 (3.5 − 3) i.e., a=
(a) Average velocity = 11
3.5 = 0.603 m/s 2
= 3.5 × 05 . 27. According to question, the velocity of the
. m/s
= 175 particle varies as shown in figure.
(b) Distance covered v
20 m/s
v

1 O 30 m/s s
2 3 3.5 t v s
–3 ∴ + =1
20 30
2 2 3.5
= ∫ 0 (6t − 3 t ) dt + ∫ 2 (3 t2 − 6r) dt or 3v + 2s = 60
or 3v = 60 − 2s …(i)
= [3 t2 − t 3 ]20 + [ t 3 − 3 t2 ] 23.5
Differentiating above equation w.r.t. time t
= 3(2)2 − (2) 3 + (3.5) 3 − 3(3.5)2 − (2) 3 + 3(2)2 dv ds
3⋅ = − 2⋅
= 12 − 8 + 42875 . − 3675. − 8 + 12 dt dt
= 14.125 m or 3a = − 2v
14.125 60 − 2 s
(c) Average speed = or 3 a = − 2 ⋅   [using Eq. (i)]
3.5  3 
= 4.036 m/s 2
4 or a = − (60 − 2 s)
26. v= 9
a 2
= − (60 − 2 × 15) [at s = 15 m]
i.e., va = 4 …(i) 9
or 20
∫ v dv2 = ∫ 4 dt =−
3
m/s2
v
or = 4t + C
2
Motion in One Dimension | 39
1
S1 = gt 2 12. Please see answer to question no. 2 of
2 objective questions (level 1).
1 h um
= g T2 =
2 g ( mg + f )( mg − f )
h
= Now, if masses of the bodies are different,
2 the value of T2 (time to reach earth) will be
∴Assertion is true. different if f (air resistance) is same.
Reason is also true as assertion is based on Thus, Assertion is false further Reason is
this. correct as explained above.

Objective Questions (Level 2)


Single Correct Option
1. Applying sine formula in ∆ AOB ∠ B′ AO = 180° − (90° + α ) − (θ − 37° )
A = 90° − α − θ + 37°
= 127° − (α + Q )
v→
Nm
→ Comparing Eqs. (i) and (ii)
vr 37°
sin (α − 37° ) sin [127° − (α + θ )]
=
α cos 37° sin (θ − 37° )
B
sin α cos 37° − cos α sin 37°

m

cos 37°
–D

O

sin 127° cos (α + θ ) + cos 127° sin (α + θ )
vm

=
sin θ cos 37° − cos θ sin 37°
37° sin α − cos α tan 37°
4 3
AO OB cos (α + θ ) + sin (α + θ )
= = 5 5
sin (96° + 37° ) sin (α − 37° ) 4 3
vr vm sin θ − cos θ
= …(i) 5 5
cos 37° sin (α − 37° ) 3 4 cos (α + θ ) + 3 sin (α + θ )
or sin α − cos α =
Applying sine formula in 4 4 sin θ − 3 cos θ
3
or sin α − cos α
A' 4
4 [cos α cos θ − sin α sin θ ] + 3 [sin α cos θ + cos α sin θ ]
→ =
vr 4 sin θ − 3 cos θ
3
or sin α − cos α
4
4 [cos α − sin α tan θ ] + 3 [sin α + cos α tan θ ]
=
→ 37° 4 tan θ − 3
vm

θ O 3
or sin α − cos α
4
B' 7 7
4 [cos α − sin α ] + 3 [sin α + cos α ]
= 8 8
37° 7
4⋅ − 3
8
∆ AOB′ 3
AO OB′ or sin α − cos α
= 4
sin (θ − 37° ) sin [(α + θ )] 4 [ 8 cos α − 7 sin α ] + 3 [ 8 sin α + 7 cos α ]
vr vm =
= …(ii) 4
sin (θ − 37° ) sin [127° − (α + θ )]
40 | Mechanics-1
→ →
or 4 sin α − 3 cos α v1 − v 2 1 ^ 1 ^
∴ =− i+ j
= 32 cos α − 28 sin α + 24 sin α + 21 cos α → → 2 2
|v1 − v 2|
√50 → → → ^
r2 − r1 = r2 = 30 i
7
→ →
α |r2 − r1| = 30
1 → → → →
r2 − r1 ^ v1 − v 2
or 32 cos α + 21 cos α + 3 cos α ∴ =i ≠
→ → → →
|r2 − r1| |v1 − v 2|
= 28 sin α − 24 sin α + 4 sin α
or 56 cos α = 8 sin α ∴Option (c) is correct.
7 4. Displacement along y-axis at time t
tan α =
1 Y
From Eq. (i)
vr vm P v0
=
cos 37° sin (α − 37° )
a = 2t
vm cos 37° u=0
∴ vr = X
sin α cos 37° − cos α sin 37°
vm cos 37°
= a =2t
cos α cos 37° [tan α − tan 37° ]
dvy
vm =2t
= dt
3
cos α 7 −  or
 4  ∫ dvy =2 ∫ 2t dt
4 vm or vy = t + C1
= ⋅
25 cos α At t = 0, vy = 0 (given)
4 5 ∴C1 = 0
= × × 50
25 1 ∴ vy = t 2
= 32 dy
⇒ = t2
Option (b) is correct. dt
dv or 2
2.
dt
= − 4v + 8 ∫ dy = ∫ t + C2
t3
a = − 4v + 8 or y=+ C2
2
At the time the body acquires terminal speed
its acceleration (a) must be zero. At t = 0, Y = 0 (given)
Thus t3
∴ Y= …(i)
− 4v + 8 = 0 3
v = 2 m/s Displacement along x-axis at time t :
Terminal speed = 2 m/s.
x = v0t
Option (b) is correct. x
or t=
3. Particles will collide if v0
→ → → →
v1 − v2 r − r1 Substituting above value of t in Eq. (i)
= 2
→ → → → 3
|v1 − v 2| |r2 − r1| 1  x
Y=  
→ → ^ ^ ^ ^ ^ ^ 3  v0 
v1 − v 2 = (5 i + 10 j + 5 k ) − (10 i + 5 j + 5 k )
^ ^ Option (a) is correct.
= −5i + 5 j
→ → 5. t = αx 2 + βx
|v1 − v 2| = 5 2
Differentiating w.r.t. time t
Motion in One Dimension | 41
dx dx 7. Force = − kx 2
1 = α ⋅ 2x + β⋅
dt dt
k2 2
or v (β + 2α ⋅ x ) = 1 ∴ Acceleration = − x
2m
or v = (β + 2α ⋅ x )−1
dv 1 dx dv kx 2
∴ =− ⋅ 2α ⋅ ⇒ v =−
dt (β + 2α x )2 dt dx m
2α kx 2
a=− ⋅v or ∫ v dv = ∫ − dx
(β + 2α x )2 m
2α v2 k x3
=− ⋅v or =− +C
(β + 2α x )2 2 m 3

= − (2α v2 ) ⋅ v Now at x = a, v = 0
= − 2α v3 k a3
∴ C=
∴Retardation = 2α v3 m 3
Option (a) is correct. v2 kx 2 ka 3
Thus =− +
6. f = a − bx 2 m ×3 m ×3
dv ∴Velocity at x = 0
∴ v = a − bx
dx
2 ka 3
or ∫ v dv2 = ∫ (a − bx )2dx v=
3m
v bx
or = a⋅x − +C
2 2 Option (d) is correct.
As at x = 0 car is at rest C = 0 8. vx = 4 + 4t and vy = 4 t
v2 bx 2 dx dy
∴ = ax − …(i) ∴ = 4 + 4t and = 4t
2 2 dt dt
For v to be maximum. or
d  bx 2 
∫ dx = ∫ (4 + 4t ) dt
 ax −  =0 and ∫ dy = ∫ 4t dt 2
dx  2 
or x = 4t + 2t + C1
i.e., a − bx = 0 ∴ C1 = 1
a
or x= Thus x = 4t + 2t 2 + 1 …(i)
b
a and y = 2t 2 + C2
Substituting x = in Eq. (i),
b At t =0x =1
2
vmax a b a 2 and at t = 0, y = 2
= a ⋅ −  
2 b 2  b ∴ C2 = 0
2
a Thus y = 2t 2 + 2 …(ii)
=
2b Substituting value of t in Eq. (i) in terms of y
a
∴ vmax = using Eq. (ii) we won’t get relationship as
b mentioned in option (a), (b) or (c).
Car will come to rest when ∴Option (d) is correct.
bx 2 1
ax − =0 9. a = gt 2
2 2
2a = 4 × tan 120°
x=
b = 4 × [ − tan 60° ]
2a
∴Distance between two stations = = − 4 3 m/s 2
b
∴Magnitude of acceleration = 4 3 ms −2.
Option (a) is correct.
Option (a) is correct.
42 | Mechanics-1
1 2 3
10. x1 = gt or θ = sin −1
2 4
1 Option (c) is correct.
x1 + x2 = g (2t )2
2 1  αβ  2
1 1 14. Distance =  t
∴ x2 = g ⋅ 4t 2 − gt 2 2  α + β
2 2
3 2 1  1 × 4 2
x2 = gt ∴ 0=  t
2 2  1 + 4
x2 − x1 = gt 2
x2 − x1 ⇒ t = 500 s
⇒ t=
g = 22.36 s
Option (a) is correct.
Option (a) is correct.
15. Let BC = t′
11. y2 + x 2 = l2
dy dx v (m/s)
∴ 2y⋅ + 2x ⋅ =0
dt dt 24
dy x dx
or =−
dt y dt
x dx
=− O A B C t (s)
x tan 30° dt
= − 2 3 m/s 24
∴ =4
Option (c) is correct. t′
12. Maximum separation (xmax ) between the ⇒ t′ = 6 s
police and the thief will be at time t (shown ∴ OB = 50 s
in figure) Let OA = t
Police
108 km/h
∴ AB = 56 − t
v 1
Thief
90 km/h 1032 = [56 + (56 − t )] × 24
2
⇒ t = 20 s
t 24 m / s
∴maximum acceleration =
2s 7s time (t) 20 s

v 90 kmh
−1 = 1.2 m/s 2
t −2 == Option (b) is correct.
a 5 ms −2
90 1000 16. From ∆ OAB
= × s
5 3600 P
0.
2t

i.e., t =7s
1
xmax = [90 kmh × 7 s ] − × 5 s × 90 kmh –1
–1
2
90 × 1000 9
= × m
(4

3600 2

0.
2t

= 112.5 m
)m

Option (a) is correct. – 0.2t 60°


13. If meeting time is t Q B O
2v (3 – 0.2t) m
sin θ = u sin 30°
3
3 OA 2 + OB2 − AB2
i.e., sin θ = cos AOB =
4 2 ⋅ OA ⋅ OB
Motion in One Dimension | 43
2 2 2 2 2
∴ OA + OB − AB = OA ⋅ OB Now v = u + 2as
i.e., AB2 = OA 2 + OB2 − OA ⋅ OB ∴ 02 = (25)2 + 2( − 10) H
= (4 − 0.2t )2 + (3 − 0.2t )2 or H = 31.25 m
+ (4 − 0.2t )(3 − 0.2t ) Maximum height by ball as measured from
or AB = 16 + 0.04t − 1.6t + 9 + 0.04t 2
2 2 ground = 31.25 + 2 + 50
− 1.2t + 12 − 1.4t + 0.04t 2 = 83.25 m
or 2 2
AB = 0.12t − 4.2t + 37 Option (c) is correct.
For AB to be minimum 19. Displacement of ball w.r.t. ground during its
flight = H = 31.25 m
0.24t − 4.2 = 0
Option (d) is correct.
or t = 17.5 s
20. Displacement of ball w.r.t. floor of elevator
∴ ( AB)min = 0.12(17.5)2 − 4.2 × (17.5) + 37
2
at time t
= 36.75 − 73.5 + 37 1
s = 15 t + ( −15) t 2 + 2
( AB)2min = 0.75 m 2 2
= 7500 cm 2 s will be maximum, when
= 50 3 cm ds
=0
Option (d) is correct. dt
17. Velocity of ball w.r.t. elevator = 15 m/s i.e., 15 − 15t = 0
Acceleration of ball w.r.t. elevator i.e., t = 1s
1
Max. height attained by ball ∴smax = (15 × 1) + ( − 10)(1)2 + 2
2
= 12 m
+ H Option (a) is correct.
21. Let the particles meet at time t
15 m/s = vBE
i.e., displacement of the particles are equal
and which is possible when
5 m/s2 2m 10 m/s
vB B'
v
50 m
A uA
θ C
P P'
θ
B uB
= ( − 10) − ( + 5) = − 15 m/s 2 A'
vA
Final displacement of ball w.r.t. elevator
= −2m O 4 R Q
1 time (s)
∴ − 2 = 15 t + ( − 15) t 2
2
Area of ∆ ACB = Area of ∆ A′ B′ C
i.e., 15t 2 − 30t − 4 = 0
[Area OBCA′ θO being common]
30 + 900 + 4 × 4 × 15
∴ t= or Area of ∆ APC = Area A′ P′ C
2 × 15 1 1
= 2.13 s AP × PC = A′ P′ × P′ C
2 2
Option (a) is correct. or AP × PC = A′ P′ × P′ C
18. Velocity of ball w.r.t. ground (vBE ) or ( PC tan θ ) PC = ( P′ C tan θ ) P′ C
= (15 + 10) m/s or PC = P′ C′
= 25 m/s
44 | Mechanics-1
∴ OR = PC + P′ C′ = 2 PC = 8 s. 23. u A = 6 ms −1, u B = 12 ms −1
Option (c) is correct. and at t = 4 s common velocity = 8 ms −1
22. Area of ∆ A′ B′ O = Area of ∆ ABO To find velocity of A at t = 10 s
1 1 vA − 8 u A − 8
∴ (vB − vA ) ⋅ 4 = (u A − u B ) ⋅ 4 =
2 2 10 − 4 4
i.e., vB − vA = u A − u B vA − 8 6 − 8
or =
= 5 − 15 = − 10 6 4
⇒ vA − vB = 10 ms −1 ∴ vA = 5 ms −1
Option (b) is correct. Option (d) is correct.

More than One Correct Options


1. Q a = − α v1/ 2 …(i) t2
v=− + C1
dv 4
i.e., = − α v1/ 2
dt At t = 0, v = 16 m/s
or ∫ v−1/ 2 dv = − α ∫ dt t2
∴ v=− + 16 …(i)
2 v1/ 2 = − α t + C1 4
Now, at t = 0, v = v0 From above relation v is zero at
∴ C1 = 2 v10/ 2 t = 8s
⇒ 2 v1/ 2 = − α t + 2 v10/ 2 Options (a) is correct.
i.e., the particle will stop at From relation (i),
2 v1/ 2 ds t2
t= 0 =− + 16
α dt 4
Option (a) is correct.  t2 
∴ ∫ ds = ∫  − 4 + 16 dt
Option (b) is incorrect.
From Eq. (i), t3
i.e., s=− + 16t + C2
dv 12
v = − α v1/ 2
dt t3
s=− + 16t [As at t = 0, s = 0]
1/ 2 12
or ∫v dv = − α ∫ dx
2 3/ 2 At t = 4 s
or v = − α + C2
3 s = 58.67 m
2 Option (b) is correct.
As at x = 0, v = v0, C2 = v03/ 2
3 The particle returns back at t = 8 s
2 3/ 2 2
∴ v = − αx + v03/ 2 From relation (ii)
3 3
83
i.e., when the particle stops s8 = − + (16 × 8) = 85.33 m
12
2 3/ 2
x= v0 103
3α s10 = − + (16 × 10) = 76.66 m
12
Option (d) is correct.
Distance travelled in 10 s
Option (c) is incorrect. = s8 + ( s8 − s10 )
2. a = − 0.5 t (m/s 2) = (2 × 85.33 ) − 76.66
dv t
=− = 94 m
dt 2
Option (c) is correct.
1
or ∫ dv = − t dt
2∫ Velocity of particle at t = 10 s
Motion in One Dimension | 45
2
(10) Position of A relative to B :
v10 = − + 16 → → →
4
S AB = S A − S B
= − 25 + 16
^ ^ ^ ^ ^
= − 9 m/s = (3 i + 4 j − 20t j) − (32t i + 24t j)
∴Speed of particle at t = 10 s is 9 m/s. ^ ^
= (3 − 32t ) i + (4 − 44t ) j
Option (d) is correct.
→ Option (b) is correct.
3. |v|is scalar.
Option (d) is incorrect.
∴Option (a) is incorrect.

5. Case I.
dv → v = (Let)
=a (by definition).
dt t1 t2 Average
velocity = v1
∴Option (b) is correct. Rest a1 – a2 Rest
v2 is scalar. S1
∴Option (c) is incorrect.
→ v = a1t1
v
is v^ (unit vector) v = a 2t2

|v| 1 1
s1 = a1t12 + a 2t22
2 2
d v^ ^ →
∴ =a≠a Case II.
dt
∴Option (d) is incorrect. v' = 2v
t1 t3 Average
4. North 2a1 – a2
velocity = v2
Y (km) A (3, 4) Rest
S2
20 km/h
h
m/
4 0k v′ = 2a1t1
37°
B East = 2v
X (km) 2v = a 2t3
2 ( a 2t2 ) = a 2t3
⇒ t3 = 2t2
South 1 1
s2 = (2a1 ) t12 + a 2t32
→ ^
2 2
vA = − 20 j kmh −1 1
= a1t1 + a 2(2 t2 )2
2
→ ^ ^ 2
vB = (40 cos 37° ) i + (40 sin 37° ) j kmh −1
= a1t12 + 2 a 2t22
^ ^
= (32 i + 24 j) kmh −1 2s2 = a1t12 + a 2t22
→ → → ∴ s1 > 2s1 …(i)
vAB = vA − vB
4s1 = 2a1t12 + 2a 2t22
^ ^ ^
= ( − 20 j) − (32 i + 24 j) ∴ s2 < 4s1 …(ii)
^ ^ Combining Eqs. (i) and (ii),
= − 32 i − 44 j
2s1 < s2 < 4s1
Option (a) is correct.
Option (d) is correct.
Option (c) is incorrect. s1
At any time In Case I. vav =
t1 + t2
→ ^ ^ ^
SA = 3 i + 4 j + ( − 20 j) t 1 1
a1t12 + a 2t22

^ ^ i.e., v1 = 2 2
S B = (32 i + 24 j) t
t1 + t2
46 | Mechanics-1
( a1t1 ) t1 + ( a 2t2 ) t2 6
= 8. a=− s+6
2 ( t1 + t2 ) 30
vt1 + vt2 v 5a = − s + 30
= =
2 ( t1 + t2 ) 2 dv
5 ⋅v = − s + 30
In Case II. ds
s2
vav = or ∫ 5v dv = ∫ ( − s + 30) ds
t1 + t3
a1t12 + 2a 2t22
i.e., v2 = 5v2 s2
t1 + t3 or =− + 30s + c1
2 2
( a1t1 ) t1 + a 2t2 (2 t2 )
= 5 2 S2
t1 + 2 t2 or v =− + 30s …(i)
2 2
vt + v (2 t2 )
= 1
t1 + 2t2 [C1 = 0 as at s = 0 particle is at rest]

= v = 2v1 Substituting s = 10 m in Eq. (i),


Option (a) is correct. 5 2 (10)2
v =− + 300
Option (b) is incorrect. 2 2
6. If the particle’s initial velocity is + ive and i.e., v = 10 m/s
has some constant − ive acceleration the
∴Option (b) is correct.
particle will stop somewhere and then
return back to have zero displacement at From Eq. (i) v to be maximum
same time t ( > 0). − s + 30 = 0
Also if particle’s initial velocity is − ive and
has some constant + ive acceleration the s = 30
2
particle will stop somewhere and then 5 2 (30)
∴ vmax = − + (30 × 30)
return back to have zero displacement at 2 2
same time t ( > 0). 5 2
∴Options (b) and (c) are correct. or vmax = 450
2
and options (a) and (d) are incorrect.
or vmax = 180 m/s
7. F = α t Option (c) is correct.
⇒ ma = α t
α 9. If particle’s path is
or a= t …(i) (i) straight with backward motion
m
∴Graph between a (acceleration) and time ( t ) (ii) not straight somewhere.
will be as curve 1. Distance moved will be greater than the
∴Option (a) is correct. modulus of displacement

From equation ∴ |v av | < vav
dv α Option (a) is correct.
= ⋅t
dt m
α If particle returns to its initial position, the

∴ ∫ dv = m ∫ t dt value of v av will be zero while its average
α t2 speed (vav ) will not be zero.
i.e., v= ⋅ + c ∴Option (c) is correct.
m 2
1
∴Graph between velocity (v) and time ( t ) will 10. If u = 0, v = at and s = at 2
be as curve 2. 2
Option (b) is correct. ∴v-t graph will be as shown in (a).
s-t graph will be as shown in (d).
Motion in One Dimension | 47
11. Swimmer will reach point B t = t1 + t2 + t3
B 2l l 2l
→ = 1 + 2 + 3
u v v v
b v cos θ l1 l2 l3 l1 l3
θ = + + + +
v v v v v
v sin θ u l 1
= + [ l1 + l2 ]
v v
if v sin θ = u
l 1 1 1
i.e., v>u = +  α t12 + β t22 
v v 2 2 
Option (c) is correct. 1 α t12 1 β t22 
l
Time to cross the river = + v 2
+ 
v 2 v 2 v2 
b b
t= = [as v sin θ = u]
v cos θ v − u2
2 l 1 α t12 1 β t22 
= + v 2 2
+ 
v 2 α t1 2 β 2t22 
Option (b) is correct.
b l v 1 1
tmin = = +  + 
v (cos θ )max v 2 α β 
b Option (b) is correct.
=
v For t to be minimum.
Option (a) is correct. d  l v  1 1 
+  +  =0
12. At time T, particle’s velocity change from − dv v 2 α β 
–ive to + ive. l 1 1 1
− +  + =0
Option (a) is correct. v2 2 α β 
As slope v-t is same throughout, particle’s l α+β
acceleration is constant. or =
v2 2αβ
Option (b) is correct. 2 lα β
As net area under the curve is zero. or v=
α+β
  vT   − vT  
 Anet =  2  +  2  , displacement of Option (d) is correct.
 
particle at time 2T is zero. 14. x = t 2 and y = t 3 − 2 t
dx dy
Option (c) is correct. = 2t and = 3 t2 − 2

dt dt
Initial speed = |v| = v dx dy
At t = 0, = 0 but = −2
Final speed = |v| = v
→ dt dt
∴at t = 0 particle is moving parallel to y-axis.
Option (d) is correct.
Option (a) is correct.
13.
Option (d) is incorrect.
t1 t2 t3 → ^
Velocity O v v O At t = 0, v = −2 j
Accel : (α) (− β) d 2x
l1 l2 l3 =2
dt 2
l
d2 y
and = 6t
v = αt1 v = βt3 dt 2
1 2 1 → ^
l1 = αt1 , l2 = vt2, l3 = βt32 At t = 0, a =2i
2 2 → →
1 1 i.e., v⊥a
= vt1 = vt3
2 2 Option (b) is correct.
48 | Mechanics-1
2 S1 + S2
At t = : Speed (v) at :
3 2
dx 2 dy 2  S + S2 
=2 but = 3   − 2 v2 = 2a  1 
dt 3 dt 3  2 
∴Particle is moving parallel to x-axis. = a ( S1 + S1 )
Option (c) is correct. = a (25a t12 )
15. ∴ v = 5a t1
Speed 2 m/s 14 m/s or v = 10 m/s
X P Y Option (a) is correct.
Starting (Mid point) Time t to reach P from T
point of X and Y
S1 10 = 2 + at
t1
S2 8
t2 ⇒ t= …(i)
S1 + S2 a
2
Time t′ to reach Y from P
at1 = 2 and at2 = 14 14 = 10 + at′
∴ t2 = 7t1 4
∴ t′ = …(ii)
1 1 a
S1 + S2 = at12 + at22 Comparing Eq. (i) and Eq. (ii),
2 2
1 t = 2t′
= a ( t1 + t22 )
2
2 Option (c) is correct.
1
= a [ t12 + ( 7t1 )2 ]
2
= 25a t12
Match the Columns
1. (a) (b)

a (acceleration) a (acceleration)
+ +

– t

v (velocity) v (velocity) v (velocity)


+ t
v (velocity)

+ t

+ +
t
– t –
Speed increasing Speed decreasing Speed decreasing Speed increasing

(a) (a) → (r) (s). (b) (b) → (r) (s).


(c) s = kt 2
ds
v= = 2kt
dt
Motion in One Dimension | 49
∴v must be increasing with time. ∴(c) → (s).
⇒ speed must also be increasing with time. → ^ → ^ ^
(d) v = 2 i and a = −2i + 2 j
(c) → (p). → → →
ω =v + at
(d) Slope is + ive and decreasing.
^ ^ ^
∴Velocity must be decreasing with time. =2i −2ti + 2t j
⇒ speed must also be decreasing with time. ^ ^
= (2 − 2 t ) i + 2 t j
(d) → (q). →
→ ^ |ω| = (2 − 2 t )2 + (2 t )2
2. (a) v = − 2 i

→ At t = 0 |ω| = 2
∴Initial speed = |v| = 2
1 →

a = −4 j
^ t= |ω| = 2
2
→ →
Velocity (ω ) at time t t =1 |ω| = 2
→ → →
ω =v + at →
t =2 |ω| = 20
^ ^
= − 2 i − 4t j
∴(d) → (s).

Speed at time t = |ω| = ( − 2)2 + ( − 4 t )2 3. (a) From A to B, v is increasing and area is
+ ive.
At t > 0
→ → ∴ (a) → (p).
|ω| > |v|
(b) From B to C, v is increasing and area is
Speed increasing. + ive.
∴ (a) → (p). ∴ (b) → (p).
→ ^
(b) v = 2 i (c) From C to D, v is decreasing while area is

+ ive.
|v| = 2 ∴ (c) → (q).
→ ^ ^ →
a =2i + 2 j (d) From D to E, v is − ive and |v|is
→ → → increasing, area is −ive.
Using ω = v + a t
∴ (d) → (r).
→ ^ ^ ^
∴ ω =2i + 2ti + 2t j 4. (a) Displacement = Area


|ω| = (2 + 2 t ) + (2 t ) 2 2 10 × 4
= = 20 unit
→ → 2
As t > 0, |ω| > |v| 20
∴ Average velocity = + = + 5 unit
(a) → (p). 4
→ ^ → ^
(c) v = − 2 i and a = + 2 i i.e., (a) → (r).
1 ×5

Using ω = v + a t
→ → (b) Displacement = + 20 − = + 17.5 unit
2

ω = −2i + 2ti
^ ^ + 17.5 unit
∴ Average velocity =
→ ^ 3
or ω = (2 t − 2) i
= 5.83 unit

∴ |ω|= |2 t − 2| ∴ (b) → (s).
At t = 0 s Speed = |ω| = 2 m/s
→  2 × ( − 10)
(c) Displacement = + 20 +  

 2 
t =1s |ω| = 0 m/s
= + 10 unit

t =2s |ω| = 2 m/s + 10
∴ Average velocity =
→ 6
t =3 s |ω| = 4 m/s
i.e., (c) → (s).
50 | Mechanics-1
(d) Rate of change of velocity at t = 4 s i.e., t =1s
→ 0 − 10 Y = 4 − 4t + t 2
a= = −5
4 −2 dy
= − 4 + 2t
∴Rate of change of speed at dt
t = 4 s would be 5. = − 4 + (2 × 1) [at t = 1 s]
i.e., (d) → (r). = − 2 unit
5. (a) x = − 20 + 5t 2 ∴(a) → (q).
0 = − 20 + 5t 2 (b) Y = 4 − 4t + t 2
0 = 4 − 4t + t 2 [For particle to cross x-axis]
⇒ t =2s
i.e., t =2s
i.e., (a) → (r).
x = 1 − 2t + t 2
2
(b) x = − 20 + 5t dy
= − 2 + 2t
dx dt
= 10 t
dt = − 2 + (2 × 2) [at t = 2 s]
d 2x = + 2 unit
= 10
dt 2 ∴(b) → (p).
Velocity will be numerically equal to dx
(c)   = − 2 unit
acceleration at t = 1 s.  dt  t = 0 s
Thus (b) → (q). dy
and   = − 4 unit
(c) At t = 0 s x = − 20 m  dt  t = 0 s
t =1s x = − 15 m ∴ Initial velocity of particle
t =2s x =0m
= ( − 2 )2 + ( − 4 )2
t =3 s x = + 25 m
= 2 5 unit
Particle is always moving along + ive x-axis.
i.e., (c) → (s).
∴ (c) → (s). d 2x
dx (d) 2 = 2
(d) v = = 10 t is zero at t = 0 s. dt
dt d2 y
∴ (d) → (p). and =2
dt 2
6. (a) x = 1 − 2t + t 2 ∴ Initial acceleration of particle = 2 2 unit
0 = 1 − 2t + t 2 [for particle to cross y-axis] i.e., (d) → (s).
Motion in One Dimension | 27
Graphy
28. OA : slope is + ive and increasing. 30. Case I
s
C
v m = tan θ s

B θ
D ⇒ s0
A t
t
v0
O t a

a = tan θ
∴ velocity is + ive and acceleration is + ive.

AB : slope is + ive and constant
∴ velocity is +ive and acceleration is zero.
BC : sope is + ive and decreases.
∴ velocity is + ive and increasing. v = mt − v0 (st-line)
CD : slope is –ive and increasing ∴ ∫ ds = ∫ (mt2 − v0 ) dt
∴ velocity is − ive and acceleration is − ive. t
i.e., s= m⋅ − v0 t + s0 (Parabola)
29. 2
v 1
or s = s0 − v0 t + at2 (Q m = a)
v
2
M2 M1 dv
Further, a = = tan θ
θ
dt
θ
O t O t As for 0° ≤ θ < 90°

v tan θ is + ive, a is + ive.


M4 Case II
M3
θ v s
t θ

s0
t t
In M1 and M 3 : 0° ≤ θ < 90°
a
∴ slope is + ive i.e., acceleration is + ive.
In M2 and M4 : 90° < θ ≤ 180° ⇒
∴ slope is − ive i.e., acceleration is − ive.
(a) M1 : Magnitude of velocity is increasing. t
M2 : Magnitude of velocity is decreasing.
v = k (constant) (st line parallel to
M 3 : Magnitude of velocity is increasing. time-axis)
M4 : Magnitude of velocity is decreasing. ⇒ ∫ ds = ∫ k dt
Ans : M1 and M 3. s = kt + s0
(b) P → M1 dv
Further, a= =0
Q → M2 dt
R → M3
S → M4
28 | Mechanics-1
Case III. Time-acceleration graph
v s time slope acceleration
v0 s0 0-2s 5 5 m/s 2
⇒ 2-4s zero 0 m/s 2
4-6s −5 − 5 m/s 2
θ 6-8s −5 − 5 m/s 2
t t 8 - 10 s +5 + 5 m/s 2

a
a


t 5

a = tan θ
v = v0 + mt 2 4 6 8 10 t
∴ ∫ ds = ∫ (v0 + mt2) dt
t
i.e., s = v0 t + m + s0
2
1 31. a = slope of v- t graph.
or s = s0 + v0 t − at2 (Q m = − a)
2
S = area under v − t graph.
dv
Further, a = = tan θ Corresponding graphs are drawn in the
dt
answer sheet.
As for 90° < θ ≤ 180° s Net area A1 − A2
32. Average velocity = = =
tan θ is − ive, a is − ive. t Time t
Time-displacement graph d A1 + A2
Average speed = =
Time Area Initial Net t t
under the Displacement displacement
graph 33. Average acceleration
at 0 s 0m − 10 m − 10 m
v f − vi −10 − 20
= = = −5 m / s2
2s 10 m − 10 m 0m time 6
4s 30 m − 10 m + 20 m v f − vi = Area under a- t graph.
6s 40 m − 10 m + 30 m
8s 30 m − 10 m + 20 m 34. (a) Acceleration = slope of v- t graph.
10 s 20 m − 10 m + 10 m 35. (b) rf − ri = s = area under v- t graph.

(c) Equations are written in answer sheet.


s
Relative Motion
+ 30 m
35. (a) Acceleration of 1 w.r.t. 2
+ 20 m = ( − g) − ( − g)
= 0 m/s2
+ 10 m (b) Initial velocity of 2 w.r.t. 1
= ( +20) − ( −5)
2 4 6 8 10 t
= 25 m/s
(c) Initial velocity of 1 w.r.t. 2
– 10 m
= ( − 5) − ( + 20)
= − 25 m/s
Motion in One Dimension | 29
1 1
∴ Velocity of 1 w.r.t. 2 at time t  = s 0 = ( + 10) + ( −16) t + ( +10) t2
 2  2
 1
= ( − 25) + (0)   or 5t2 − 16t − 10 = 0
 2 t = 3.65 s
= − 25 m/s Position of elevator when it meets ball
(d) Initial relative displacement of 2 w.r.t. 1 = 5 + (2 × 3.65)

S 0 = − 20 m = 12.30 m level
→ → → 1→ 38. time = 0 time =ts
Using S = S 0 + u rel t + a rel t2 , 2.2 m/s2
2 3.5 m/s2
as at time t (= time of collision of the T T
A
particles) the relative displacement of 2
→ →
w.r.t. 1 will be zero (i.e., S = O) A
x
→ 1
O = ( − 20) + ( − 25) ⋅   60 m
 2
1
⇒ t = 0.85 s (a) (2.2) t2 = 60 …(i)
2
36. Let length of escalator L (= 15 m)
L ⇒ t = 7.39 s
walking speed of man = 1 2
90 (b) (3.5) t = 60 + x …(ii)
L 2
Speed of escalator =
60 Dividing Eq. (ii) by Eq. (i),
60 + x 3.5
Time taken by man walking on a moving =
L 60 2.2
escalator = x 1.3
L L or =
+ 60 2.2
90 60
= 36 s x = 35.5 m
Required time has been found without (c) At the time of overtaking
using the actual length of the escalator. Speed of automobile = (3.5) (7.39)

37. S 0 = Initial displacement of elevator w.r.t. = 25.85 m/s
ball = + 10 m. Speed of truck = (2.2) (7.39)
Relative velocity of elevator w.r.t. ball = 16.25 m/s
= (2) − (18) = − 16 m/s
39. Let, acceleration of lift = a (upward)
Accelerator of elevator w.r.t. ball
= (0) − ( − 10) ∴ acceleration of thrown body w.r.t. lift
= 10 m/s2 = ( − g) − ( + a)
= − ( g + a)
18 m/s
If time of flight is t, using

10 m
vrel = urel + arel t
12 m 2 m/s
Elevator
( − v) = ( + u) + { − ( a + g)} t
5m ⇒ ( a + g) t = 2 u
2u
or a= −g
→ → 1 t
Using S = S 0 + urel t + a t2
2 rel
30 | Mechanics-1
40. A 2 4
B =
OAB = OA = 20 m sin ( 45° − θ) sin 45°
 1  
20 m 2√2 m/s θ = sin −1   − 45°
45°   2 2 
O 42. Let pilot heads point R to reach point Q
2 2
OB = (OA) + ( AB) North
R Q
= (20)2 + (20)2
= 20 2 m 1000 km
Wind θ
Speed along OB is 2 2 m/s
20 2 m 200 km/h East
∴ Time taken to reach B = P
2 2 ms −1 South
= 10 s RQ
sin θ =
41. In ∆ OPQ, PR
A B 200 t
=
River flow 500 t
P
θ Q ⇒ θ = sin −1 (0.4)
45° ( PR)2 = ( RQ)2 + ( PQ)2
O
or (500 t)2 − (200 t)2 = (1000)2
→ 1000
OP = |v br | or t=
→ (500)2 − (200)2
OP = |v r |
10 10
PQ OP = = h
= 25 − 4 21
sin ∠ POQ sin ∠ OPQ

Objective Questions (Level 1)


Single Correct Option
1. As the packet is detached from rising  mg + f 
0=u−   T1
balloon its acceleration will be g in the  m 
downward direction. um
T1 = …(i)
Option (b) is correct. mg + f
2. While going up : Using v2 = u2 + 2as
 mg + f 
Stops 02 = u2 − 2  s
 m 
mu2
i.e., S=
2( mg + f )
S While coming down
u
1  mg − f  2
s=   T2
2 m 
f (Air resistance) mu2 1  mg − f  2
mg or =   T2
2 ( mg + f ) 2  m 
Motion in One Dimension | 31
um 8. For first one second
⇒ T2 = …(ii)
( mg + f ) ( mg − f ) 1
2= a × 12
Using Eq. (i) and Eq. (ii) 2
T2 mg + f ⇒ a = 4 m/s 2
=
T1 mg − f Velocity at the end of next second
⇒ T2 > T1 v = ( 4) × (2)
Option (c) is correct. = 8 m/s
3. Angular speed (ω) of seconds hand Option (b) is correct.

= rad s −1 9. x = − 3 t + t 3
60
Speed of the tip of seconds hand ∴ displacement at time t (= 1 s)

v=
π
× 1 cm/s [Q v = rω and = − 3(1) + (1) 3 = − 2 m
30
and displacement at time t (= 3 s)
r = 1 cm]
As in 15s the seconds hand rotates = − 3(3) + (3) 3 = 18 m
through 90°, the change in velocity of its And as such displacement in the time
tip in 15 s will be interval (t = 1 s to t = 3 s)
π 2
=v 2= cm s −1 = (18 m ) − ( − 2 m )
30
= + 20 m
Option (d) is correct.
5+5 Option (c) is correct.
4. Average speed = = 40 ms −1
5 5 10. Acceleration a = bt
+
30 60 dv
Option (c) is correct. ∴ = bt
dt
5. Relative velocity of boat w.r.t. water
or ∫ dv = ∫ bt dt
= (3 ^i + 4 ^j) − ( − 3 ^i − 4 ^j) = 6 ^i + 8 ^j
1 2
or v= bt + C
Option (b) is correct. 2
(18 × 11) + ( 42 × v)
6. 21 = Now, at t = 0, v = v0
18 + 42
∴ C = v0
⇒ v = 25.29 m/s 1
i.e., v = bt2 + v0
8 t3 2
7. x = 32t −
3 ds 1
or = v0 + bt2
dx dt 2
v= = 32 − 8 t2 …(i)
dt  1 2
or ∫ ds = ∫ v0 + 2 bt  dt
∴ Particle is at rest when
1
32 − 8 t2 = 0 or s = v0 t + bt 3 + k
6
i.e., t = 2s
At t = 0, s = 0,
Differentiating Eq. (i) w.r.t. time t
dv ∴ k=0
a= = − 16 t
dt 1
⇒ s = v0 t + bt 3
∴ a at time t = 2 s (when particle is at rest) 6
= − (16) × (2) = − 32 m/s2 Option (a) is correct.
Option (b) is correct.
32 | Mechanics-1
1
11. g (2t2 + 6) m (given) v2 = 2 ^i + 4 ^j
2
3rd drop Option (b) is correct.
time = t
1 gt2 13. v20 = 2 gh and u2 = 2 g (3 h)
2
u2
2nd drop ∴ =3
v20
or u = v0 3
1 g (2t)2
2 Option (a) is correct.
time = t
14. vx = 8 t − 2
dx
∴ = 8t − 2
dt
1st drop Ground
or ∫ dx = ∫ (82 t − 2) dt
1 2 5 or x = 4 t − 2t + k1
∴ gt = m
2 4 Now at t = 2, x = 14
∴ Height of 2nd drop from ground ∴ 14 = 4 ⋅ 22 − (2) 2 + k1
5
= 5m − m i.e., k1 = 2
4
Thus, x = 4 t2 − 2t + 2 …(i)
= 3.75 m
Further, vy = 2
Option (c) is correct.
dy
12. i.e., =2
[at time = 1] 0 m/s 20 m/s dt
or ∫ dy = ∫ 2 dt
or y = 2t + k2
S
Now, at t = 2, y = 4
∴ k2 = 0
A B
Thus, y = 2t …(ii)
y
Substituting t = in Eq. (i),
[At time = t − 1] 2
1 y
2
y
s = × 10t2 x = 4   − 2   + 2
2  2  2
1
= 20 ( t − 1) + × 10 ( t − 1)2 or x = y2 − y + 2
2
Option (a) is correct.
or t2 = 4 ( t − 1) + ( t2 − 2t + 1)
15. x = 5t and y = 2t2 + t
or 2t − 3 = 0
3 dx dy
or t= s ∴ = 5 and = 4t + 1
2 dt dt
2 dy dy / dt
1 3 Now, =
∴ s = × 10 ×   dx dx / dt
2  2
4t + 1
= 11.25 m tan 45° =
5
Option (c) is correct.
⇒ t =1s
t3 ^
∴ v = t ^i + j Option (b) is correct.
2
16. y = 8 t − 5t2 and x = 6t
Thus, velocity of particle at time t (= 2 s)
dy dx
will be ∴ = 8 − 10t and =6
dt dt
Motion in One Dimension | 33
At t = 0 20. v
dy dx vmax
= 8 and =6
dt dt
∴ Velocity of projection
2 2
dx dy
=   +   = 62 + 82 t1 t2 t
 dt   dt 
= 10 ms −1 In time interval t1
vmax
Option (c) is correct. Acceleration = =a
t1
2H
17. T= In time interval t2
g
vmax
T 2 Retardation = = 2a
∴ =  H t2
 
3 g 9 vmax v
∴ = 2 ⋅ max
Top of tower t2 t1
H/9
i.e., t1 = 2t2
T t =T
3 Now t1 + t2 = t
O
H W ∴ 2t2 + t2 = t
H – H = 8H t
E 9 9 or t2 =
R
3
Ground Depth of shaft = Displacement of lift
t=T 1
= vmax ( t1 + t2 )
Option (c) is correct. 2
1
18. Distance of farthest corner from one = vmax (2t2 + t2 )
corner 2
3
= a + a + a = 3a = vmax t2
3a 2
∴ Time taken = 3
u = (2at2 ) t2
2
Option (a) is correct.
= 3 at22
19. Time-velocity graph of the given t
2
time-acceleration graph will be = 3 a  
3
at2
=
3
v 8 Option (b) is correct.
(m/s)
1 1
21. s = 2ut + at2 = ut + 2at2
2 2
a
4 8 12 t (s) 2u
P P

∴ Height of lift above the starting point time = 0 time = t


when it comes to rest u Q
Q
2a
= Area under t-v graph
4 + 12 s
= × 8 = 64 m
2 1 2
⇒ ut − at = 0
Option (b) is correct. 2
34 | Mechanics-1
1
or u− at = 0 v (m/s)
2
2u 10
or t=
a A1
2
 2u 1  2u O 1 A2 2 t
∴ s = 2u   + a  
 a 2  a
– 10
4u2 2u2
= +
a a
6u2 ∴ Displacement of particle at time (t = 2)
=
a
= ( + 5 m ) + ( −5 m )
Option (a) is correct.
= 0 m.
22. u = vertical speed (w.r.t. cart) of the i.e., the particle crosses its initial position
particle. at t = 2 s.
Actual path followed
u by the particle
Option (b) is correct.
24. As downward direction is considered to be
+ ive, velocity of ball at t = 0 will be − v0 .
t=0 t=T
30 m/s u = Vertical speed Thus, option (a) and (c) are incorrect.
80 m (w.r.t. cart of
the particle)
Now, as the ball will have +ive
acceleration throughout its motion option
80 m 8 (b) is also incorrect.
For cart T = = s
30 m / s 3 ∴ Correct option is (d).
1
For particle 0 = uT + ( − g) T2 25. Let acceleration of lift = a (upwards)
2
Displacement of ball in time t
1 2
i.e., gT = uT = Displacement of lift in time t
2
1 1
T=
2u 8
= v0 t − gt2 = at2
g 3 2 2
gt at
2u 8 or v0 − =
i.e., = 2 2
10 3 2 v0 − gt
40 ∴ a=
or u= ms −1 t
3
Option (a) is correct.
Option (c) is correct.
1 26. a = − 0.2 v2
23. A1 = ( +10) ( +1) = + 5 m dv
2 ∴ = − 0.2 v2
1 dt
A2 = ( −10)( +1) = − 5 m −2
2 or ∫ v dv = − 0.2 ∫ dt
v −2 + 1
f (Air resistance) or = − 0.2t + C
−2+1
1
or − = − 0.2t + C
u=0 v
Now, at t = 0, v = 10 m/s (given)
1
∴ − =0+C
S 10
1
i.e., C=−
10
Motion in One Dimension | 35
1 1 Substracting Eq. (ii) from Eq. (iii),
Thus, − = − 0.2t −
v 10 1
h − H = u ( t − T) + g ( T2 − t2 )
Q For velocity v at time t (= 2 s) 2
1 1 1
− = − (0.2 × 2) − = gT ( t − T) − g ( t − T) ( t + T)
v 10 2
1 4 1 1
− =− − = ( t − T) g T − ( t + T)

v 10 10  2 
1 1  2T − t − T 
= = ( t − T) g  
v 2  2
1
⇒ v = + 2 m/s. = − g ( t − T)2
2
Option (a) is correct. 1
i.e., h = H − g ( t − T)2
27. For displacement ( S1) of train 1 before 2
coming to rest Option (d) is correct.
02 = (10)2 + 2 ( −2) S1 t2
30. x=
i.e., S1 = 25 m 2
For displacement ( S2 ) of train 2 before dx
∴ vx = =t
coming to rest dt
02 = (20)2 + 2 ( −1) S2 x2 ( t2 / 2)2 t4
y= = =
i.e., S2 = 200 m 2 2 8
∴ Smin = S1 + S2 dy t 3
∴ vy = =
= 225 m dt 2
Option (b) is correct. 31. x = t+3
28. Let the balls collide after time t the first
i.e., x = t2 + 6t + 3
dx
ball is shot. = 2t + 6
dt
∴ displacement (S) of ball 1 at time t
i.e., v∝t
= displacement ( S) of ball 2 at time ( t − 2)
1 1 Option (b) is correct.
40t + ( − g) t2 = 40 ( t − 2) + ( − g) ( t − 2)2
2 2 32. Aeroplane’s velocity at time t (= 20 s)
1 2 2 = Area under curve ( t-a graph)
or g[ t − ( t − 2) ] = 80
2 1
= (20 × 5) −  × 10 × 2
or ( t + t − 2) ( t − t + 2) = 16 2 
or 2t − 2 = 8 = 90 m/s
or t = 5s Option (c) is correct.
1
S = 40 × 5 + ( −10) 52 33. v = 5 1 + 5 …(i)
2
At s = 0 (i.e., initially),
= 200 − 125
velocity of particle = 5 m/s
= 75 m
Option (b) is correct.
Option (b) is correct.
Differentiating equation (i) w.r.t. s
29. 0 = u − gT 1
dv 1 −
i.e., u = gT …(i) = 5 (1 + 5) 2
ds 2
1
H = uT − gT2 …(ii) dv 5 −
1 1
2 v⋅ = (1 + 5) 2 ⋅ 5 (1 + 5) 2
1 2 ds 2
and h = ut − gt …(iii)
2 = 12.5 m/s 2
36 | Mechanics-1
Option (b) is correct. = 12.5 − 16.0 + 10
34. See answer to question no. 2. = 6.5 m/s
Time of ascent mg − f Option (b) is correct.
=
Time of descent mg + f 37. u2 = 2 gH
mg − ma u2
= ⇒ H=
mg + ma 2g
H
g−a 102 = u2 − 2 g
= 2
g+a 2
⇒ u = 100 + gH
10 − 2
=
10 + 2 Stops

2
=
3
Option (b) is correct. H 10 m/s

35. H
N 2
θ W E u
10 t River flow
S
100 + gH
5t Thus, H=
2g
5t 1
sin θ = = or gH = 100
10 t 2
⇒ H = 10 m
⇒ θ = 30° Option (b) is correct.
Option (b) is correct.
36. Fnet = 3 t2 − 32
i.e., manet = 3 t2 − 32 38. 10 m/s
3 t2 − 32
or anet =
10
2
or anet = 0.3 t − 3.2
dv 15 m
or = 0.3 t2 − 3.2
dt
2
or ∫ dv = ∫ (30.3 t − 3.2) dt u
0.3 t t=0
or v= − (3.2) t + k
3 10 = u2 − 2 g × 15
2

or v = 0.1t 3 − (3.2) t + k 100 = v2 − 2 × 10 × 15


Now, at t = 0, v = 10 m/s i.e., u = 20 m/s
∴ 10 = k Now, v = u − gt = 20 − 10 × 3
Thus, v = 0.1 t 3 − (3.2) t + 10 = 20 − 10 ⋅ (3)
∴ at t = 5 s = − 10 m/s, downward
v = 0.1 (5) 3 − (3.2) (5) + 10 Option (d) is correct.
Motion in One Dimension | 37
JEE Corner
Assertion and Reason
→ Displacement
1. As acceleration (a ) is just opposite to =
→ t0
velocity ( v), the particle will move along a
1
straight line. v0 t0
v
=2 = 0
t0 2
a
∴ Assertion is true.
Reason is also true as proved.
4. Acceleration (a) will be zero only when
velocity does not change with time
v dv
a =v⋅
ds
∴ Assertion is false (motion being one ds dv
= ⋅
dimensional). dt ds
→ → dv
Reason is true as v and a do not depend =
upon time. dt
dv
Option (d) is correct. ∴ a = 0 if =0
dt
2. Displacement-time graph is parabolic only
or v is constant with time.
when the slope of the straight line
velocity-time graph is not zero i.e., Thus, Assertion is wrong.
acceleration is not zero. dv
Reason is correct as a is equal to which
If acceleration is zero. dt
is instantaneous acceleration.
s = ut
5. If acceleration is in the opposite direction
i.e., the displacement-time graph will be a
to the velocity of the particle the speed
straight line.
will decrease.
∴ Assertion is wrong.
u = 10 m/s
Reason : v = u + at
a = − 2t m/s 2
ds
i.e., = u + at dv
dt = − 2t
dt
or ∫ ds = ∫ (u1 + at) dt
s = ut + at2 + k
∫ dv = ∫ − 22 t dt
t
2 v = − 2⋅ + k
2
If at t = 0, s = 0
At t = 0 s, velocity = + 10 m/s
The value of k will be zero. ∴ k = + 10 m/s
1
∴ s = ut + at2 ∴ v = − t2 + 10
2
Time (t) v m/s speed (m/s)
Thus, reason is true.
15 9 9
3. Displacement in time t0
25 6 6
= Area under v-t graph 35 1 1
1
= v0 t0 ∴ Assertion is true.
2
∴ average velocity in time interval t0
38 | Mechanics-1
Reason is false as when acceleration is Time t Relative Displacement
positive the speed will increase. 0s 0m
da 1s 4m
6. = 2 (ms −3) (given)
dt 2s 24 m
[This implies that reason is true] 3s 60 m

∫ da = ∫ 2 dt As relative displacement is increasing the


a = 2t + C relative velocity would also be increasing.
If at t = 0, a = 0 ∴ Assertion is false.
we have C=0 Reason is true.
∴ a = 2t 10. If v = u + ( − a) t [acceleration being made
dv − ive]
or = 2t
dt Velocity (v) of the particle will be zero
∴ Assertion is false. u
at t= .
Option (d) is correct. a
u
7. If initially particle velocity is − ive and Thus, for t < , v is + ive
a
acceleration is uniform and + ive, the
particle will return to its initial position i.e., the acceleration can change its
direction without change in direction of
after a certain time interval. In this time
velocity.
interval the average velocity will be zero
as net displacement will be zero. ∴ Assertion is true.
∴ Assertion is false. If ∆V changes sign say from + ive to − ive,
For average velocity to zero, the particle ∆V
the acceleration which equals will also
must return to its initial position (as ∆t
discussed above) and for this velocity can’t change sign from + ive to − ive.
remain constant.
∴ Reason is true but it is not the correct
∴ Reason is true. explanation of the assertion.
Option (d) is correct. 11. At time t when the two are at the same
8. From O to A velocity of the particle is height.
increasing while from A to B it is A u=0
decreasing without change in direction as
S1
for the velocity to change its direction the a=+g
B
slope of s-t graph must be negative. A time t
∴ Assertion is false.
S2
If the slope of s-t graph is + ive the velocity
of the particle will be + ive while if it is u = √gh
− ive the velocity of the particle will also B
be − ive.
∴ Reason is true. S1 + S2 = h
2  1 gt2  + ut − 1 gt2  = h
9. S1 = 2t − 4 t
2   2 
and S2 = − 2t + 4 t2
ut = h
⇒ displacement of particle 2 w.r.t. 1.
gh ⋅ t = h
S21 = S2 − S1
h
= ( − 2t + 4 t2 ) − (2t − 4 t2 ) t=
g
= − 4 t + 8 t2
4 Projectile Motion

Introductory Exercise 4.1


1
1. A particle projected at any angle with = 80 − × 10 × 22 = 60 m
horizontal will always move in a plane 2
and thus projectile motion is a ∴ v = v2x + v2y
2-dimensional motion. The statement is
thus false. = 402 + 202
2. At high speed the projectile may go to a = 20 5 m/s
place where acceleration due to gravity v y 20
tan φ = =
has some different value and as such the vx 40
motion may not be uniform accelerated. 1
The statement is thus true. i.e., φ = tan −1  
 2
3. See article 4.1.
s = s2x + s2y
4. u = 40 2 m/s, θ = 45°
= 802 + 602
As horizontal acceleration would be zero.
vy v = 100 m
φ
sy
uy vx
tan α =
u B sx
α sy g 60
=
A ux 80
sx
3
i.e., α = tan −1  
vx = ux = u cos θ = 40 m/s  4
sx = ux t = (u cos θ) t = 80 m 1
5. s y = u y t + ( − g) t2
A : position of particle at time = 0. 2
1 2
B : position of particle at time = t. or s y = (u sin θ) t − gt
2
As vertical acceleration would be − g 1 2
or 15 = 20t − gt
v y = u y − gt 2
= u sin θ = gt or t2 = 4 t + 3 = 0
= 40 − 20 i.e., t = 1 s and 3 s
= 20 m/s 6. See figure to the answer to question no. 4.
1
s y = u y ⋅ t − gt2 u = 40 m/s, θ = 60°
2
∴ ux = 40 cos 60° = 20 m/s
52 | Mechanics-1
Thus, vx = ux = 20 m/s → ^ ^
10. r = [3 t i + ( 4 t − 5t2 ) j] m
As φ = 45°
vy y-coordinate will be zero when
tan φ = =1 4 t − 5t2 = 0
vx
4
i.e., t = 0 s, s
∴ y y = vx = 20 m/s 5
Thus, v = v2x + v2y = vx 2 t = 0 belongs to the initial point of
projection of the particle.
= 20 2 m/s →
∴ r = 0 ^i m
Before reaching highest point
i.e., x = 0m
v y = u y + ( − g) t
At t = 0.8 s,
∴ 20 = 40 sin 60° − 10 t →
or 2=2 3 − t r = 2.4 ^i m
⇒ t = 2 ( 3 − 1) s i.e., x = 2.4 m
After attaining highest point 11. vx = ux = 10 m/s
− 20 = 40 3 − 10t v
u = 20 m/s φ
i.e., −2=2 3 − t vx
or t = 2 ( 3 + 1) s
Range 60°
7. Average velocity =
Time of flight ux = 20 cos 60°
u2 sin 2α / g
= = 10 m/s
2 u sin α / g
vx = ux = 10 m/s
2 u2 sin α cos α g
= × vx = v cos φ
g 2 u sin α v
⇒ cos φ = x
= ucos α v
8. Change in velocity 10
=
v
+ u sin α u
t=T 10 20
= [as, v = (given)]
10 2
α u sin α
u cos α α =1
i.e., φ = 0°
– u sin α u ∴ Speed will be half of its initial value at
= ( − u sin α) − ( + u sin α) the highest point where φ = 0°.
= − 2usin α u sin θ
Thus, t=
= 2usin α (downward) g
9. Formulae for R, T and H max will be same 20 sin 60°
= = 3s
if the projection point and the point where 10
the particle lands are same and lie on a
horizontal line.
Projectile Motion 53
Introductory Exercise 4.2
1. Time of flight 10 3
= ×
vx 3 2
vy = 5 m/s
vnet
u
3. Let the particle collide at time t.
v 2
α
β u
α = 60°
β = 30° 1 θ
u = 10 m/s2
h
g = 10 m/s2
10 m
2u sin (α − β)
T=
g cos β
x1 x2
(2 × 10 sin(60° − 30° ) d
=
10 cos 30°
2 x1 = (u cos θ) t
= s and x2 = v t
3
∴ d = x2 − x1
Using, v = u + at
= (v + u cos θ) t
vx = ux = u cos 60°
1 = [10 + 10 2 cos 45° ] t = 20 t
= 10 × = 5 m/s 1
2 Using equation, s = ut + at2
2
v y = u y + ( − g) T
For vertical motion of particle 1 :
= u sin 60° − gT 1
h − 10 = (u sin θ) t + ( − g) t2
3 2 2
= 10 − 10 1
2 3 i.e., h = 10 + (u sin θ) t − gt2 …(i)
3 − 4 2
= 10   1
2 3  or h − 10 + 10 t − gt2
2
5
=− m/s For the vertical motion of particle 2 :
3 1
20 − h = gt2
vnet = v2x + v2y 2
1
−5
2 i.e., h = 20 − gt2 …(ii)
= 52 +   2
 3
Comparing Eqs. (i) and (ii),
1 1 1
=5 1+ 10 + 10 t − g t2 = 20 − g t2
3 2 2
10 ⇒ t = 1s
= m/s
3 ∴ d = 20 m
2. Component of velocity perpendicular to 4. u = 10 m/s
plane v = 5 2 m/s
= vnet cos β v
u
10
= × cos 30° θ φ
3 A B
d
54 | Mechanics-1
θ = 30° u = 40 m/s
φ = 45° α = 60°
d = 15 m β = 30°
Let the particles meet (or are in the same g = 10 m/s 2
vertical time t). ∴ φ=β
∴ d = (u cos θ) t + (v cos φ) t vx = ux = u cos α
⇒ 15 = (10 cos 30° + 5 2 cos 45° ) t vx = v cos φ = v cos β
or 15 = (5 3 + 5) t or u cos α = v cos β
3 u cos α 40 cos 60°
or t= s ⇒ v= =
3 +1 cos β cos 30°
= 1.009 s 1
40  
Now, let us find time of flight of A and B  2
=
2 u sin θ 3
TA =
g 2
40
= 1s = m/s
3
As TA < t, particle A will touch ground
before the expected time t of collision. 7. (a) At time t, vertical displacement of A
∴ Ans : NO. = Vertical displacement of B
5. For range to be maximum
/s time t
m
20
u =
vA vB = 10 m/s
a = – 10 m/s2
α β = 30° = π
6
A
x B
π β
α= + 1 2 1
4 2 (v A sin θ) t − gt = vB t − gt2
π π/6 2 2
= +
4 2 i.e., v A sin θ = vB
π vB
= sin θ =
3 vA
= 60° 10 1

= =
6. At point A velocity ( v) of the particle will 20 2
be parallel to the inclined plane. ∴ θ = 30°
A
(b) x = (v A cos θ) t
v 1
A φ = (20 cos 30° ) ×
2
u
=5 3 m
α
β
Projectile Motion 55
AIEEE Corner
Subjective Questions (Level 1)
u2 sin 2θ (20 2)2 sin 90° or T2 − 4 T − 8 = 0
1. (a) R = =
g 10 − ( −4) + ( −4)2 − 4(1)( −8)
∴ T=
= 80 m 2 (1)
2 2 2 2
u sin θ (20 2) sin 45° Leaving − ive sign which is not positive.
H= =
2g 10 4 + 48 4 + 4 3
= =
= 40 m 2 2
2 u sin θ 2 (20 2) sin 45° = (2 + 2 3 ) s
T= =
g 10 = 5.46 s
=4s R = 20 × 5.46 = 109.2 m
1
→ →
(b) u = (20 i + 20 j) m/s and a = − 10 ^j m/s2
^ ^ (b) s = ut + at2
2
→ → →
−v=u+at + 20 m/s

= (20 ^i + 20 ^j) + ( − 10 ^j) 1(at t = 1 s)


– 40 m
= (20 ^i − 10 ^j) m/s a = – 10 m/s2

(c) Time of flight T


2 u sin θ R
T=
g 1
( −40) = 0 × T + ( −10) T2
1 2
2 × 20 2 ×  
 2 ⇒ T =2 2s
=
10 = 2.83 s
=4s R = 2.83 × 20
(c) ∴ Velocity of particle at the time of = 56.6 m
collision with ground. 1 2
(c) s = ut + at
= (20 ^i + 20 ^j) + ( − 10 ^j ) 4 2

= (20 ^i − 20 ^j ) m/s
1 2
2. (a) s = ut + at 20 √2 m/s
2 – 40 m
20 m/s
20√2 m/s T
R
+ 45° 1
( −40) = ( −20) T + ( −10) T2
2
– 40 m or 5T2 + 20T − 40 = 0
a = – 10 m/s2
or T2 + 4 T − 8 = 0
T
− 4 + ( 4)2 − 4 (1) ( −8)
R ∴ T=
2 (1)
1
∴ ( −40) = ( + 20) T + ( −10) T2 −4+ 4 3
2 = = − 2 + 2 3 = 1.46 s
2
or 5T2 − 20T − 40 = 0
56 | Mechanics-1
∴ R = 20 × 1.46 100
∴ v=
1
= 29.2 m 40 ×
2
3. (a) Change in velocity (vc ) 5
= m/s
= Change in vertical velocity 2
(as horizontal velocity does not change).
5. Horizontal compnent of velocity at
= (u sin θ − gt) − (u sin θ) P = Horizontal component of velocity at O
= − gt v sin θ
= − (10 × 3) m/s v
P
= − 30 m/s θ Q
60 m/s θ
t = t1 v cos θ
= 30 m/s (downward) a = –10m/s2 – v sin θ
v
→ + 60°
(b) u = (20 ^i + 20 ^j) m/s O 60 cos 60°
→ –
a = − 10 ^j m/s 2
∴ v cos θ = 60 cos 60°
∴ Displacement at time t (= 3 s)
⇒ v cos 45° = 60 cos 60°
→ 1→
= u t + a t2 60 cos 60°
2 ⇒ v=
cos 45°
1
= (20 i + 20 ^j) 3 + ( −10 ^j) 32
^
= 30 2 m/s
2
For point P :
= 60 ^i + 60 ^j − 45 ^j
v sin 45° = 60 sin 60° + ( −10) t1
= 60 ^i + 15 ^j or 30 = 60
3
− 10 t1
2
→ 60 ^i + 15 ^j
∴ v av = t1 = 3 ( 3 − 1)
3
= 2.20 s
= 20 ^i + 5 ^j For point Q :

⇒ |v av |= (20)2 + (5)2 − v sin 45° = 60 sin 60° + ( −10) t2
∴ t2 = 3 ( 3 + 1)
= 425
= 8.20 s
= 20.6 m/s
6. Y = bx − cx2
4. R + vT = 50
u Differentiating above equation w.r.t. time t
dy dx dx
∴ =b − 2cx …(i)
t=0 Player dt dt dt
R v dy dx
t=0 ⇒ =b (at x = 0)
Coach t=T dt dt
50 m 2 2
 dx   dy 
2
u sin θ2 v=   + 
⇒ vT = 50 −  dt   dt 
g 2 2
 dx   dx 
2 u sin θ (20)2 =   + b2  
v× = 50 −  dt   dt 
g 10
dx
 2 × 20 × sin 45°  = 1 + b2 …(ii)
v×  = 10 dt
 10 
Projectile Motion 57
2
Differentiating Eq. (i) w.r.t. time t (6.1) = u2y
+ 2 ( −10) (9.1)
2
d2 y d2 x  dx  d2 x ⇒ u y = 37.21 + 182
2
= b ⋅ 2
− 2c   − 2cx 2 …(iii)
dt dt  dt  dt = 14.8
Acceleration of particle Angle of projection
2  uy 
 d2 x   d2 x  θ = tan −1  
− a =  2 +  2  ux 
 dt   dt 
 uy 
d2 x = tan −1  
∴ =0  ux 
dt2
14.8 
d2 x d2 y = tan −1  
Substituting = 0 and = − a in  7.6 
dt2 dt2
Eq. (iii) = 62.82°
 dx 
2
u2 sin 2θ
− a = − 2c   Range =
 dt  g
dx a (u2x + u2y ) sin 2θ
i.e., = =
dt 2c g
dx (v2x + u2y ) sin 2θ
Substituting above value of in Eq. (ii) =
dt g
a
v= (1 + b2 ) [(7.6)2 + (14.8)2 ] sin 125.64 °
2c =
10
^ ^
7. (a) Velocity at point A = 7.6 i + 6.1 j = 22.5 m
vy → (c) Magnitude of velocity just before the
v
B ball hits ground
+h 1
A
uy = [( −u y )2 + (ux )2 ]2
u
9.1 m a = – 10 m/s2 1
θ = [( −14.8)2 + (6.1)2 ]2 [ux = vx ]
O φ −1
= 16 ms
−1  u y 
∴ Initial vertical velocity at a = 6.1 ^j m/s. (d) φ = θ = tan  
 ux 
Final vertical velocity at B (highest point) 14.8 
= tan −1  
= 0 m/s  7.6 
2 2
Using v = u + 2as (Between A and B) = tan −1 (1.95)
02 = (6.1)2 + 2 ( −10) ( + h) 8. As initial vertical velocities of both
(6.1)2 particles will be zero and both fall under
⇒ h=
20 same acceleration (g), at anytime t, the
= 1.86 m vertical displacement of both will be same
∴ Maximum height attained by ball
= 9.1 + 1. 86
u1 12 u2
= 10.96 m
(b) Let magnitude of vertical velocity at O +h a=+g
(point of projection) = u y A B
θ time = t time = t (90°− θ)
Using v2 = u2 + 2as (Between O and A)
58 | Mechanics-1
i.e., both will always remain in the same Y y
or + = tan α
horizontal line as shown in figure. x R−x
At time t : or tan θ + tan φ = tan α
Vertical velocity of A (Students to remember this formula)
= Vertical velocity of B 3 3
⇒ tan α = +
= 0 + ( + g) t 6 12
9
= gt =
12
At A : 3
Vertical velocity of particle 1 ⇒ α = tan −1  
tan θ =  4
Horizontal velocity of particle 1
10. On the trajectory there be two points P
gt
or tan θ = …(i) and Q at height h from ground.
u1 P Q
At B : u
h a=–g
Vertical velocity of particle 2 α
tan (90°− θ) =
Horizontal velocity of particle 2 O

If particle takes t time to reach point A


gt (i.e., vertical displacement of + h)
or cos θ = …(ii)
u2 1
( + h) = (u sin α) t + ( − g) t2
Multiplying Eq. (i) by Eq. (ii), 2
gt gt 2
1= × or gt − 2 (u sin α) t + 2h = 0 …(i)
u1 u2 The above equation is quadratic in t. Two
u1u2 values of t will satisfy Eq. (i). One having
∴ t= lower value will be time ( = t1) to reach
g
point P while the higher value will be the
Distance between A and B time ( = t2 ) to reach point Q.
= R1 + R2 ∴ Time to reach point Q from point P
= u1t + u2 t = t2 − t1
= (u1 + u2 ) t
uu = ( t2 + t1)2 − 4 t1t2
= (u1 + u2 ) 1 2 2
g  2u sin α   2h
=   − 4 
7 2  g   g
=
9.8
[Using Eq. (i)]
= 2.47 m 4u2 sin2 α − 2 gh
x =
9. Y = x 1 −  = tan α g
 R
(y, x) 16 gh sin2 α − 8 gh
or ( t2 − t1) =
u g
Y = 3m (Qu = 2 gh)
α θ φ
Distance between P and Q :
x (R – x)
=6m 2h = (u cos α) ( t2 − t1)
YR ∴ 4 h2 = u2 cos2 α ( t2 − t1)2
= tan α
x( R − x)  16 gh sin2 α − 8 gh
or 4 h2 = ( 4 gh) cos2 α  
 g2 
Projectile Motion 59
2 2
or 1 = cos (16 sin α − 8) u
or 1 = cos2 α [16 (1 − cos2 α) − 8 ]
β
or 1 = cos2 α [8 − 16 cos2 α ] R b
or 16 cos4 α − 8 cos2 α + 1 = 0 2
u sin 2β
⇒ ( 4 cos2 α − 1)2 = 0 R+ b=
g
1
⇒ 4 cos2 α = au2 sin 2β
4 i.e., aR + ab = …(ii)
1 g
cos α =
2 Adding Eqs. (i) and (ii), we have
cos α = 60°  b sin 2α + a sin 2β 
bR + aR = u2  
1  g 
(cos α = − being not possible).
2 ( b sin 2 α + a sin 2β )
or R = u2
16 gh sin2 60° − 2 gh ( a + b) g
∴ ( t2 − t1) =
g2 u2 sin 2θ ( b sin 2α + a sin 2β)
= u2
h g ( a + b) g
=2 (Proved)
g 1  b sin 2α + a sin 2β 
⇒ θ= sin −1  
11. Initial vertical velocity = usin α 2  a+b 
v
v (Proved.)
β
t 2h
13. R1 = v1
a=0 g
α
a=–g v1

At time t vertical velocity = v sin β h


∴ v sin β = u sin α = ( − g) t …(i)
Now, as horizontal acceleration will be R1 = 10 m
zero.
v2
v cos β = u cos α
Thus, Eq. (i) becomes
 u cos α 
  sin β = u sin α − gt R2
 cos β 
2
or u sin α cos β − u cos α sin β = gt cos β 1  R1 g 
i.e., h=  
gt cos β 2  v1 
or u=
sin (α − β) 2
1  10 × 9.8 
2 =  
u sin 2α 2 5 
12. R − a =
g = 19.6 m
2h
∴ T=
u
g
2 × 19.6
α =
a 9.8
R
=2s
bu2 sin 2α
i.e., bR − ab = …(i) R2 = v2 ⋅ T = 7.5 × 2
g
= 15 m
60 | Mechanics-1
14. Vertical velocity of balloon (+ bag) = 1.69 s
2u2 sin (α − β) cos α
R=
O α g cos2 β
12 km/h A 2 × (20 2)2 × sin 15° × cos 45°
=
vw = 20 km/h 50 m 10 cos2 30°
α = 39 m
2 u sin (α + β)
5 16. T=
= 12 × m/s g cos β
18
10 2 × 20 2 × sin ( 45° + 30° )
= m/s =
3 10 cos 30°
Horizontal velocity of balloon (+ bag) = 6.31 s
5
= Wind velocity = 20 km / h = 20 × m/s u2
18 R= [sin (2α + β) + sin β ]
50 g cos2 β
= m/s (20 2)2
9 = [sin (90° + 30° ) + sin 30° ]
= 5.55 m/s 10 cos2 30°
12 = 145.71 m
∴ tan α = . i.e., sin α = 0.51
20 2 u sin (α + β)
17. T=
Bag is released at point A. g cos β
Let t be time, the bag takes from A to 2u sin β
= (Qα = 0°)
reach ground. g cos β
1 2u
Using, s = ut + at2 = tan β
2 g
10 1
( −50) =  sin α t + ( − g) t2

=
2 × 20
tan 30°
3  2 10
i.e., 5t2 − 1.7 t − 50 = 0 = 2.31 s
1.7 + ( −1.7)2 − 4 × 5 × ( −50) u2
∴ t= R= [sin (2α + β) + sin β ]
2×5 g cos2 β
= 3.37 s u2 ( 2 sin β)
= [as α = 0°]
Vertical velocity of bag when it strikes g cos2 β
ground uT 20 × 2.31
10 = =
vB = − + (10) (3.37) cos β cos 30°
3
= 53.33 m
= 37.03 m/s u2
vω = 5.55 m/s 18. R = [sin (2α + β) + sin β ]
g cos2 β
∴ Velocity of bag with which it strikes u2
ground = [sin {2 (α + β) − β } + sin β ]
g cos2 β
vnet = v2B + v2ω
u2
= 37.44 m/s = [sin ( π − θ) + sin θ ]
g cos2 θ
2 u sin (α − β) π
15. T = [Q(α + β) = ]
g cos β 2
=
2 × 20 2 × sin ( 45° − 30° ) 2 u2
or R= tan θ sec θ
10 cos 30° g
Projectile Motion 61
19. (a) Acceleration of particle 1 w.r.t. that of Vertical acceleration of stone = g
particle 2 Horizontal velocity of stone = v
= ( − g) − ( − g) ∴ Path of the stone will be parabolic.
=0 21. (a) g eff = g − ( − a)
^
(b) Initial velocity of 1st particle = 20 j m/s
w m/s a m/s2
Initial velocity of 2nd particle
= (20 2 cos 45° ^i + 20 2 sin 45° ^j) m/s u

= (20 ^i + 20 ^j) m/s θ

∴ Initial velocity of 1st particle w.r.t. that


of 2nd particle
=g+a
= [ (20 ^j) − (20 ^i + 20 ^j ) ] m/s
= 10 + 1
= − 20 ^i m/s
= 11 m/s 2
= 20 m/s (downward) 2 u sin θ
T=
(c) Horizontal velocity of 1st particle g eff
= 0 m/s
2 × 2 × sin 30°
Horizontal velocity of 2nd particle =
11
= 20 ^i m/s
= 0.18 s
∴ Horizontal velocity of 1st particle w.r.t.
that of 2nd particle (b) Dotted path [(in lift) acceleration
upwards]
= 0 − (20 ^i)
u
= − 20 ^i m/s
θ
Relative displacement of 1st particle w.r.t.
2nd particle at t = 2 s Full line path [In lift at rest or moving
= − 20 ^i × 2 with constant velocity upwards or
downwards].
= − 40 ^i m/s (c) If lift is moving downward with
acceleration g.
∴ Distance between the particles at t = 2 s
= 40 m Path of particle
u
20. (a) As observed by passenger
Vertical acceleration of stone θ
= g −0= g
Horizontal velocity of stone g eff = g − g = 0
=v−v=0 22. Horizontal motion :

∴ Path of the stone will be a straight line x1 = u1 cos θ1 and x2 = u2 cos θ


(downwards). ∴ u1 cos θ1 + u2 cos θ2 = 20 …(i)
(b) As observed by man standing on
ground
62 | Mechanics-1
1
Vertical motion : 20 + (u1 sin θ1) t + ( − g) t2 = 30
u2 2
1
+ (u2 sin θ2 ) t + ( − g) t2
u1
t θ2 2
or (u1 sin θ1 − u2 sin θ2 ) t = 10 …(ii)
1 θ1
20

x1 x2

d = 20 m

Objective Questions (Level 1)


→ ^ ^ → ^ ^ 20
1. v = 3 i + 4 j and F = 4 i − 3 j =
2
→ →
v ⋅ F = (3 ^i + 4 ^j) ⋅ ( 4 ^i − 3 ^j) = 14.14 m/s
= 12 − 12 = 0 = 14 m/s (approx)
→ → Option (b) is correct.
∴ F⊥v
u2 sin2 α
Path of the particle is circular. 4. H (maximum height) =
2g
Y
u2 sin2 θ u2 sin2 (90° − θ)
∴ H1 = and H2 =
→ 2g g
v
u2 cos2 θ
=
g
H1 sin2 θ
Thus, =
j H2 cos2 θ
i X Option (c) is correct.
5. Equation to trajectory is

F 1 gx2
Y = x tan θ − 2
2 u cos θ sin θ
Option (c) is correct. x2
Y = x tan θ −
2. Projectile motion is uniformly accelerated R
everywhere even at the highest point. R
Area = ∫ 0 Y dx
Option (a) correct.
Option (b) incorrect. R x2 
= ∫0  x tan θ −  dx
At the highest point acceleration is  R
perpendicular to velocity. R
 x2 x3 
Option (c) incorrect. A =  tan θ −
2 3 R  0
3. For range to be maximum
R2 R3
θ = 45° = tan θ −
ux 1 2 3
i.e., = cos 45° = 2  tan θ 1
u 2 =R −
u  2 3 
or vx = (Qvx = ux )
2
Projectile Motion 63
4v40 2 2
sin θ cos θ  tan θ 1  9. Rmax = 1.6 m
= −
g2  2 3  u2
= 1.6
4v40 sin 3 θ cos θ sin2 θ cos2 θ  g
= −
g2  2 3 
 ⇒ u = 4 m/s
2 v40 2u sin 45°
= [3 sin2 θ cos θ − 2 sin2 θ cos2 θ ] ∴ T=
3 g2 g
4 2
6. v cos φ = u cos θ =
v 10
φ 10 2 10 2
Number of jumps = =
u β T 4 2 / 10
= 25
θ ∴ Grass hopper would go
A
u cos 60° = 25 × 1.6 m i.e., 40 m.
or v=
cos 30° Option (d) is correct.
u |Displacement|
or v= 10. |Av. velocity|=
3 time
1 2
KE at B = mv2 1  R 2
2 =   +H
T / 2  2
1 u2
= m⋅ 2 2
2 3 1  u2 sin θ cos θ   u2 sin2 θ 
=   + 
K Q 1 mu2 = K  u sin θ  g   2g 
=  
3  2  g
∴ Option (b) is correct. sin2 θ
= u cos2 θ +
u2 sin 2θ 1  u2  4
7. =  
g 2 g u
= 1 + 3 cos2 θ
1 2
or sin 2θ =
2 Option (b) is correct.
or 2θ = 30° 11. d2 = R2 + u2 T2
∴ θ = 15° T
Ball
2 /g
Option (a) is correct. u =u
R
2 u sin θ
8. T1 = 45°
g 90° α
2 u sin (90° − θ)
∴ T2 =
g uT
2u cos θ
=
g T
2 u2 sin θ cos θ
2 u  2 2
 2 u sin 45° 
2
Thus, T1T2 = =   + u2 
g g 
 g  g 
2R
or T1T2 = u4 2 u4
g = + 2
1 g2 g
⇒ R = g T1T2
2 3u4
=
Option (d) is correct. g2
64 | Mechanics-1
u2 302 u2 sin 2φ u2 sin 2α
∴ d= 3= 3 = 90 3 m if, =
g 10 g g
Option (b) is correct. sin 2φ = sin 3α
12. At maximum height 2φ = π − 2α
dy π
=0 or φ= −α
dt 2
d π π π
i.e., (10t − t2 ) = 0 = − = = 30°
dt 2 3 6
or 10 − 2t = 0 ∴ Maximum height attained by second
particle
or t = 5s
u2 sin2 φ
∴ Maximum height attained = 10 (5) − 52 =
2g
= 25 m
102 × 2 g sin2 30°
Option (d) is correct. = ×
sin2 60° 2g
u2
13. R = [sin (2α + β) + sin β ] 102 × 1 / 4
g cos2 β = = 34 m
3/4
u
α Option (d) is correct.
1
15. s = ut + at2
a = –g 2
1
R ∴ ( −70) = (50 sin 30° ) t + ( −10) t2
2
or 5t2 − 25t − 70 = 0
β or t2 − 5t − 14 = 0
( t − 7) ( t + 2) = 0
As α = 0 (according to question) t=7s
u2 (− 2 s not possible)
R= [2 sin β ]
g cos2 β Option (c) is correct.
(50)2 × 2 sin 30° 16. Initial separation
=
10 × cos2 30°
1000
= m
3 53° T
Option (b) is correct. x
14. First particle : R 5T

H max = 102 m x = R + 5T
u2 sin2 α = (u cos α) T + 5T
∴ = 102 = T [(u cos α) + 5]
2g
2u sin α
102 × 2 g π = [(u cos α) + 5]
⇒ u2 = [As α = = 60°] g
sin2 60° 3
4
2 × 80 ×
= 5 80 × 3  + 5 = 256 m
Second particle : 10  5 
Range of the second particle will be equal Option (d) is correct.
to that of particle
Projectile Motion 65
JEE Corner
Assertion and Reason
1
1. Assertion is wrong while the explanation and y= a y t2
as given in reason is correct. 2
∴ Option (d) is correct. 2y 1 2y
Thus, x = ux + ax
2. Assertion and Reason both are correct. ay 2 ay
both carry the same meaning.
or x = k1 y + k2 y
Option (a) is correct.
u2 sin2 α or ( x − k2 y)2 = k12 y
3. H = [H = maximum height]
2g
or x2 + k22 y2 − 2k2 y − k12 y = 0
2
g  2 u sin α  g 2
=   = T or x2 + k22 y2 − (2 k2 + k12 ) y = 0
8 g  8
→ →
i.e., H ∝ T2 (Reason) v2 − v1
6.
∴ H will become four times if T is made t2 − t1
two times. → → → →
( u + a t2 ) − ( u + a t1)
Thus, assertion is correct and also reason =
t2 − t1
is the correct explanation of the assertion.

Option (a) is correct. = a (which is a constant quantity in
→ → → projectile motion).
4. Reason is correct as A ⋅ B = 0 when A and
→ ∴ Assertion is correct and the reason for
B are perpendicular to each other. this also correct, as explained.
Thus, option (a) is correct.
7. Let initial velocity of A = u A
→ Initial velocity of B = uB
u
B is projected at an angle α with
45° horizontal.
45°
→ For time (TA ) taken by A to return to the
v
point of projection :

In the case mentioned v is perpendicular ( − u A ) = ( + u A ) + ( − g) TA

to u only at t = T and not at any time t. 2u
⇒ TA = A = 4 (s)
Option (b) is correct. g
→ →
[Note : v will never be perpendicular to u Time taken by B to reach ground
if angle of projection is less than 45°.]
2 uB sin α
1 TB = = 4 (s)
5. x = ux t + ax t2 g
2
y ay a (Q TB = TA given)
∴ uB sin α = 2 g

ux ax Height attained by B
x
u2B sin2 α
H=
2g
66 | Mechanics-1
(2 g)2 Reason is correct as the given relation
=
2g under heading reason leads to
2v
= 2g Range = vH × V
g
= 20 m ∴ Option (a) is correct.
∴ Assertion is correct. 9. Assertion is incorrect as it is the velocity
Reason for this incomplete as then TA will which would decrease by 10 m/s in the
not be equal to TB . downward direction.
Thus, option (c) is correct. Acceleration is nothing but rate of change
u2 of velocity written against reason is
8. H= V
2g correct.
∴ Option (d) is correct.
∴ H1 : H2 = 4 H : H
u12V u22 V 10. Using law of conservation of mechanical
i.e., : = 4 :1 energy
2g 2g
1 1
mv2 + mgh = mu2
or u1V : v2 V = 2 : 1 …(i) 2 2
2 uV uH 2
R= ∴ v = u − 2 gh
g
Thus, assertion is correct.
For R1 = R2
If the particle is projected with vertical
u1V u1H = u2 V u2 H
component of velocity as u y the vertical
u1H u2 V component of velocity of the particle at
=
u2 H u1H height h would be u2y − 2 gh is correct as
u1H 1
or = [Using relation (i)] written against reason but it is not the
u2 H 2 correct explanation of the assertion as
∴ Assertion is correct. nothing is given regarding the change in
the horizontal component of velocity.

Objective Questions (Level 2)


Single Correct Option
1 2
1. s = ut + at
2 Time for attaining maximum height
R u sin θ
=
g
16.8 m u P Q = 1s
4.8 m ∴ Above displacement is for point Q.
θ For θ = 90° : s = 16.8 m
O D
For θ = 30° : OD = (u cos θ) t
1
= (u sin θ) t + ( − gt)2 = (20 cos 30° ) 1 × 2
2
= 20.8
= 24 sin θ − 7 × 2
∴ QR = (20.8) + (16.8 − 4.8)2
2
For θ = 30° : s = 4.8 m
= 24 m
Option (c) is correct.
Projectile Motion 67
d 1 x
2. t = also h = gt2 Slope of trajectory = tan θ − 2
u 2 u sin θ cos θ
d Substituting x = (u cos θ) t
u
dy (u cos θ) t
= tan θ − 2
dx u sin θ cos θ
h t
= tan θ −
u sin θ
time = t Option (a) is correct.
1 d2 5. y = βx2
∴ h= g⋅ 2 dy dx
2 u ∴ = β ⋅ 2x
2 u2 h dt dt
2
⇒ d = d2 y  d2 x  dx  2 
g and = 2β x ⋅ 2 +   
dt2  dt  dt  
Option (b) is correct.
u2  d2 x  dx  2 
3. R = [sin (2α + β) + sin β ] or α = 2β x 2 +   
g cos2 β  dt  dt  
2
Substituting v = 10 m/s d x
Now, as 2 = 0
g = 10 m/s2 dt
α + β = 90° (acceleration being along y-axis only)
and β = 30° dx α
=
(10)2 dt 2β
R= [sin (90° + 60° ) + sin 30° ]
10 cos2 30° Option (d) is correct.
10 1 1 40
= + = m 6. As the projectile hits the inclined plane
 3  2 2 3 horizontally
 
 4
Option (c) is correct. u P
1 x2
4. y = x tan θ − g 2 α
R'
2 u sin θ cos θ
β
O Q
Slope
u2 sin2 α
PQ = H max =
2g
x u2 sin α cos α
and OQ =
g
dy 1 2x ∴ Range on inclined plane
= tan θ −
dx 2 u2 sin θ cos θ R ′ = ( PQ)2 + (OQ)2
Slope 2 2
T = Time of flight
 u2 sin2 α   u2 sin α cos α 
tan θ =   + 
 2g   g 
T/2 T u2 sin4 α
t
= + sin2 α cos2 α
O g 4
4 2
u2 1  3  3   1 2
=   +   
g 4 4   2   2
68 | Mechanics-1
u2 9 3 Acceleration of the particle
= +
g 64 16 d2 x
= 2
u2 21 dt
= dy
8g = 50 m/s2 [as = constant]
dt
Option (d) is correct.
9. tan α = tan θ + tan φ
7. Let the projectile hits the inclined plane at
P at time t
H
v = 10 √3 m/s u = 10 √3 m/s α θ φ
i.e., v = u
v R/2 R/2
u
= tan θ + tan θ = 2 tan θ
β α
tan α 
R ∴ θ = tan −1  
 2 
P (time = t)
Option (c) is correct.
β
Q R 10. At any time t,
(u cos α) t
20√3 m/s
vt 20 m/s
1
PQ = (u sin α) − gt2 60° 30°
2
Further, QR = vt − (u cos α) t 20√3m
PQ Horizontal distance between particles
and tan β =
QR x = 20 3 − (10 3 + 10 3 ) t
i.e., OQ tan β = PQ = 20 3 (1 − t)
1
t [v − cos α ] tan β = u sin α − gt t Vertical distance between particles at
 2  time t
or 10 [1 − cos 60° ] tan 30° y = (30 − 10) t = 20 t
1
= 10 sin 60° − × 10 × t Distance between particles at time t
 2 
D = x2 + y2
5
or = 5 3 − 5t
3 or D2 = [20 3 (1 − t)]2 + [20 t ]2
1 = 400 [3 (1 − t)2 ] + 400 t2
or t= 3 −
3 For D to minimum
2 dD
= s =0
3 dt
8. y2 + 2 y + 2 = x 3 × 2 (1 − t)( −1) + 2 t = 0
dy dy dx
∴ 2y +2 = or 6t − 6 + 2t = 0
dt dt dt 3
dy dx i.e., t= s
or (2 y + 2) = 4
dt dt  2 2
3  3
dx ∴ D2min = 400 3 1 −   + 400  
or = 10 ( y + 1)
dt   4   4

d2 x  dy  = 75 + 75 × 3
or = 10  
dt2  dt  ⇒ Dmin = 10 3 m
= 50 m/s 2 Option (b) is correct.
Projectile Motion 69
11. Let time of flight = T Substituting v = u and α = β = 30° in Eq.
(ii)
u cos 30° + u cos 30°
u T=
g
α
θ Q 2 u cos 30°
β =
P α g
β
u u 3
=
g
α β
Option (b) is correct.
As, horizontal component of the velocity of 2 u2 sin θ cos θ
12. PQ =
the particle will not change g
u cos (90° − α) = v cos (90° − β) 2 u2 sin (90° − α) cos (90° − α)
or2a cos α =
or u sin α = v sin β …(i) g
Using v = u + at for the horizontal u2 sin α
component of velocity or a=
g
( − v cos β) = ( + u cos α) + ( − g) T
or u2 = 2ag (as α = 30°)
⇒ gT = u cos α + v cos β
u cos α + v cos β or u2 = 2 × 4.9 × 9.8
or T= …(ii) or u = 9.8 m/s
g
Option (a) is correct.
Now, as α = β ( = 30° ), u = v from Eq. (i)
More than One Correct Options
1. Two particles projected at angles α and β 2u sin α
t1 (time of flight of first) =
with same speed ( = µ ) will have same g
range if 2u sin β
t2 (time of flight of second) =
α + β = 90° g
Option (a) is correct. t1 sin α sin α
∴ = = = tan α
2 u2 sin α cos α t2 sin β sin (90° − α)
R=
g Option (c) is correct.
h1 sin α
h1 (maximum height attained by first) Also, = = tan α
u2 sin2 α h2 sin β
=
2g Option (d) is correct.
h2 (maximum height attained by second) 2. Horizontal displacement in time t (Time of
u2 sin2 β flight)
= P
2g
u2 sin α sin β
∴ h1h2 =
Pa

2g
th

Wind
o

a = constant
u2 sin α sin (90° − α)
fp
ar

= a=g
tic

2g
e l

u2 sin α cos α R Q
= = R
2g 4
1 2
Option (b) is correct. x= at …(i)
2
70 | Mechanics-1
Vertical displacement in time t Option (c) is correct.
1
y = gt2 …(ii) Option (d) is incorrect.
2 → ^ ^
y g 4. v = 10 i + 10 j
∴ =
x a v
or y = constant x [as a is constant] H
O
Option (a) is correct. a = – 10 m/s2
– 15m t
Substituting y = 49 m in Eqs. (ii)
1
49 = × 9.8 × t2 If θ be the angle of projection
2 v y 10
⇒ t = 10 = 3.16 s tan θ = = =1
vx 10
PQ > PR ( = 40 m )
i.e., θ = 45°
Option (d) is correct.
Option (a) correct.
3. As there will be no change in the 1 2
horizontal component of the velocity of the Using relation, s = ut + at
2
particle 1
( − 15) = 10 t + ( −10) t2
2
2
i.e., t − 2t − 3 = 0
or ( t − 3) ( t + 1) = 0
v θ + φ = 90° w cos φ ∴ t=3s
Q φ
P
θ
w Option (b) is incorrect.
v cos θ
Horizontal range of particle
w cos φ = v cos θ = vx ⋅ t
or w cos (90° − θ) = v cos θ = 10 × 3 = 30 m
or w sin θ = v cos θ Option (c) is incorrect.
or w = v cos θ Maximum height of projectile from ground
Option (b) is correct. = H + 15
Option (a) is incorrect. u2 sin2 θ
= + 15
Vertical velocity of particle at Q 2g
= wsin φ v2y
= + 15
= wsin (90° − θ) 2g
= wcos θ (10)2
= + 15
Using, v = u + at 2 × 10
( − w cos θ) = ( + v sin θ) + ( − g) T = 20 m
[where T = time from P to Q] Option (d) is correct.
⇒ gT = v sin θ + w cos θ 5. Average velocity between any two points
= v sin θ + v cot θ cos θ can’t remain constant as in projectile
cos2 θ motion velocity changes both in
= v sin θ + v ⋅
sin θ magnitude and direction.
= v cosec θ Option (a) is incorrect.
v cosec θ Similarly option (b) is also incorrect.
∴ t=
g
Projectile Motion 71
In projectile motion [where H = Maximum height]
→ 8 ( H − 15)
a = constant ⇒ 2=
→ 10
dv
∴ = constant ∴ H = 20 m
dt
Option (c) is correct. Option (b) is correct.
→ u2 sin2 α
d2 v i.e., = 20
also =0 2g
dt2
Option (d) is correct. u sin α = 40 g
1 = 20 m/s
6. ( + h) = ( + u sin α) t + ( − g) t2
2 Option (c) is correct.
t = t1
A Bt=t
(u cos α) tAB = 40
v 2
15 m h 40
α ⇒ ucos α =
40 m 2
20 m
= 20 m/s
⇒ gt2 − (2 u sin α) t + 2h = 0 …(i) Option (d) is correct.
2 u sin α 2h
t1 + t2 = , t1t2 = Substituting values of g, usin α and h in
g g Eq. (i)
( t2 − t1)2 = ( t2 + t1)2 − 4 t1t2 10t2 − 40t + 30 = 0
4u2 sin2 α 2h i.e., t2 − 4 t + 3 = 0
= = 4⋅
g g
∴ t = 1 or 3
8 ( H − h) t = 1 s for A and t = 3 s for B.
∴ t AB =
g Option (a) is correct.
Match the Columns
1. (a) At t = 2 s (b) Vertical distance between the two
2 (t = 1 s)
at t = 2 s = 30 m
S1 10 m 2 ∴ (b) → (s)
1 (at t = 1s) at t = 2 s
10 m/s (c) Relative horizontal component of
S2
velocity
30 m
= 10 − 0 = 10 m/s
S1 : S2 : S3
1 (at t = 2 s) =1:3:5 ∴ (c) → (p)
(d) Relative vertical component of velocity
(at t = 2 s)
= Velocity of 1 at t = 2 s
S3 50 m
− Vertical velocity of 2 at t = 2 s
= Velocity of 1 at t = 2 s
− Velocity of 1 at t = 1 s
= (2 g) − (1 g) = g
horizontal distance between 1 and 2 = 10 m/s
= horizontal displacement of 2 (d) → (p).
= 10 × 1 = 10 m
∴ (a) → (p)
72 | Mechanics-1
2. Given, H = 20 m ∴ On increasing u⊥ to two times the T
u2 sin2 α (time) of flight will also become two times
∴ = 20
2g i.e., (a) → (q).
2
usin α = 20 u2 sin2 α  u⊥ 
(b) H = = 
∴ (d) → (s) 2g  2g 
2u sin α H max will become 4 times.
Time of flight = =4s
g ∴ (b) → (r)
(a) → (s) 2(u⊥ ) (u||)
(c) R =
g
R
H= (given) R will become two times
2
(c) → (q)
u2 sin2 α 1 2 u2 sin α cos α
∴ = (d) Angle of projection with horizontal
2g 2 g u
u sin α tan α new = ⊥ (new)
⇒ =2 u||
u cos α
2u⊥
∴ (b) → (q) =
u sin α u||
u cos α = = 10 m/s
2 = 2 tan α
∴ (c) → (r) α new = tan −1 (2 tan α) ≠ 2α
u2 sin (2 × 15° ) ∴ (d) → (s)
3. = 10
2g → ^ ^
5. (a) u (at 0) = (u cos θ) i + (u sin θ) j
2
u
∴ = 20 m →
2g v (at A) = (u cos θ) ^i

⇒ Rmax = 20 m |Change in velocity between A & B|


(a) → (q) = usin θ
u2 ∴ (a) → (q)
(b) Maximum height = = 10 m
4g (b) |Average velocity between 0 and A|

∴ (b) → (p) |OA|
=
(c) Range at ( 45° + θ) = Range at ( 45° − θ) T /2
∴ Range at 75° = Range at 15° 2
 R
= 10 m (given) H2 +  
 2
(c) → (p) =
T /2
u2 sin2 30°
(d) Height at 30° =  u2 sin2 θ  2 1  2 u sin θ cos θ  2 
2g
  +   
1  u2   2 g  4 g  

=   =
4 2g u sin θ
= 5m g
u
∴ (d) → (s) = 1 + 3 cos2 θ
2 u sin α 2
4. (a) T =
g ∴ (b) → (s)
2u⊥
=
g
Projectile Motion 73
(c) Velocity at B ∴ Vertical displacement of (2) w.r.t. (1),
→ ^ ^
vB = (u cos θ) i − (u sin θ) j = (90 ^i − 40 ^j) m
→ →
∴|v B − u| = 2 u sin θ = 50 ^j m
Thus, (c) → (s). ∴ (b) → (r)
(d) |Average velocity between 0 and B| (c) Horizontal velocity of (1) at t = 2 s
Range →
= v1H = 30 ^i
Time of flight
(u cos θ) T Horizontal velocity of (2) at t = 2 s
= = ucos θ →
T v2 H = − 20 ^i
∴ (d) → (p). Relative horizontal component of the
6. (a) Horizontal displacement of (1) at t = 2 s velocity of (2) w.r.t. 1
Y → → →
20 j v RH = v2 H − v1H

(2) = ( − 20 ^i) − ( + 30 ^i)


– 20 i
= − 50 ^i m/s
30 j →
∴|v RH|= 50 m/s
X Thus, (c) → (r)
(1) 30 i
(d) Vertical velocity of (1) at t = 2 s
→ →
S1H = 30 ^i m/s × 2 s v 1 V = 30 ^j + ( − 10 ^j) (2)

= + 60 ^i m = 10 ^j
Horizontal displacement of (2) at t = 2 s Vertical velocity of (2) at t = 2 s
→ ^ ^ →
S2 H = 130 i m + ( −20 i m / s) (1 s) v 1 V = 20 ^j + ( − 10 ^j) (1)

= 110 ^i m = 10 ^j
Horizontal displacement of (2) w.r.t. (1) Relative vertical component of the velocity
^ ^ ^ of (2) w.r.t. 1
= (110 i − 60 i) m = 50 i m →
v RV = (10 ^j) − (10 ^j) = 0
∴ (a) → (r)

(b) Vertical displacement of (1) at t = 2 s ∴ |v RV|= 0 m/s
→ 1
S1V = 30 ^j m/s × 2 s + ( − 10 ^j m / s2 ) (2 s)2 Thus (d) → (s)
2 2 uy
7. T = ∴ T ∝ uy
= 60 ^j m − 20 ^j m g
u2y
= 40 ^j m H= ∴ H ∝ u2y
2g
Vertical displacement of (2) at t = 2 s
→ H A = HB = HC ∴ TA = TB = TC
S2 V = 75 ^j m + (20 ^j m / s) (1 s)
Further, R = ux T
1
+ ( −10 ^j m / s2 ) (16)2 T is same. But
2
R A < RB < RC
= 90 ^j m ∴ u XA < u XB < u XC
5 Laws of Motion
Introductory Exercise 5.1
1. N = Normal force on cylinder by plank R = Normal force on sphere A by left wall,
R N = Normal force on sphere A by ground,
N ′ = Normal force on sphere A by sphere B,
w A = Weight of sphere A.
θ N ′ cos θ = R
w
Force acting on cylinder N ′ sin θ + w A = N
f
N
N (R)
R = Normal force on cylinder by ground, B
f = Force of friction by ground by cylinder,
R' R
w = Weight of cylinder. θ O Force of sphere B
N cos θ = f
w + N sin θ = R, wB
N ( R) = Reaction to N,
N(R) R′ = Normal force on sphere B by right
Q wall,
O
(Force acting on N ( R) = Reaction to N i.e. normal force on
R'
plank)
w θ sphere B by sphere A,
f' P Ground wB = Weight of sphere B,
i.e.,normal force on plank by cylinder R′, N ( R) and wB pass through point O, the
R′ = Normal force on plank by ground, centre sphere B.
w = Weight of plank, 3. N = Normal force on sphere by wall,
f ′ = frictional force on plank by ground.
C
Resultant of f ′ and R′, N ( R) and w pass
through point O. θ

2. N B
O N
A
A
θ R Force on sphere A

w w
N'
Laws of Motion 75
w = Weight of sphere, 7. R cos 30° + 3 = f cos 60°
T = Tension in string. R f

4. Component of F1 60°
30°
along x-axis : 4 cos 30° = 2 3 N 3N

along y-axis : 4 sin 30° = 2 N 10 N


→ 60°
Component of F2
R 3 f
along x-axis : 4 cos 120° = − 2 N i.e., +3=
2 2
along y-axis : 4 sin 120° = 2 3 N or R 3 +6= f …(i)

Component of F3 and R sin 30° + f sin 60° = 10
1 3
along x-axis : 6 cos 270° = 0 N i.e., R +f = 10
2 2
along y-axis : 6 sin 270° = − 6 N or R + f 3 = 20 …(ii)

Component of F4 Substituting the value of f from Eq. (i) in
Eq. (ii)
along x-axis : 4 cos 0° = 4 N R + ( R 3 + 6) 3 = 20
along y-axis : 4 sin 0° = 0 N 4 R + 6 3 = 20
5. Taking moment about point A 20 − 6 3
⇒ R= = 2.4 N
T T sin 30°
4
∴ f = (2.4) 3 + 6
= 10.16 N
O 30°
A AB = l
B 8. At point B (instantaneous vertical
acceleration only)
w
N
AB = l A
l T
( T sin 30° ) l = w
2 T
45°
⇒ T=w B
6. See figure (answer to question no. 3)
OA
sin θ = mg
OB + BC
a 1 ∴ mg − T sin 45° = ma …(i)
= =
a+a 2 At point A (instantaneous horizontal
T cos 30° = w acceleration only)
3 ∴ T cos 45° = ma …(ii)
or T =w
2 Combining Eqs. (i) and (ii)
2 mg − ma = ma
or T= w
3 g
⇒ a=
2
76 | Mechanics-1
Introductory Exercise 5.2
1. Acceleration of system
( + 120) + ( − 50) nt)
a= n sta
co
1+ 4 +2 v(

= 10 m/s2
T θ
Let normal force between 1 kg block and
4 kg block = F1
mg
∴ Net force on 1 kg block = 120 − N
120 − F1
∴ a=
1 θ =30°
or 10 = 120 − F1
i.e., F1 = 110 N Tension in spring = mg
Net force on 2 kg block = 2 × a = 1 × 10
= 2 × 10 = 10 N
= 20 N 5. Pseudo force ( = ma) on plumb-bob will be
2. As, 4 g sin 30° > 2 g sin 30° as shown in figure
The normal force between the two blocks a
will be zero.
mg T φ
3. N ( R) = ma
4
θ
°–
90 mg
N
a ma θ = 30°

A T cos φ = mg + ma cos (90° − θ)


i.e., T cos φ = mg + ma sin θ …(i)
mg and T sin φ = ma cos θ
N (R) = N
Squaring and adding Eqs. (i) and (ii),
mg T2 = m2 g2 + m2 a2 sin2 θ + 2m2 ag sin θ
∴ N=
4
+ m2 a2 cos2 θ …(iii)
As lift is moving downward with T = m g + m2 a2 + m2 ag
2 2 2
(Qθ = 30°)
acceleration a, the pseudo force on A will
g2 g g
be ma acting in the upward direction. = m2 g2 + m2 + m2 ⋅ g (Q a = )
For the block to be at rest w.r.t. lift. 4 2 2
7 m2 g2
N + ma = mg =
mg 4
or + ma = mg mg 7
4 or T=
3 2
⇒ a= g
4 =5 7 N
4. Angle made by the string with the normal Dividing Eq. (i) by Eq. (ii),
to the ceiling = θ = 30° ma cos θ
tan φ =
As the train is moving with constant mg + ma sin θ
velocity no pseudo force will act on the
plumb-bob.
Laws of Motion 77
a cos θ 6. A B
= 2kg 1kg F=8N
g + a sin θ
cos θ 8
= a= = 2 m/s 2
2 + sin θ 2+1+1
cos 30°
= T1 T1 B
2 + sin 30° 2kg 1kg 8N
A T2 T2
3
=
5 Net force on 1 kg mass = 8 − T2
 3 ∴ 8 − T2 = 1 × 2
i.e., φ = tan −1  
 5  ⇒ T2 = 6 N
Net force on 1 kg block = T1
∴ T1 = 2a = 2 × 2 = 4 N

Introductory Exercise 5.3


1. F = 2 g sin 30° = g

T1
T1
T1 T1
T2 F T1
T2
T1
mg
T1
T2
T2 4g
T2 3g
T2
2g

For the system to remain at rest 1g

T2 = 2 g …(i) ∴ T1 = 4 g
T2 + F = T1 …(ii) and T2 = 1 g
or T2 + g = T1 …[ii (a)] T1
∴ =4
T1 = mg …(iii) T2
Substituting the values of T1 and T2 from 3. 2 g − T = 2a
Eqs. (iii) and (i) in Eq. [ii(a)]
2g + g = mg
i.e., m = 3 kg
T T
2. As net downward force on the system is
zero, the system will be in equilibrium T
a T
a
1g

2g
78 | Mechanics-1
T − 1 g = 1a u2 ( g / 3)2 g
= = =
Adding above two equations 2a 2g 18
1g = 3a Downward displacement of 2 kg block
g 1 1 g
2
g
∴ a= = at2 = g ⋅   =
3 2 2 3 18
Velocity of 1kg block 1 section after the
As the two are just equal, the string will
system is set in motion 1
again become taut after time s.
v = 0 + at 3
g
= ⋅1 4. F + 1 g − T = 1a …(i)
3
g
= (upward)
3
On stopping 2 kg, the block of 1kg will go
upwards with retardation g. Time ( t′ ) T T
taken by the 1 kg block to attain zero T
a
velocity will be given by the equation.
g
0 =   + ( − g) t ′ 1g
T
a
3 F
1
⇒ t′ = s
3 2g

If the 2 kg block is stopped just for a and T − 2 g = 2a …(ii)


moment (time being much-much less than
1 Adding Eqs. (i) and (ii),
s), it will also start falling down when
3 F − 1g = 3a
the stopping time ends. 20 − 10 10
∴ a= = ms −2
1 3 3
In t′  = s time upward displacement of
 3 
1 kg block

Introductory Exercise 5.4


1. 2T = 2 × a …(i) ∴ Acceleration of 1 kg block
2 g 20
a 2a = = ms −2
T T 3 3
2T 2T Tension in the string
2 kg T T
g 10
T T= = N
3 3
T
2. Mg − T = Ma …(i)
2a
T T
1g T T
M
T
and 1 g − T = 2a …(ii) a T
Solving Eqs. (i) and (ii), a
g
a= mg
3
Laws of Motion 79
g
T = Ma …(ii) or T = 4  g + 
 5
Solving Eqs. (i) and (ii)
g 24 g
a= =
2 5
and T = Mg Substituting value of above value of T in
3. Block of mass M will be at rest if Eq. (i),
24
M=
5
= 4.8 kg
T
T T 4. = m1 ⋅ 2a
T
2
M a
T T/2 T/2
Mg 2a T T
T/2 T/2 F

T/2 T/2
T/2 i.e., T = 4 m1a …(i)
T/2 F − T = m2 a …(ii)
a a
3g or F − 4 m1a = m2 a
F
2g or a=
4 m1 + m2
T = Mg …(i) 0.40
=
For the motion of block of mass 3 kg ( 4 × 0.3) + 0.2
T
3g − = 3a …(ii) 0.40
2 =
1.4
For the motion of block of mass 2 kg 2
T = ms −2
− 2 g = 2a …(iii) 7
2
∴ T = 4 m1a
Adding Eqs. (ii) and (iii),
2
g = 5a = 4 × 0.3 ×
g 7
i.e., a= 2.4
5 =
7
Substituting above value of a in Eq. (iii),
12
T = N
= 2 ( g + a) 35
2
80 | Mechanics-1
Introductory Exercise 5.5
1. Block on triangular block will not slip if 1 2
2. (a) Using s = s0 + ut +at
m1a cos θ N 2
Displacement of block at time t relative to
car would be
m1a
a = 5i ms–2
m1 g m1g sin θ a = – 5i ms–2
N sin θ cos u = 10i ms–1 v=0
θ
m1a sin θ (at t = 0)
θ O x
T x0 Car
N
T 1
M a x = x0 + 10 t + ( −5) t2
2
Mg or x = x0 + 10 t − 2.5 t2
Velocity of block at time t (relative to car)
m1a cos θ = m1 g sin θ will be
i.e., a = g tan θ …(i) dx
v= = 10 − 5t
N = m1 g cos θ + m1a sin θ …(ii) dt
For the movement of triangular block (b) Time ( t) for the block to arrive at the
T − N sin θ = m2 a …(iii) original position (i.e., x = x0 ) relative to
car
For the movement of the block of mass M
x0 = x0 + 10 t − 2.5 t2
Mg − T = Ma …(iv)
⇒ t=4s
Adding Eqs. (iii) and (iv),
3. (a) In car’s frame position of object at time
Mg − N sin θ = ( m2 + M ) a
t would be given by
Substituting the value of N from Eq. (ii) in In car’s frame
the above equation
(at t = 0 s)
Mg − ( m1 g cos θ + m1a sin θ) sin θ O x0
z0 x a = 5i ms–2
= ( m2 a + Ma) v=0
i.e., M ( g − a) = m1 g cos θ sin θ a = –5 ms–2
u = 10k ms–1
+ ( m2 + m1 sin2 θ) a
Substituting value of a from Eq. (i) in the
above equation,
z
M (1 − tan θ) = m1 cos θ sin θ
1
+ ( m2 + m1 sin2 θ) tan θ x = x0 + 0 × t + ( − 5) t2
2
m cos θ sin θ + ( m2 + m1 sin2 θ) tan θ
∴M = 1 i.e., x = x0 − 2.5 t2 …(i)
(1 − tan θ)
and z = z + 10 t …(ii)
Substituting θ = 30° , m1 = 1 kg and Velocity of the object at time t would be
m2 = 4 kg dx
vx = = − 5t …(iii)
(cos 30° sin 30°+4 + sin2 30° ) tan 30° dt
M=
(1 − tan 30° ) dz
and vz = = 10 …(iv)
0.443 + ( 4.25) (0.577) dt
=
0.423 (b) In ground frame the position of the
= 6.82 kg object at time t would be given by
Laws of Motion 81
2
In ground frame ∴ Net deceleration = (3 + 5) m/s
At t = 0 = 8 m/s 2
O x0
z0 a = 5i ms–2 ∴ Displacement of object at any time t
v=0 (relative to car)
1
u = 10k ms–1 x = x0 + 10t + ( −8) t2
2
or x = x0 + 10t − 4 t2
Thus, velocity of object at any time t
x = x0
(relative to car)
and z = z0 + 10 t dx
vx = = 10 − 8 t
Velocity of the object at time t would be dt
dx
vx = =0 The object will stop moving relative to car
dt when
dz
and vz = = 10 ms −1 10 − 8 t = 0 i.e., t = 1.25 s
dt
∴ vx = 10 − 8 t for 0 < t < 1.25 s
4. m = 2 kg
For block not to slide the frictional force
u = 10i ms–1
x0 ( f ) would be given by
O x a = 5i ms–2 f a = 3 ms–2
v=0
ma
mg sin θ
θ θ = 37°
z
Normal force on object = mg f + ma cos θ = mg sin θ
Maximum sliding friction = µ s mg or f = mg sin θ − ma cos θ
3 4
= 0.3 × 2 × 10 = 6 N = m × 10 × − m × 3 ×
6 5 5
Deceleration due to friction = = 3 m/s2 18 m 9
2 = = mg
5 25
Deceleration due to pseudo force = 5 m/s2

AIEEE Corner
Subjective Questions (Level 1)
1. FBD is given in the answer. T1 T1
At point P, F1 = and T2 =
2 2
2. FBD is given in the answer. T T1
At point Q, F2 = 1 and W =
3. FBD is given in the answer. 2 2
4. 5.
T2

F1 NB
P 45°
T1 30°
T1 NA
45° W

Q F2
N A sin 30° = N B
and N A cos 30° = W
W
82 | Mechanics-1
6. T2 3T
V + =W …(i)
2
45° T
H= …(ii)
T1 2
Net moment about O = zero
l 3T
∴. W × = ×l …(iii)
W 2 2
100 − 40
T2 10. (a) a = = 3 m/s2
= W and 6 + 4 + 10
2
T2 (b) Net force = ma
= T1
2 ∴ F6 = 18 N, F4 = 12 N and F10 = 30 N
7. f
(c) N − 40 = F10 = 30
∴ N = 70 N.
F 60
11. a = = = 1 m/s2
m1 + m2 + m3 60
40 N (a) T1 = m1a = 10 N
x
T2 − T1 = m2 a
N
∴ T2 − 10 = 20
10 cm ∴ T2 = 30 N
W = 20 N
(b) T1 = 0 . New acceleration
N = 40 …(i) 60
a′ = = 1.2 m /s2
f = 20 …(ii) 50
and f × 10 = N × x …(iii) T2 = m2 a ′ = 24 N
8. N 12. (a) T1

T 0.1 kg
a

30° 1.9 kg 2g

f
T1 − 2 g = 2a
30° (b) T2

W
2.9 kg
T + f cos 30° = N sin 30° …(i)
N cos 30°+ f sin 30° = W …(ii) a 0.2 kg
T × R = fR …(iii)
9. V 1.9 kg 5g
3T
2

O H T/2
T2 − 5 g = 5a

W
Laws of Motion 83
200 − 16 g 20.
13. (a) a = a
16 T2 = T1
(b) T1 − 11 g = 11a 1
(c) T2 − 9 g = 9 a
T1
14. If the monkey exerts a force F on the rope
upwards, then same force F transfers to T1 a
bananas also. If monkey releases her hold 2
on rope both monkey and bananas fall
freely under gravity. 2T1
15. Tension on B = T
Tension on A = 3 T T1
1 T1
Now in these situations a ∝ ar
2
T
ar
16. x A + xC + 2xB = constant. 3

Differentiating twice w.r.t. time we get the T1 = 1a …(i)


acceleration relation.
T1 − 20 = 2 ( ar − a / 2) …(ii)
17. 30 − T1 = 3( ar + a / 2) …(iii)
aB 21. a T 2T
θ
aA M a
θ 2
° 2M
30
n
aA si
= sin θ g
M
aB 2 Mg

∴ a A = aB sin θ T − Mg sin 30° = Ma …(i)


18. x + y = 6 …(i) a
2Mg − 2T = 2M ⋅ …(ii)
y−x=4 …(ii) 2
Solving, we get 22. T = 1a …(i)
x = 1 m/s2 x 10 − T = 1a …(ii)
23. 2T 2T
1

x
B A 2a
a
y
50 40
2
y 50 − 2T = 5a …(i)
3
T − 40 = 4(2a) …(ii)
7g − 3g 24. (a) N = 40 N, µ s N = 24 N
19. a= = 4 m/s2
10 F < µsN
40 − T1 = 4 a ∴ f = 20 N and a = 0
30 + T1 − T2 = 3 a (b) N = 20 N, µ s N = 12 N
T3 − 10 = 1a and µ k N = 8 N
84 | Mechanics-1
F > µsN 27. f = (06
. )(2)(10) = 12 N
∴ f = µ kN = 8 N a2
20 − 8 2 kg
and a= = 6 m/s 2 f
–ve +ve
2
a1
(c) N = 60 − 20 = 40 N
f
1 kg
µ s N = 8 N and
µ kN = 4 N 12
a2 = = 6 m/s2
2
Since, F cos 45° > µ s N
12
and a1 = = 12 m/s2
∴ f = µ kN = 4 N 1
20 − 4 (a) Relative motion will stop when
and a=
6
v1 = v2
8
= m/s2 or u1 + a1t = u2 + a2 t
3
or 3 − 6t = −18 + 12t
25. a = µ g = 3 m/s2 7
∴ t= s
(a) v = at 6
∴ 6 = 3 t or t = 2 s (b) Common velocity at this instant is
1 v1 or v2 .
(b) s = at2
2 1
(c) s1 = u1t + a1t2 and
1 2
∴ s = × 3 × 4 = 6 m.
2 1
s2 = u2 t + a2 t2
26. f = 0.4 × 1 × 10 = 4 N 2
28. N = 20 N
a1
µ s N = 16 N
1 kg
f and µ k N = 12 N
a2 Since, W = 20 N > µ s N, friction µ k N will
act.
2 kg
20 − 12
f ∴ a= = 4 m/s2
2
f 29. N = 20 N
a1 = = 4 m/s2
1 µN = 16 N
f Block will start moving when
a2 = = 2 m/s2
2 F = µN
(a) Relative motion will stop when or 2t = 16
v1 = v2 or t = 8 s.
or 2 + 4 t = 8 − 2t After 8 s
2t − 16
∴ t = 1s a= = t−8
2
(b) v1 = v2 = 2 + 4 × 1 = 6 m/s
1 i.e., a-t graph is a straight line with
(c) s1 = u1t + a1t2 positive slope and negative intercept.
2
1 30. N = 60 N, µ s N = 36 N, µ k N = 24 N
s2 = u2 t − a2 t2
2 Block will start moving when
F = µsN
Laws of Motion 85
or 4 t = 36 (b) v
F
∴ t=9s 12 N

After 9 s
4 t − 24 2
a= = t−4 52 N
6 3
31. N = mg cos θ = 30 N F + 12 = 52
∴ F = 40 N
mg sin 30° = 30 3 N ≈ 52 N.
(c) a F
µ s N = 18 N and
µ k N = 12 N
(a) F
18 N 52 N
12 N

F − 52 − 12 = 6 × 4
52 N
∴ F = 88 N
F = 52 − 18 = 34 N.

Objective Questions (Level-1)


Single Correct Option
mg − F F mg − T = ma
1. a = =g−
m m Tmax
∴ amin = g −
m A > mB m
∴ a A > aB 2
mg
g
or ball A reaches earlier. =g−3 =
4 g − 2g g m 3
2. a = = 10 g − 5 g g
6 3 5. a = =
g 15 3
Now, 2 g − T = 2 g
3 10 g − T = 10 ×
4g 3
∴ T= = 13 N 20 g
3 ∴ T= = Reading of spring
3
3. T1
balance.
30° 2mg sin 30°− mg
6. a = =0
T2 3m
∴ T = mg
7. a1 = g sin θ − µ g cos θ = g sin 45°−µ g cos 45°
100 N
a2 = g sin θ = g sin 45°
3 T1 2s 1
= 100 …(i) Now t = or t ∝
2 a a
T1 t1 a2
= T2 …(ii) ∴ =
2 t2 a1
4. T
g sin 45°
a or 2=
g sin 45°− µ g cos 45°
3
Solving, we get µ =
mg 4
86 | Mechanics-1
8. F1 = mg sin θ + µmg cos θ 14. f = mg sin θ (if block is at rest)
F2 = mg sin θ − µmg cos θ 15. 2T cos 30° = F
Given that F1 = 2F2
60° T
9. For equilibrium of block, net force from
30°
plane should be equal and opposite of a F
weight.
10. No solution is required.
11. Angle of repose θ = tan −1(µ ) = 30° F
∴ T=
3
T cos 60°
a=
m
θ
F
=
2 3m
h
16. F
 3
∴ h = R − R cos θ = 1 − R θ
 2 
12. Net pulling force = 15 g − 5 g = 10 g = F
Net retarding force = (0.2)(5g) = g = f N = mg − F sin θ
F−f 9 µN = (tan φ) N = (tan φ)( mg − F sin θ)
∴ a= = g
25 25 F cos θ = µN
9 17. µmg = 0.2 × 4 × 10 = 8 N
T1 − 5 g = 5a = g
5
At t = 2 s, F = 4 N
34
∴ T1 = g Since F < µmg
5
27 ∴ Force of friction f = F = 4 N
15 g − T2 = 15a = g
5 18. a1 = g sin θ
48 a2 = g sin θ − µg cos θ
∴ T2 = g
5
25 1
T1 17 t= or t ∝
∴ = a a
T2 24
t1 a2
13. a =
t2 a1
1 g sin θ − µg cos θ
=
2 g sin θ

θ v 19. Net accelaration of man relative to ground


Relative to lift, ar = ( g + a) sin θ along the = a + a = 2a
plane. T − mg = m (2a)
2s ∴ T = m( g + 2a)
Now, t =
ar
20. a=0
2L 3 F = (50 + 25) g = 75 g
=
( g + a) sin θ ∴ F = 25 g = 250 N
Laws of Motion 87
21. A N1
1 + µ2
µ
4m N2
θ
W B
1
O 3m
µN2
or tan θ = µ
µmg
N1 = µ N2 Fmin =
1 µ2
N2 = W +
1 + µ2 1 + µ2
Net moment about B should be zero.
3 µ
∴ W × = N1 × 4 = mg
2 1 + µ2
22. 1
N1 = mg
2
N2 25. µmg = 32 N

W Since ∴ f = 30 N
θ
N 30 N > µmg
f
∴ Normal reaction N = mg = 40 N
N2 = W = 250 (always) ∴ Net contact force = (30)2 + ( 40)2
∴ fmax = µN2 = 75 N
= 50 N
1
Mg − Mg 26.
2 g 0.2
23. a = = 2 kg
M 2
a 8 kg 25 N
F
f f
0.5
1 M M Mg
T− × ×g= ×a= fmax = (0. 5)(8 + 2)(10) = 50 N > 25 N
2 2 2 4
Mg ∴ Blocks will not move and therefore force
∴ T= of friction between two blocks = 0.
2
3
24. F 27. mg sin θ = 10 × 10 × = 60 N
5
θ
This 60 N > 30 N
∴ Force of friction is upwards.
Net contact force is resultant of friction
N = mg − F sin θ and normal reaction.
F cos θ = µ N = µ ( mg − F sin θ)
µ mg 28. In first figure T = F
F=
cos θ + µ sin θ and in second figure, T = 2F .
mg 
For F to be minimum,
dF
=0 29. F = 4   → upwards
dθ  2 
W = 2mg → downwards
F =W
∴ a=0
88 | Mechanics-1
2 µm g
30. ∴ t=
m a

m
31. a1 = g sin 30° = g/2
F = at
3
a2 = g sin 60° = g
Maximum acceleration of upper block due 2
→ →
to friction Angle between a 1 and a 2 is 30°.
µmg
amax = =µ g ∴
m
F at → → ( g/2)2 + ( 3 g/2)2 − 2 ( g/2)
= =µ g ar = |a 1 − a 2|=
2m 2m ( 3 g/2) cos 30°
= g/2
JEE Corner
Assertion and Reason
1. Even if net force = 0, rotational motion can m1 g − T = m1a
take place. ∴ T = m1( g − a)
2. No solution is required. 2m1m2 g
=
( m1 + m2 )
3. a1 = g sin θ + µ g cos θ
Mathematically, we can prove that
a2 = g sin θ − µ g cos θ
a1 1 + µ m2 g < T < m1 g
= as θ = 45° and sin θ = cos θ
a2 1 − µ Similarly if m2 > m1, then we can prove
1 that,
Substituting µ = we have,
3 m1 g < T < m2 g
a1 1 + 1 / 3 2
= = 8. If accelerations of both the frames are
a2 1 − 1 / 3 1
same then one frame as observed from
4. There is no force for providing ( − 2 i$) m/s2 other frame will be inertial.
to the block. Further, a frame moving with constant
5. If we increase F1, maximum value of velocity is inertial.

friction will increase. But if we increase F2 9. No solution is required.


friction acting on the block will increase. 10. No solution is required.
6. No solution is required. 11. Force of friction is in the direction of
( m1 − m2 ) g motion.
7. a = (if m1 > m2 )
m1 + m2
Laws of Motion 89
Objective Questions (Level 2)
Single Correct Option
1. Tmax = Fmax + fmax + 2T ′ max 3. N sin θ will accelerate B towards left.
T' T' N
A
f T T A 90° – θ
T T ma (pseudo
B 2T
T' T' force)
N sin θ mg
F co
T θ sθ
T N B
θ

Let a be the acceleration of B.


mg Due to acceleration of B towards left
pseudo force equal to ma will act on block,
= µ ( m A + mB ) g + µ m A g + 2µm A g toward right.
= µ ( 4m A + mB ) g Thus, N + ma sin θ = mg cos θ
But, Tmax = M max ⋅ g
∴ M max ⋅ g = µ ( 4 m A + mB ) g ⇒ N = mg cos θ − ma sin θ
i.e., M max = µ ( 4 m A + mB ) Option (b) is correct.
= 0.3 × [( 4 × 100) + 70] = 81 kg 4. If a ≤ µg
Option (c) is correct.
→ m1
2. v = 8 t ^i − 2 t2 ^j m2 a

→ dv
∴ a=
dt m2a
^ ^ m1a
= 8 i − 4t j m1 m2 N=0
y f1 f2
N=0

When a is f1 N f2

a
Zero Zero Zero Zero
µg µg Zero µg
2i m1 m2
x 2 2 2
µg m1 µg Zero m2 µg
4j →

a (Not yet (Attained
g attained limiting
limiting value)
∴ Force (pseudo) on sphere value
→ which is
Fs = 1 ⋅ ( − 8 ^i + 4 ^j ) 2 µm1 g)

= − 8 ^i + 4 ^j Now, if a > µg, the block of mass m2 would


be just greater like to move towards left
Gravitational force on sphere than µg as a then
= − mg ^j = − 10 ^j m2 a > f2 (max)
∴ Net force on sphere While the block of mass m1 will remain at
→ rest as then f1 has margin to increase.
F = − 8 ^i + 4 ^j − 10 ^j = − 8 ^i − 6 ^j Thus, m1 will apply normal force ( N ) or m2
→ and so will do m1 on m2 to stop its motion
|F |= 10 N
(towards left).
Option (b) is correct. Option (d) is correct.
90 | Mechanics-1
5. N + ma sin θ = mg cos θ Fig. 3
N
T
ma sin θ a3 T T
ma (pseudo T a3
a force) θ
in
gs
m1
mg sin θ mg θ
m2g

m2 g − T = m2 a3
The block will fall freely if N = 0 and T − m1 g sin θ = m1a3
i.e., ma sin θ = mg cos θ m − m1 sin θ
∴ a3 = 2 g
⇒ a = g cos θ m1 + m2
Option (c) is correct. Substituting m1 = 4 kg, m2 = 3 kg
6. Speed of A w.r.t. C = v1 and θ = 30°
A 3
Speed of C w.r.t. a1 = g
ground = v0 4+3
∴ Speed of A w.r.t. 3
C B = g
ground = v1 − v0 7
Now, speed of B w.r.t. 4−3
a2 = g
ground = speed of A is 4+3
w.r.t. ground 1
∴ v2 = v1 − v0 = g
7
⇒ v1 − v2 = v0 1
3 − 42 ⋅
Option (a) is correct. a3 = 2g
4+3
7. Fig. 1
1
T T = g
m1 7
∴ a1 > a2 = 3
T
T Option (b) is correct.
8. F − fmax − T = ma (For lower block)
m2g T T

f f
m2 g − T = m2 a1
m2 F
and a1 = g T T
m1 + m2
Fig. 2 T − fmax = ma (For upper block)
⇒ F − 2 fmax = 2ma
m1 g − T = m2 a2
or F − 2 µmg = 2ma
and T − m2 g = m1a2 F
m − m2 T T or a= −µ g
⇒ a2 = 1 g 2m
m1 + m2 T
Option (c) is correct.
a1
T 9. a2 cos(90° − θ) = a1
a2
mg θ
2 i.e., a2 sin θ = a1 [θ < 0°] a2
mg
1 Option (b) is correct. θ
Laws of Motion 91
10. Had θ been 90°. M−m 2T
⇒ a= g
M+m 2T

∴ T = m ( g + a)
v1
 M−m 
= m g + g T T
v2
v1 90° – θ θ
 M + m 
m T
 M − m
B = mg 1 + m T
w  M + m 
= mg [1 + 1] mg
For every x displacement of wedge ( w) the
{as m << M , M − m ≈ M and Mg
vertical fall in mass would have been 2x as
M + m ≈ M}
the string passes through pulley B.
= 2 mg
i.e., v1 = 2 v2
∴ Tension in the string suspended from
For the situation given the speed of mass ceiling = 2T = 4 mg
would thus be
Option (a) is correct.
v1 = 2 v2 cos (90° − θ)
i.e., v1 = 2 v2 sin θ 14. For 0° < θ ≤ φ frictional force ( F ) = mg sin θ
For θ ≥ φ frictional force ( F ) = µ mg cos θ
Option (c) is correct.
[φ = angle of repose]
11. The cylinder will start rising up the
For example
inclined plane if 1 1
Let µ = ∴ tan φ =
ma cos θ 3 3
ma φ = 30°
(Pseudo O
θ F (force of friction) Condition
furce)
10° mg sin 10° = 0.174 mg
20° mg sin 20° = 0.342 mg Increase but not
ma cos θ > mg sin θ linearly only
i.e., a > g tan θ 30° µmg cos 30° = 0.500 mg
∴ amin = g tan θ [cylinder will be at the 40° µmg cos 40° = 0.442 mg Decrease but not
point of rising up the inclined plane] linearly only
12. At the position of maximum deflection the 60° µmg cos 60° = 0.287 mg
net acceleration of bob (towards its mean 90° Zero
position will be)
g sin θ − a cos θ
as explained in figure. F

a cos θ
O φ 90° θ
a
Option (b) is correct.
90°–

g cos (90° – θ)
µ mg
θ

= g sin θ 15. aB =
g 2m
µg
=
13. Mg − T = Ma 2
T − mg = ma
92 | Mechanics-1
F F θ
µmg Am cos
ma
B F
a
µmg F
2m
F θ
sin
mg θ

For no slipping ∴ Time ( t) to move s distance would be


a A = aB given by
F − µ mg µ g 1
= s = g eff t2
m 2 2
3 2s 2s
i.e., F = µmg i.e., t = =
2 g eff ( a cos θ − g sin θ)
2 F
µ= Substituting s = 1 m,
3 mg
θ = 30°,
Slipping will obviously be m2 there if µ is a = 10 3 m/s 2 and g = 10 m/s 2
greater than above mentioned value
2 F 2×1
∴ µ min = t=
3 mg  3  1
10 3 ×  _ 10 × 
For no slipping.  2  2
Option (c) is correct. 1
= s
5
16. Fnet (downward) = mg sin θ + ma cos θ
Option (b) is correct.
a w w
18. N + sin 30° = w +
ma 2 2
w/2
ma cos θ N w sin 30°
mg sin θ 2 w/2
θ
30° w cos 30°
= m ( g sin θ + a cos θ) 2
∴ g eff = g sin θ + a cos θ f
w
Time ( T) required to cover 2L distance
along inclined would be f = frictional force
2L 5w
T= N=
g eff 4
5w
2L fmax = µ N = µ ⋅
= 4
( g sin θ + a cos θ) The block will remain stationary if
w µ 5w
Option (c) is correct. cos 30° ≤
2 4
17. Fnet on block along incline in the upward w 3 5w
direction or ≤µ
2 2 4
= ma cos θ − mg sin θ
or 3 ≤ 5µ
= m ( a cos θ − g sin θ) 3
or ≤µ
∴ g eff = a cos θ − g sin θ 5
3
∴ Block will move if µ <
5
Option (d) is correct.
Laws of Motion | 93
19. F1 = 2t T T 20. 2 mg sin α − T = 2ma
F = 15N
y
f1 µ1 = 0.6 f2 µ2 = 0.5 60° j
At t = 2 s P x
i
F1 = 4 N T
T T
2m
4N T T
B A 15N T
T T m
α = 30°

f1 and f2 are the frictional forces and T = ma


( f1) max = 0.6 × 1 × 10 ∴ 2 mg sin α = 3 ma
= 6N 2 g sin 30° g
⇒ a= =
( f2 ) max = 0.5 × 2 × 10 3 3
mg
= 10 N ∴ T=
3
At t = 2 s
Force ( R) applied by clamp on pulley
Net external force ( Fnet ) on system would be
= 15 N − 4 N → →
|T1|= |T2|= T
= 11 N → mg mg
T1 = (cos 30° ) ^i + (sin 30° ) ^j
As Fnet > ( f1) max + ( f2 ) max , the system will 3 3
remain at rest and the values frictional mg 3 ^ mg ^
forces on the blocks will be given = i+ j
6 6
T = 4 + f1 and T = 15 − f2 → mg ^
|T2|= j
4 + f1 = 15 − f2 …(i) 3
f1 + f2 = 11 N Force by clamp on pulley P
Let direction being + ive for Eq. (i) y →
T2

Option (a) f1 = − 4 N, f2 = = 5 N T1

⇒ f1 + f2 = 1 N wrong 60°
Option (b) f1 = − 2 N, f2 = + 5 N x
→ →
⇒ f1 + f2 = 3 N wrong = T1 + T2
Option (c) f1 = 0 N,f2 = + 10 N mg 3 ^ mg ^ mg ^
= i+ j+ j
⇒ f1 + f2 = 10 N wrong 6 6 3
Option (d) f1 = + 1 N, f2 = + 10 N mg 3 $ 3 mg $ mg
= i+ j = ( 3 ^i + 3 ^j)
⇒ f1 + f2 = 11 N correct. 6 6 6
OR Option (b) is correct.
As the likely movement would be towards 21. f1(max) = 0.3 × 4 × 10 = 12 N
right f2 will be at its maximum. 2 kg 4 kg 16N
∴ f2 = 10 N f2 µ1 = 0.6 f1 µ2 = 0.3
⇒ f1 = 1 N
f1 and f2 are frictional forces.
Option (d) is correct. f2 (max) = 0.6 × 2 × 10 = 12 N
As, f1(max) + f2 (max) < 16 N ( Fext )
94 | Mechanics-1
T T 1
2 kg 4 kg 16N
∴ x= m
f2 f1 2
Option (a) is correct.
The system will remain at rest.
25. N = ma sin θ + mg cos θ …(i)
For the equilibrium of 4 kg mass : N
∴ 16 = T + f1 …(i) ma sin θ
As f1 will be at its maximum value
ma
f1 = 12 N a
∴ T = 16 − 12 mg sin θ
mg cos θ
= 4 N [from Eq. (i)] θ
Further, for the equilibrium of 2 kg mass.
Now, as the block does not slide
T = f1
ma cos θ = mg sin θ
∴ f1 = 4 N
i.e., a = g tan θ
Option (c) is correct.
Substituting the found value of a in Eq. (i)
22. For the rotational equilibrium of rod
Taking moment about O. N = m ( g tan θ) sin θ + mg cos θ
l l sin2 θ 
R × cos θ = s sin θ = mg  θ + cos θ = mg sec θ
2 2  cos 
When, the block stops a = 0, the value of
A s (= ma) normal force will be
R (= mg) a
ma
O N ′ = mg cos θ
N ′ mg cos θ
mg θ ∴ =
N mg sec θ
or mg cos θ = ma sin θ
⇒ a = g cotθ Option (c) is correct.
Option (d) is correct. 26. For the rotational equilibrium of the block
2 N
23. v = 2 t
dv d f
∴ =a= (2 t2 ) = 4 t B
O
dt dt φ
x
At t = 1 s, a = 4 ms −2 mg sin θ
mg cos θ
A
As a = µsg
a 4
µs = = = 0.4 mg
g 10
θ = 45°
Option (c) is correct.
Taking moment about O.
24. Just at the position of tipping off, R1 will a
be zero. Nx = f
2
R1 R2 a
or ( mg cos θ) x = ( mg sin θ)
8m 2
A P B
a
1m
or x = tan θ
2
x x
10 g or = tan θ
1m a/2
∴ Taking moment about point Q or tan φ = tan θ
(10 g) ( 4) = (80 g) ( x) or φ=θ
Laws of Motion | 95
Thus, the normal force ( N ) will pass 30. N sin θ = mB aB …(i)
through point A. 9 ms–2
N
Option (a) is correct. A
[Note : The cube will be just at the point of N sin θ
tilting (about point A). The cube will tilt if θ is
made greater than 45°]. N
θ = 37° B
27. For the rotational equilibrium of the cube
Moment of couple ( N , mg) a A = aB tan θ …(ii)
= Moment of couple ( F , f ) aA
⇒ N sin θ = mB
∴ mgx = Fa
N (= mg) tan θ
mg mB a A
mg F (= 3 ) N=
or mgx = ⋅a sin θ tan θ
3 x
a
a ∴ Force on rod by wedge
i.e., x =
3 f=F mg
Option (b) is correct. →
aA

aA = | a A |
28. Taking moment about point O. →
θ aB = | a B |
N1 N2 →
aB
l/4 l/6 mB a A cos θ
N cos θ =
sin θ tan θ
mB a A
=
mg tan2 θ
10 × 9 5
= = 160 N 3
l l l l 2
N1  −  = N2  −  3
2 4 2 6   37°
 4
l l 4
N1 = N2 Option (c) is correct.
4 3
∴ N1 : N2 = 4 : 3 31. Net downward force on ring = mg − µ ma
Option (c) is correct. = m ( g − µa)
a
29. T f = µN
T = µma a = 4 ms–2
T N = ma L = 1m
mg
ma

∴ g eff = g − µ a
mg
2L
t=
( T / 2) = ma → Box g eff
ma mg 2L
T+ = → Pendulum with =
2 2 g −µa
respect to 2×1
box =
10 − (0.5 × 4)
ma mg
∴ 2ma + = 1
2 2 = = 0.5 s
2
∴ a= g/3
96 | Mechanics-1
32. The direction of the normal reactions 33. T − Mg = Ma
between any one hemisphere and the T
sphere will be along the centres of the two.
The three centres of the hemisphere and
that of sphere will form a tetrehadron of
edge equal to 2R.
In figure, C1, C2 and C3 are the centres of
the hemispheres and C is the centre of the
sphere
C1C2 = C2C3 = C3C1
( 500 + 80n )g
= C1C = C2C = C3C = 2R
N i.e., T = M ( g + a)
N
N ∴ 2 × 104 ≥ M ( g + a)
C
C1 or 2 × 104 ≥ (500 + 80n)(10 + 2)
or 14.58 ≥ n
or n = 14
O C3
Option (b) is correct.
φ θ
[Note : Tension in lift cable will increase
C2 when the lift is accelerated upwards].
34. Normal reaction between the surface and
∠COC2 = 90°
2R the particle will be zero throughout the
C2O = motion if the path of the particle is that of
3
a projectile motion (particle is free from
C2O
cos θ = surface).
C2C
v sin φ v
2R / 3
= φ
2R v cos φ
1 u sin θ u
= a=–g
3 + h

φ is the angle which any N makes with φ


vertical – + u cos θ

φ = 90° − θ v2 = u2 + 2as
1 √3
sin φ = cos θ = 1 (v sin φ)2 = (u sin θ)2 + 2 ( − g) h
3
2 φ v sin φ = u2 sin2 θ − 2 gh
∴ cos φ = 2
3 = (20)2 (sin2 60° )2 − 2 ⋅ 10 ⋅ 5
For the vertical equilibrium of the sphere.
3
3 N cos φ = mg =  400 ×  − 100
2  4
or 3N × = mg
3 = 10 2
mg
or N= v cos φ = u cos θ = 20 cos 60° = 10
2 3 v sin φ 10 2
Option (b) is correct. =
v cos φ 10
Laws of Motion | 97
tan φ = 2 Velocity along PQ = 20 m/s.
φ = tan −1 2 ∴ Velocity along PO = 20sin θ
Option (c) is correct. 3
= 20 × = 12 m/s
5
35. Acceleration of block B will be g
throughout its motion while that of block Velocity of sphere (along vertical
A will increase from 0 to g and as such direction)
12 12
t A < tB = = = 15 m/s
cos θ 4 / 5
Option (b) is correct.
Option (c) is correct.
36. f1 (max) = 0.5 × 10 × 10 = 50 N 38. String is winding on the motor shaft the
100 N block B will move up. Further, as shaft is
A
µ = 0.5f1 also moving down, B will further.
f1 2 m/s
B
µ = 0.25f2
C f2

Here, f1 and f2 are friction forces.


As, f1 (max.) < Fext . (100 N), block A will
B
move. 2 m/s
f2 (max.) = 0.25 (10 + 20) 10 = 75 N
As, f1 (max.), [driving force for block B] Thus,
< f2 (max.), the block will not slop over Velocity of block B
block C. = Velocity of lift + Velocity of winding of
As, there is no friction between block C string on shaft + Velocity of moving down
and surface below it, both the blocks B and of shaft
block C will move together with
q= 2 m/s + 2 m/s + 2
acceleration m/s
f1 (max.)
a= = 6 m/s.
(mass of B + C)
50 Option (d) is correct.
= 39. F ′ ′ = F ′ cos θ
(20 + 30)
F is resultant of two equal forces F ′ and F ′
= 1 ms −2  2 φ
F = 2F ′ cos  
∴ a A = 1 ms −2  2
Option (c) is correct.
R F φ
R−
37. cos θ = 5 =4
R 5
F'

20 m/s F'' F''

N = 2F ′ cos φ
= 2F ′ cos (90° − θ)
O
θ = 2F ′ sin θ 5
P  F′ ′ 
Q R/5 4
= 2  sin θ
 cos θ  θ
3
98 | Mechanics-1
= 2F ′ ′ tan θ For A to remain in contact with B, B must
= 2( ma ′ ′ ) tan θ accelerate in the − ive x-direction.
5 4 →
= 2 × 03. × × = 2 N 15 j aA (new)
2 3
Option (b) is correct.
40. Velocity of block A = 2 v cos 37 ° φ

15 i bi

2v Let acceleration of B = − b ^i
37° Due to this pseudo force with act on A in
A the + ive direction.
15
tan φ =
v
B 15 + b
3 15
4 or =
= 2 × 10 × 4 15 + b
5
or 45 + 3 b = 60
= 16 m/s
or 3 b = − 15
Option (d) is correct.
or b= −5
41. As the mass is applying maximum
possible force without moving, the blocks ∴ Acceleration of B = − 5 ^i
would at the point of slipping, Option (d) is correct.
T T
fA A 43. For block A not to slide on block B
T' T' fAT T
B N B
fC A 2F
fB F
N sin θ N sin θ
T ′ = fC (max) = 0.5 × 60 × 10 = 300 N
θ N
fB (max) = 0.3 × (60 + 60) × 10
= 360 N Fnet on block A = Fnet on block B
As, T ′ < fB (max) the value of T will be F − N sin θ = N sin θ − 2F
zero.
2 N sin θ = 3 F
Option (d) is correct.
3F
[Note : The value fB will be 300 N and the N=
values of T and f A will be zero] 2 sin θ

42.

a A = |5 ^i + 15 ^j| = 3F (as θ = 30°)
Option (d) is correct.
∴ θ = 45°
44. Equation to circle is
As,φ < θ the block will leave contact with Y
B.
aA (x, y) B
y
15 j
θ 15 i φ
x
O x
θ
B A
φ = 37° 3
tan φ = tan 37°=
4
Laws of Motion | 99
2 2 2
x + y = r (where r = OB) …(i) l1 l2
dx dy 2T
∴ 2 +2 =0
dt dt M1 2T
dx dy
⇒ =−
dt dt
= − ( −u) = + u T T
T T
=u
Speed of bead B
2 2 M3g
 dx   dy  M2 g
=   + 
 dt   dt 
M1 gl1 = 2Tl2
2 2
= (u) + ( − u)  2M1M 3 
or M1 gl1 = 2  g l2
=u 2  M2 + M 3 
 M2 
Option (a) is correct.  M2 ⋅ 
or M1l1 = 4  3 3l
45. At maximum acceleration value of a, the 1
block would be in a position to move  M2 + M2 
 3 
upwards. M1
ma cos θ ⇒ =3
N M2
47. fmax = µ k N 2
ma N√2
ma sin θ
mg sin θ
mg cos θ
fm

NN Block
ax

N = mg cos θ
a
θ = 45° Trongh

= µ k 2 mg cos θ
f = frictional force
∴ ma = mg sin θ − 2 µ k mg cos θ
N = ma sin θ + mg cos θ
i.e., a = g (sin θ − 2 µ k cos θ)
ma cos θ = fmax + mg sin θ Option (c) is correct.
or ma cos θ = µ N + mg sin θ 48. f1 (max) = 0.5 × 3 × 10
or ma cos θ = µ ( ma sin θ + mg cos θ) = 15 N
+ mg sin θ (Qθ = 45° ) f2 (max) = 0.3 × (3 + 2) × 10
or a = µ ( a + g) + g = 15 N
or a (1 − µ ) = (1 + µ ) g F
1+µ 3kg
i.e., a= g f1
f1
1−µ
2kg
f2
Option (b) is correct. f2
1 kg B
46. For the beam to have no tendency to
f3
rotate
f3 (max) = 0.1 × (3 + 2 + 1) × 10
=6N
100 | Mechanics-1
Value of maximum frictional force is 2s 2.1 2
∴ Required time t = = = s
between block 1 kg and the ground. a AB 9/2 3
Increasing from zero when F attains 6 N, Option (a) is correct.
the block of mass 1 kg will be at the point
of slipping over ground below it. 50. F = µ s mg cos θ − mg sin θ
 4 3 1
Option (c) is correct. = mg  − 
49. f2 (max) = µ ( m1 + m2 ) g 3 3 2 2
mg
m1 =
6
µ1 A
f1 f1 Option (b) is correct.
m2 B F (= 30N) 51. µ s = 2 µ k
a F
f2 µ2
µ SN
θ
s in
= 0.5 (1 + 2) 10 mg
= 15 N
aS = Acceleration of both as one θ = 30°
a A = Acceleration of A Force required to just slide block
f1 (max) = µ 1m1 g = 0.2 × 1 × 10 downwards
= 2N 0 F
a=
F − f2 (max) 30 − 15
aS = = = 5 m/s2 in
θ µ KN
m1 + m2 3 mgs
2
µ m g
a A = 1 1 = µ 1 g = 0.2 × 10 = 2 m/s2
m1
θ = 30°
As, F > f2 (max.)
F = µ s N − mg sin θ
both will move.
= µ s mg cos θ − mg sin θ
Further, as aS > a A both will not
accelerate as one unit. F = mg sin θ − µ k N
F − µ 2 ( m1 + m2 ) g − µ 1m1 g = mg sin θ − µ kmg cos θ
aB =
m2 Thus, µ s mg cos θ − mg sin θ
Acceleration of A w.r.t. B = mg sin θ − µ kmg cos θ
a AB = a A − AB (µ s + µ k) mg cos θ = 2mg sin θ
f _ µ 2 ( m1 + m2 ) g − µ 1m1 g µ + µ s  = 2 tan θ
= µ1 g −  s 
 2
m2
3 2 4
µ 1m2 g − F + µ 2 ( m1 + m2 ) g + µ 1m1 g µs = ⇒µs =
=− 2 3 3 3
m2
Option (a) is correct.
F − (µ 1 + µ 2 ) ( m1 + m2 ) g
=− F + mg sin θ − µ k mg cos θ
m2 52. a =
m
30 − (0.2 + 0.5) (1 + 2) 10 mg mg 2 3
=− + − ⋅ mg
2
9 = 6 2 3 3 2
2 m
= − m/s
2 g
=
Negative sign indicates that the direction 3
of a AB will be opposite to that of a A . Option (d) is correct.
Laws of Motion | 101
53. Minimum force required to start the For retarted motion
motion upward 02 = v2max − 2a ′ s2
= mg sin θ + µ k mg cos θ v2
⇒ s2 = max
1 4 3 2a ′
= mg  + ⋅
2 3 3 2 
 v2max  1 1 
s1 + s2 = +
7 2  a a ′ 
= mg
6 v2  8 6 
25 = max  + 
54. Minimum force required to move the block 2  g g
up the incline with constant speed 50 × 9.8
vmax = = 5.92 ms −1
= mg sin θ + µ k mg cos θ 14
1 2 3 Option (c) is correct.
= mg  + ⋅  8
 2 3 3 2  57. mg sin θ = 170 × 10 × = 906.67 N
5 15
= mg µ = 0.4
6
B f2
(5.22)2 T
55. S1 = = 14.3 m µ = 0.2 nθ
9.8  T si
2 ⋅   mg
 8  θ A
sin f1
mg
Option (c) is correct. θ
1200 g − 1000 g
56. a′ = 15
1200 f1 (max) = 0.2 × 170 × 10 × = 300 N
g 17
= 15
6 f2 (max) = 0.4 × 170 × 10 ×
17
= 600 N

17
8
T' = 1000g Stops
– a' θ
15
Speed
= vmax 25 m The whole system will accelerate as
T = 1350g
a mg sin θ is greater than both f1 (max) and
1200 g f2 (max).
Total force of friction
= f1 (max) + f2 (max)
1200 g = 900 N
1350 g − 1200 g Option (a) is correct.
a=
1200 58. mg sin θ − 300 − T = ma …(i)
g and mg sin θ + T = ma …(ii)
=
8 Substituting Eq. (i) by Eq. (ii),
For accelerated motion 2T + 300 = 0
v2max = 02 + 2as1 T = − 150 N
v2 = 150 N, compressive.
⇒ s1 = max
2a Option (a) is correct.
102 | Mechanics-1
More than One Correct Options
1. (a) Normal force between A and B = m2 g i.e., T =3N
F A
T T Option (d) is correct.
2N
2. At point A
B
2N T T
T1
α
= 1 × 10 = 10 N
T1
∴ Force of limiting friction by B on A (or by
A
A on B)
= µ × 10 = 2 N β T2

Total force opposing applied external force T2


F = 2N + T
B F =
mg
= 2N + 2N
= 4N
mg
Thus, if F ≤ 4 N
The block A will remain stationary and so T1 cos α = T2 cos β + mg …(i)
block B also. The system will be in and T1 sin α = T2 sin β …(ii)
equilibrium.
At point B
∴ Option (a) is correct.
T2 cos β = mg …(iii)
(b) If F > 4 N
T2 sin β = F − mg …(iv)
F − T −2=1a
and T − 2 = 1a …(i) Using Eq. (iii) in Eq. (i),
Adding above equation T1 cos α = 2T2 cos β …(v)
F − 4 = 2a …(ii) Dividing Eq. (ii) by Eq. (v),
i.e., F = 4 + 2a 2 tan α = tan β …(vi)
Option (a) is correct.
For F > 4 N Squaring and adding Eqs. (iii) and (v),
2a + 4 > 4 T12 = 4 T22 cos2 β + T22 sin2 β …(vii)
or a>0 Dividing Eq. (iii) by Eq. (iv)
⇒ T − 2> 0
√2
i.e., T > 2N 1
∴ Option (b) is incorrect.
β
(c) Block A will move over B only when 1
F > 4 N and then the frictional force
between the blocks will be 2 N if a is just 0 tan β = 1
[as explained in (b)]. 1
∴ cos β = = sin β
Option (c) is correct. 2
(d) If F = 6 N using Eq. (ii) Substituting the values of sin β and cos β
2a = 6 − 4 in Eq. (vii)
1 1
⇒ a = 1 m/s 2 T12 = 4 T22   + T22  
 2  2
∴ Using Eq. (i), 5
= T22
T −2=1 2
Laws of Motion | 103
2
⇒ 2T1 = 5T2 or a ( M + m sin θ) = mg cos θ sin θ
Option (c) is correct. mg cos θ sin θ
i.e., a =
M + m sin2 θ
3. Displacement of block in 4 s
0.6 × g × cos 45° sin 45°
v (ms–1) =
1.7 + (0.6 × sin2 45° )
3g
=
17 + 3
φ 3g
θ =
20
O 4 t (s)
Let aB = Acceleration of block B
S = Area under curve Net force on B (along inclined plane)
= 16 m. maB = ma cos θ + mg sin θ
∆K =Workdone by frictional force ⇒ aB = a cos θ + g sin θ
1 Thus, ( aB ) V = ( a cos θ + g sin θ) cos θ
× 1 × 42 = µ × 1 × 10 × 16
2 = a cos2 θ + g sin θ cos θ
⇒ µ = 0.1 1
= ( a + g)
Option (a) is correct. 2
 3g 1
Option (b) is incorrect. = + g
 20 2
Acceleration, a = tan φ
23 g
= tan ( π − θ) =
40
= − tan θ
( aB ) H = ( a cos θ + g sin θ) sin θ − a
= − 1 m/s 2 23 g 3 g
= −
If half rough retardation = 0.5 m/s2 40 20
1 17 g
∴ 16 = 4 t + ( − 0.5) t2 =
2 40
2
i.e., t − 16t + 64 = 0 5. f1 (max.) = µ 1m A g
or t=8s
T(pull) = 125 N
Option (d) is correct.
f1 A f1
Option (c) is incorrect.
4. Let acceleration of wedge ( A) = a B
f2
a ma sin θ N F = mg cos θ
= 0.3 × 60 × 10
ma (Pseudo
N sin θ F force) = 180 N
Fnet on B = f1 (max.) + T
θ mg
A M θ = 180 + 125
= 305 N
N + ma sin θ = mg cos θ A will remain stationary as
N = mg cos θ − ma sin θ T < f1 (max.)
Acceleration of ∴ f1 = 125 N
N sin θ Force of friction acting between A and
a=
M B = 125 N
or Ma = ( mg cos θ − ma sin θ ) sin θ ∴ Options (c) and (d) are incorrect.
104 | Mechanics-1
f2 (max) = µ 2 ( m A + mB ) g (a)
7.5 N
= 0.2 (60 + 40) 10 17.5 N
F C T B T
= 200 N
37.5 N
f1 + T = 125 + 125 = 250 N 17.5 N

As, f1 + T > f2 (max. block B /along the A T = 17.5 + 7.5 = 25 N


as A is stationary) will move towards right F = T + 37.5 + 17.5 = 80 N
with acceleration. (c) T − 7.5 − 17.5 = 4 a …(i)
Option (a) is correct. F − T − 37.5 − 17.5 = 8 a …(ii)
( f + T) − f2 (max.) F = 200 N …(iii)
aB = 1
mB + m A Solving these equations we get,
250 − 200 a = 10 m/s 2
=
40 + 60 8. Maximum value of friction available to
block is less than the maximum value of
= 0.5 m/s 2 friction available to man.
Option (b) is correct. 9. N sin θ = ma = 1 × 5 = 5 …(i)
6. (See solution to Question no. 4). N cos θ = mg = 1 × 10 = 10 …(ii)

ma
m Solving these two equations we get,
A 1
a tan θ = and N = 5 5 N.
2
10. Let f1 = friction between 2 kg and 4 kg
f2 = friction between 4 kg and ground
N = mg cos θ − ma sin θ
( fs1) max = 0.4 × 2 × 10 = 8 N
Option (c) is correct and option (d) is ( f k1) = 0.2 × 2 × 10 = 4 N
incorrect. . × 6 × 10 = 36 N
( fs2 ) max = 06
As angle between the directions of a and
Fk2 = 0.4 × 6 × 10 = 24 N
g sin θ will be less than 90°, acceleration of
block A will be more than g sin θ. (b) At t = 1 s, F = 2 N < ( fs2 ) max
A ∴ Both the blocks are at rest.
a ∴ f1 = 0
(c) At t = 4 s, F = 8 N < ( fs2 ) max
θ
∴ Both the blocks are at rest.
anet > g sin θ
f2 = F = 8 N,
Option (a) is correct and option (b) is 11. a = 0, T1 = 10 N,
incorrect.
T2 = 20 + T1 = 30 N,
7. Maximum value of friction.
f1 = between A and B T3 = 20 N.
= 0.25 × 3 × 10 = 7.5 N 12. fmax = 0.3 × 2 × 10 = 6 N
f2 = between B and C (a) At t = 2 s, F = 2 N
= 0.25 × 7 × 10 = 17.5 N ∴ f = 2N
f3 = between C and ground (b) At t = 8 s, F = 8 N > 6 N
= 0.25 × 15 × 10 = 37.5 N ∴ f =6N
Laws of Motion | 105
6
(c) At t = 10 s, F = 10 N and f = 6 N a = = 1 m/s 2
10 − 6 6
∴ a= = 2 m/s2
2 ∴ Pseudo force on 2 kg
(d) Block will start at 6 s. After that, =2×1=2 N (backward)
net impulse 14. N = Mg − F sin θ
1
= × 4 × (6 + 10) + 2 × 10 − 6 × 6 F cos θ = µN = µ ( Mg − F sin θ)
2
µMg
= 16 N-s = mv ∴ F=
16 cos θ + µ sin θ
∴ v= = 8 m/s.
2 For F to be minimum,
13. fmax = 0.4 × 2 × 10 = 8 N dF
=0

(b) At t = 3 s, F = 6 N
∴ Common acceleration

Match the Columns


1. Acceleration after t = 4 s ∴ (a) → (s)
At t = 4 s, F = 8 N At θ = 90°
∴ Fmax = 8 Normal force on block by plane will be
i.e., µ s mg = 8 zero.
8 8 ∴ Force of friction = 0
⇒ µs = = = 0.4
mg 2 × 10 ∴ (b) → (s)
At θ = 30°
∴ (a) → (r)
Angle of repose = tan −1 µ
At t = 4 s, a = 1 ms −2
= tan −1(1) = 45°
t = 4 s, F =8N
As θ < angle of repose, the block will not
F − µ k N = ma slip and thus,
F − ma F − ma
i.e., µ k = = force of friction = mg sin θ
N mg
= 2 × 10 × sin 30° = 10 N
8 − (2 × 1)
= = 0.3 ∴ (c) → (p)
2 × 10
At θ = 60°
(b) = (q) As θ > angle of repose
At t = 01. s, F = 0 . 2 N Block will accelerate and thus force of
∴ Force of friction (at t = 01 . s) = 0.2 N friction = µ N
∴ (c) → (p)
= 1 × 2 × 10 × cos 60°
At t = 8 s, F = 16 N
= 10 N
F − µ kmg
∴ a= ∴ (d) → (p).
m
16 − (0.3 × 2 × 10) 3. All contact forces (e.g., force of friction and
= =5
2 normal reaction) are electromagnetic in
a nature.
i.e., = 0.5
10 ∴ (a) → (q), (r)
∴ (d) → (s). (b) → (q), (r).
2. At θ = 0°, dragging force = 0
∴ Force of force = 0
106 | Mechanics-1
Nuclear force is the force between 5. (a) As block A is stationary, f = 10 N
nucleons (neutrons and protons). Between T T T T
two protons field force also acts. B
T
4. (a) N R − 10 = 2 ( + 5)
C T=F
F = 10N
5 m/s2 A
+ ive f
∴ (a) → (p)
10 N (b) As block C is stationary force of friction
between C and ground will be zero.
NR ∴ (b) → (s)
(c) Normal force ( N C ) on C from ground
N C = N B + mC g
⇒ N R = 20 N = mB g + mC g
∴ (a) → (f) = ( mB + mc ) g
(b) mg ( = 20 N) > F ( = 15 N) = (1 + 1) 10 = 20 N
f ∴ (c) → (q).
(d) As block A is stationary
NR
T = F (as shown in figure)
20 N = 10 N
∴ (a) → (p).
6. If friction force ( f ) is less than the applied
F (= 15 N)
force ( F ).
Block would be slipping in the downward F
direction.
Force of friction will be in the upward Net force on body = F − f
direction. The body will be in motion and thus the
Frictional force = 20 − 15 friction will be kinetic.
=5N ∴ (a) → (q)
(b) If friction force ( f ) is equal to the force
∴ (b) → (p)
applied, the body will be at rest. If the
(c) If F = 0, the block will slip downwards
body is at the point of slipping the
due to mg ( = 20N)
force of force will be limiting too.
Limiting friction = µ s N R Emphasis is being given to the word
= 0.4 × 20 “may be” as when a body is moving and
=8N the external force is made just equal to
Minimum value of F for stopping the block the frictional force, the body would still
be moving with friction force at its
moving down = 20 − 8
limiting (kinetic) value.
= 12 N ∴ (b) → (p), (r)
∴ (c) → (s). (c) If object is moving the friction would
(d) F = mg + 8 be kinetic as explained in (a).
∴ (c) → (q).
= (2 × 10) + 8
(d) If the object is at rest, then friction
= 28 N mg fmax = 8 N may be static and limiting as
∴ (d) → (s) F explained in (b).
∴ (d) → (p), (r)
Laws of Motion | 107
7. (a) Normal force between A and B = m A g (d) Friction force on 3 kg block due to 5 kg
f1 (max.) = µ 1m A g (towards left) block = f2 (max)
= 5 N, towards left.
A ∴ (d) → (q), (s).
µ1 f1( R)
f1
8. (a) and (b)
B 3 kg F
µ2
f2
C 5 kg
T
f2(R) T
T T T
= 0.2 × 2 × 10 2 kg
=4N ° √3
30 A µ=
s in 2 B
Normal force between B and C mg 30°
3kg
= ( m A + mB ) g
f2 (max) = µ 2 ( m A + mB ) g
= 0.1 (2 + 3) × 10 3
µ=
=5N 2
Total friction force on 3 kg block 3
∴ Angle of repose = tan −1 = 40.89 °
= f1 (max) + f2 (max) = 4 + 5 2
Now as, angle of incline (30° ) < angle
= 9 N towards left
repose (= 40.89 °)
∴ (a) → (q), (s) The block A and so also B will remain
(b) Friction force on 5 kg block stationary.
= f2 ( R) = f2 (max) ∴ (a) → (r),
= 5 N, towards right (b) → (r)
(c) Tension ( T) in the string connecting
∴ (b) → (p), (s) 2 kg mass = mg sin 30°
(c) Friction force on 2 kg block due to 3 kg 1
= 2 × 10 ×
block = f1( R) 2
= f1 (max) = 10 N
∴ (c) → ( s)
= 4 N, towards right
(d) Friction force on 2 kg mass = zero.
∴ (c) → (p), (s) ∴ (d) → (r).
6 Work, Energy and Power

Introductory Exercise 6.1


→ → → → → →
1. Work done by F = F⋅ x 2. Work done by F = F ⋅ l

N
→ → →
→ T T → T
F N β →
l
→ → → →
W → |F| = F, |N| = N
–x → → →
F → |W| = W, |T| = T
→ → → α W →
|F|= F ,|N|= N, |W |= W |l| = l
→ → → →
|T|= T and |x|= x = |F||
⋅ l | cos π
→ →
= |F ||x| cos π =−Fl
→ → →
=−Fx Work done by N = N ⋅ l
→ → →
Work done by |N|= N ⋅ x → → π
= |N||
⋅ l | cos =0
→ → π 2
= |N||x| cos → → →
2 Work done by |W |= W ⋅ l
=0 → → π
→ → → = |W || l | cos  + α
Work done by W = W ⋅ x 2 
→ → π = − W l sin α
= |W ||
⋅ x| cos
2 → → →
Work done by |T|= T ⋅ l
=0
→ → → → →
Work done by T = T ⋅ x = |T|| l |cos β
→ → = T l cos β
= |T||x|cos 0
=Tx
Work, Energy and Power | 109

→ → g Substituting value of N from Eq. (ii) in
3. W − T = m Eq. (i).
4
g µ (W − F sin 45° ) = F cos 45°
→ a=4
T 1 1 1
or W − F  =F
4  2  2
F F
or W − =4
2 2
5F
or =W
2
→ 2 2
W = mg
or F= W = mg
5 5
→ →
Work done by force F = F ⋅ s

l → → 1
= |F||s |cos 45° = F s
2
 2  1 mgs
= mg s =
 5  2 5
1.8 × 10 × 2
=
→ →
|g|= g and | l |= l 5
→ = 7.2 J
→ → mg → →
∴ |T|= m g − Work done by friction = µ N ⋅ s
4
→ →
3 → = µ |N||s|cos π
= mg
4
→ → = − µ Ns
Work done by string = T ⋅ l = − F cos 45° s
→ →
= |T|| l |cos π = − 7.2 J
→ →
3 → → Work done by gravity = g ⋅ s
= m g l cos π
4  → → π
= |g||
⋅ s | cos
3 2
= − mgl
4 =0
4. µ N = F cos 45° …(i) 5. F = mg sin 45°
F sin 45° F F 2ms–1
N 45°
F
F cos 45°

µN mg sin 45°
s
W mg cos 45°

→ → 45°
N = |N|, W = |W |
1
→ → → = 1 × 10 ×
s = |s |, F = |F|, g = |g| 2
N + F sin 45° = W …(ii) =5 2N
Displacement of lift in 1s = 2 m
110 | Mechanics-1
→ →
Work done by force of friction ( F ) = F ⋅ s 7. Work done = Area under the curve
→ → F(N)
= |F||s |cos 45°
1 10
= F s cos 45° = 5 2 ⋅ 2 ⋅ = 10 Nm
2
–4 –2 A2 A3 A4
6. Total work-done by spring on both masses x (m)
A1 2 4
m m

x0 – 10
m m

= A1 + A2 + A3 + A4
= PE of the spring when stretched by 2 x0
1 − 2 × − 10  − 2 × 10
= k (2x0 )2 = 
2  2  +  2  + [10 × 2]
= 2 k x20 2 × 10
+ 
∴ Work done by spring on each mass  2 
2 k x20 = 30 Nm
= = k x20
2

Introductory Exercise 6.2


−1
−20 ms
1. a = = − 10 ms −2 B
2s fP P F
−2
∴ F = ma = 2 kg × − 10 ms = − 20 N
Q a
s = Area under curve
1
= × 25 × 20 ms −1 According to B (non-inertial frame)
2
work done
= 20 m
= Work done by F + Work done by f p
∴ Work done = F s = ( −20 N) (20 m )
(pseudo force)
= − 400 Nm
= ( mas) + ( − mas) = 0
2. According to A (inertial frame)
As P is at rest , ∆K = 0
P m
A ∴ ∆K = W (Work -Energy theorem)
Q a Note In inertial frames one has to also to consider
work-done due to pseudo forces, while applying
Work-energy theorem.
Acceleration of P = a
3. v=α x
∴ Force on P = ma dv 1 dx
Work done over s displacement = mas ∴ a= =α
dt 2 x dt
Now, v2 = u2 + 2as 1 α2
=α α x=
= 2as (Qu = 0) 2 x 2
1 2 1 α 2
∴ Gain in KE = mv = m 2 as = mas F = ma = m
2 2 2
∴ ∆K = W (Work -Energy theorem) mα2
∴ W = b
2
Work, Energy and Power | 111
2
4. ∆K = Work done by F 1 x
or − mv20 = − A
+ Work done by gravity 2 2
m
F = 80 N ⇒ x = v0
A
7. (a) If T = mg, the block will not get
4m
g accelerated to gain KE. The value of T
must be greater that Mg.
F

5g
= 80 ⋅ 4 ⋅ cos 0 + 5 g ⋅ 4 ⋅ cos π
= 320 + ( − 200)
T F
or K f − K i = 120 J
T T
or K f = 120 J (as K i = 0)
5. Change in KE = Work done
ma = mg Hand
R(1 – cosθ) mg
R sin θ mg Mg
θ
R ∴ Ans. False
mg (b) As some negative work will be done by
O
Mg, the work done by T will be more
that 40 J.
∴ Ans. False
1 (c) Pulling force F will always be equal to
mv2 = mgR (1 − cos θ) + mgR sin θ T, as T is there only because of pulling.
2
⇒ v = 2 gR (1 − cos θ + sin θ) ∴ Ans. True
6. ∆K = W (d) Work done by gravity will be negative
1 x Ans. False
or 0− mv20 = ∫ 0 − Ax dx
2
Introductory Exercise 6.3
1. In Fig. 1 Spring is having its natural length.
In Fig. 2 A is released. A goes down byx .
Spring get extended by x. Decrease in PE
of A is stored in spring as its PE.
T T 1
∴ mAg x = k x2
T T 2
A
A Now, for the block B to just leave contact
x
with ground
mAg
T
kx = mg
T i.e., 2m A g = mg
B m B m m
Ground Ground ⇒ mA =
2
mg
Fig. 1 Fig. 2
112 | Mechanics-1
l 1 1
2. Decrease in PE = mg or mv2C = × 500 × 00924
.
2 2 2
1 l 500
∴ mv2 = mg or vc = × 0.0924 = 2.15 m s −1
2 2 100
m
i.e., v= gl 4. Work done by man = gh + Mgh
2
3. OA = 50 cm m
C 20 cm B 40 cm =  + M  gh
2 
A
5. When block of man M goes down by x, the
30 cm
spring gets extended by x. Decrease in PE
of man M is stored in spring as its PE.

∴ Extension in spring (when collar is at A)


= 50 cm − 10 cm = 0.4 m m m
Extension in spring (collar is at B)
37°
= 30 cm − 10 cm
= 20 cm = 0.2 m T
T
KE of collar at B T
= PE of spring − PE of spring T
T
(collar at A) (collar at B)
mg sin 37° T
1 x
× K × [(0.4)2 − (0.2)2 ]
=
2 µmg cos 37°
1 1 37°
or mv2B = × 500 × 12 mg
m 2
500 × 012
. 1 2
or vB = = 2.45 s −1 ∴ Mgx =
kx
10 2
Extension in spring (collar arrives at C) or kx = 2Mg
= [ (30)2 + (20)2 − 10] cm = 0.26 m For the block of man m to just slide
k x = mg sin 37 ° + µ mg cos 37 °
KE of collar at C 3 3 4
or 2Mg = mg + mg
= PE of spring − PE of spring 5 4 5
(Collar at A) (Collar at C) 3
or M= m
1 5
= × 500 × [(0.4)2 − (026
. )2 ]
2

Introductory Exercise 6.4


1. Velocity at time t = 2 s F
2. Velocity at time = a t = ⋅t
v = g t = 10 × 2 = 20 ms −1 m
Ft
Power = Force × velocity ∴ vav = (acceleration being constant)
2m
= mgv = 1 × 10 × 20 = 200 W F2 t
Pav = F × vav =
2m
Work, Energy and Power | 113
Instantaneous power As at t = 0, KE = 0, c = 0
= Force × instantaneous velocity ∴ KE = t2
Ft F 2 t 1 2
=F⋅ = i.e., mv2 = t2 or v = t
m m 2 m
Energy 2.0 + 2 t
3. Power = Pav = =t
Time 2
KE 5. U = − 20 + ( x − 2)2
P= ⇒ KE = P ⋅ t
t PE( = U) is minimum at x = 2.
1
∴ mv2 = Pt ∴ Equilibrium position is at x = 2 m
2
dU
2 Pt = 2 ( x − 2)
or v= dx
m d2U
ds 2 P 1/ 2 =2
or = ⋅t dx2
dt m
∴ Equilibrium is stable.
2P 1/ 2
or ∫ ds = m ∫ t dt 6. F = x − 4
For equilibrium, F = 0
2 P t 3/ 2
or s= +c i.e., x− 4=0
m 3/2
i.e., x=4m
At t = 0, s = 0, ∴ c = 0 dU
As, F = −
8 P 3/ 2 dx
Thus, s= t
9m dU
= − ( x − 4)
4. P = 2 t dx
KE = ∫ P dt = ∫ 2t dt d2U
∴ = −1
= t2 + c dx2
Thus, equilibrium is unstable.

AIEEE Corner
Subjective Questions (Level I)
(a) Work done by a constant force 3. m1 g − T = m1a …(i)
1. (a) Work done by a constant force T − m2 g = m2 a …(ii)
Work done by applied force = F s cos 0 Solving Eq. (i) and Eq. (ii),
= 40 × 2 = 80 Nm m − m2
a= 1 g
Work done by force of gravity = mgs cos π m1 + m2
= 2 × 10 × 2 × − 1 = − 40 Nm 4−1
= g
→ → → 4+1
2. r21 = r2 − r1
Q m1 = 4 kg 
^
= (2 ^i + 3 ^j − 4 k) − (1 ^i + 4 ^j + 6 k)
^  
and m2 = 1 kg T T
^ ^ ^ 3 a
= i − j − 10 k = g T
→ →
5 a
Work done = F ⋅ r21 1 T
s = ut + at2
^ ^ 2
= (6 ^i − 2 ^j + k) ⋅ ( ^i − ^j − 10 k) m2g
m1g
= 6 + 2 − 10 = − 2 Nm
114 | Mechanics-1
1 3  2 = 2 × 10 × 2 sin 60°
=  g 2
2 5  = 20 3 Nm
= 12 m = 34.6 Nm
Work done by gravity on 4 kg block Work done by force of friction
= 4 g × 12 cos 0 = f s cos π
= 480 Nm = µ Ns cos π
Solving Eq. (i) and Eq. (ii), = − µ ( mg cos θ) s
2 m1m2 1
T= g = − (2 × 10 × cos 60° ) × 2
m1 + m2 2
4 = − 10 Nm
=2× g
5 (b) Work done by a variable force.
= 16 N 6. W = ∫ F dx
Work done by string on 1 kg block x =− 4
=∫ − 2x dx
= 16 × 12 cos 0 x=2
x= −4
= 192 Nm  x2 
= − 2 
4. Work done by applied force = F s cos 45°  2 x = 2
N
F = 16 N = − [( −4)2 − (2)2 ]
F
= − 12 Nm
45°
7. W = ∫ F dx
x=2 4
=∫ dx
s = 2.2 m x = 4 x2
x=2
mg = ∫x =4 4x −2 dx
x=2
1  x− 2 + 1 
= 16 × 2 ⋅ 2 × = 4 
2  − 2 + 1 x = 4
= 24.9 Nm 1
x=2
= − 4 
Work done by normal force  x  x = 4
= N s cos 90° = 0 1 1
Work done by force of gravity = − 4  −  = − 1 Nm
2 4 
= mgs cos 90° = 0
(c) Work done by area under F- x graph
Total work done on the block
= 24.9 + 0 + 0 8. (a) W = 3 × (5 − 10) = − 15 Nm

= 24.9 Nm Fx(N)

5. Work done by gravity = mgs sin θ 3


N µ N
f=

s x(m)

mg 2 4 6 8 10 12 14 16

(b) W = 3 × (1 0 − 5) = + 15 Nm
θ = 60 °
Work, Energy and Power | 115
3 × (12 − 10) ∴ minimum PE = ( 4 − 4) − 16 2
(c) W = = 3 Nm
2 = − 16J
(10 − 4) + (12 − 0)
(d) W = ×3 Now, (KE) max + (PE) min = Total
2
mechanical energy
= + 27 Nm
(3 − 2) + (3 − 0) or (KE) max + ( − 16) = − 4
9. (a) W = ×2
2 or (KE) max = 12 J
Fx(N) (c) PE max = KE max = 12 J

PEmax PEmax
2 x
O x=4
KEmax

x(m) U = ( x − 4)2 − 16
O
or 12 = ( x − 4)2 − 16
1 2 3 4 5 6 7
or x2 − 8 x − 12 = 0
–1
⇒ x= 4+2 3
= 4 Nm and = 4−2 3
(b) W = 0 Nm (d) U = ( x − 4)2 − 16
1 dU
(c) W = (6 − 4) ( − 1 − 0) = − 1 Nm = 2 ( x − 4)
2 dx
dU
(d) W = 4 Nm + 0 Nm + ( − 1) Nm ∴ Fx = − = − 2 ( x − 4)
dx
= 3 Nm.
or Fx = 8 − 2x
Conservative force field and Potential
Energy. (e) Fx = 0
d i.e., 8 − 2x = 0
10. F=− U
dr or x=4m
d
=− Ar−1 Kinetic energy and Work-energy theorem
dr
A p2
= ( −) ( − A ) r − 1 − 1 = 2 12. K =
r 2m
2
11. U = ( x − 4)2 − 16  p + p
 2
∴ PE (at x = 6.0 m) ∴ K′ =
2m
= (6 − 4)2 − 16 = − 12 J
(K ′ is the KE when momentum p is
KE (at x = 6.0 m) = 8 J increased by 50%)
(a) ∴ Total mechanical energy 9 p2
or K′ =
= ( − 12) + (8) = − 4 J 4 2m
(b) KE will be maximum where, PE, is 9
minimum. or K′ = K
4
For U to be minimum, 9 5
dU or K′ − K = K − K = K
=0 4 4
dx K′ − K 5
d ∴ =
i.e., [( x − 4)2 − 16] = 0 K 4
dx
5
or 2 ( x − 4) = 0 = × 100% = 125 %
4
or x=4m
116 | Mechanics-1
13. p = (2 mK )1/ 2 102
16. a = = 2.5 ms −2 F (Push of air)
∴ p′ = [2m ( K + 1% of K ) ]1/ 2 2 × 20
( p′ is the momentum when KE i.e., K is Now, mg − F = ma a
increased by 1%) ∴ F = m ( g − a)
1
= [2 mK (1 + 1%) ]2 = 5 (10 − 2.5) mg

1 
1/ 2 = 37.5 N
= (2 mK )1/ 2 1 +  Work done by push of air
 100
1 
1/ 2 = F s cos π
i.e., p′ = p 1 +  = − (37.5 × 20)
 100
1 1  = − 750 Nm
= p 1 + × 
 2 100 17. (a) W = ∫ F dx
1 2
= p 1 + %  = ∫ (2.5 − x2 ) dx
 2  0
1 2
p′ = p + % of p  x3 
2 = 2.5x − 
 2 0
∴ Increase in momentum = 0.5 %.
23
14. s = (2t2 − 2t + 10) m = (2.5 × 2) −
3
ds
= 4t − 2 = 2.33 Nm
dt
d2 s ∴ ∆K = 2.33 J (1 Nm = 1 J)
= 4 ms −2
dt2 i.e., KE (at x = 2) − KE (at x = 0) = 2.33 J
d2 s ∴ KE (at x = 2) = 2.33 J
F=m 2
dt (b) Position of maximum KE
= 2 × 4= 8 N F = 2.5 − x2
s (at t = 0 s) = 10 m ∴ F decreases as x increases and F is zero
s (at t = 2 s) = 2.22 − 2.2 + 10 = 14 m when x = 2.5 m
∴ ∆s = 14 m − 10 m = 4 m Thus, work will be +ive from x = 0 to
Work = F ∆s x = 2.5 m an so KE will be maximum at
x = 2.5 m.
=8N× 4m
KE (at x = 2.5 m)
= 32 Nm 2.5
v2 (0.4)2 0.16 = ∫0 (2.5 − x2 ) dx
15. a = = = = 0.04 m s −2
2s 2 × 2 4 2.5
mg − T T  x3 
=a = 2.5x − 
m  3 0
a
i.e., T = m( g − a)  2.5 2.5 
= 2.5 2.5 − 
= 30 (10 − 0.04)  3 
= 298.8 N 2
= × 2.5 2.5
Work done by chain = T s cos π 3
= − (298.8 × 2) Nm = 2.635 J
= − 597.6 Nm
Work, Energy and Power | 117
F − mg Adding Eq. (i) and Eq. (ii),
18. a =
m m1 g = ( m1 + m2 ) a
g F m1
or = −g ∴ a= g
10 m m1 + m2
11
or F= mg 1
10 = g (Q m1 = 1 kg and m2 = 4 kg)
1+ 4
T
g
= = 2 ms −2
F
5
Now, v2 = u2 + 2as
F
i.e., v2 = 2as (Qu = 0 ms −1)
or v = 2as
mg = 2×2×1
= 2 ms −1
(a) ∴ Work done on astronaut by F 50
20. T − 50 = a
11mg 10
= × 15
10
11 T
= × 72 × 10 × 15
10
= 11642.4 Nm T T T
T
(b) Work done on astronaut by
gravitational force T
= mghcos π T' B a
= − 72 × 9.8 × 15 = − 10584 Nm a T' 50 (N)
(c) Net work done on astronaut
A
= (11642.4) + ( − 10584) = 1058.4 Nm
∴ KE = 1058.4 J 300 (N)
1
(d) mv2 = 1058.4 or T − 50 = 5a …(i)
2
1058.4 × 2 and T ′ = 2T
∴ v= = 5.42 ms −1 T′
72 i.e., T= …(ii)
2
19. T = m2 a …(i)
and m1 g − T = m1a …(ii) From Eq. (i) and Eq. (ii),
a T′
− 50 = 5a
T T 2
m2
or T ′ − 100 = 10a …(iii)
T 300
Also, 300 − T ′ = a
10
T i.e., 300 − T ′ = 30a …(iv)
a
Solving Eq. (iii) and Eq. (iv),
m1g a = 5 ms −2
1m T ′ = 100 + 10a = 150 N
118 | Mechanics-1
1 1
21. NR cos (180° − α ) + mgR = mv2s 1 × 10 × 1 = µ k × 4 × 10 × 2 + × 1 × (0.3)2
2 2
1 or 10 = 80 µ k + 0.045
( mg cos α) R ( − cos α) + mgR = mv2s
2 10 − 0.045
1 2 2
or µk = = 0.124
or mvs = mgR (1 − cos α) 80
2
24. f = force of friction while disc in slipping
over inclined surface = µ mg cos 30°
N N
α f
s(
say
R )
α s/2 Stops
f'
mg 30°
2R
0.50 m
α f ′ = force of friction while disc is slipping
over plane surface = µ mg
or v2s= 2 gR sin α 2
Now, decreases in PE of disc = Work done
or vs = 2 gR sin α against frictional force
(vs is the speed with which sphere hits s
mg = fs + f ′ (0.5)
ground) 2
1 1 s
mv2w = mgR − mv2s or mg = (µ mg cos 30° ) s + µ mg (0.5)
2 2 2
1
= mgR − m 2 gR sin2 α or mgs (0.5 − µ cos 30° ) = µ mg (0.5)
2 µ × 0.5
= mgR (1 − sin2 α) = mgR cos2 α ⇒ s=
0.5 − µ cos 30°
∴ vw = 2 gR cos α 0.15 × 0.5
= = 0.2027 m
(vw is the speed of wedge when the sphere 3
hits ground) 0.5 − 0.15 ×
2
22. For 45 kg mass to drop 12 mm, the Work performed by frictional forces over
increase in length of the spring will be the whole distance
24 mm. s 50 0.2027
= − mg = − × 10 ×
Now, decrease in PE of 45 kg mass 2 1000 2
= Increase in KE of 45 kg mass + Increase = − 0.051 J
in PE of spring 25. m A g sin θ − µ m A g cos θ − 2 T = m A a
1 1
45 × 9.8 × 12 × 10−3 = × 45 × v2 + × 1050 T
2 2
× [(75 + 24)2 − 752 ] × 10−6 T
T
i.e., 5.292 = 22.5 v2 + 2.192 T
or 22.5 v2 = 3.0996 T
a 2T
or v2 = 0.13776 2T T
a
v = 0.371 ms −1
θ

or µmAg cos θ B
s

A
co
Ag

(b) With friction (when mechanical mBg


m

S θ
energy does not remain conserved) 3
4
23. Decrease in PE of 1 kg mass
3 4
= Work done against friction due to 4 kg or 300 × − 0.2 × 300 × − 2 T = 30a
mass + Increase in KE of 1 kg mass 5 5
Work, Energy and Power | 119
1 1
or 180 − 48 − 2T = 30a m2 g × 4 = µ m1 g × 4 + m1v2 + m2v22
or 132 − 2T = 30a …(i) 2 2
1
Also, T − mB g = mB a 5 × 10 × 4 = 0.2 × 10 × 10 × 4 + (10 + 5) v2
2
or T − 50 = 5a
Solving, v = 4 ms −1
or 2T − 100 = 10a …(ii)
Three types of Equilibrium
Adding Eq. (i) and Eq. (ii), dU
28. (a) F = −
32 = 40a dr
or a = 0.8 ms −2 Point dU F
Speed (v) of block A after it moves 1 m dr
down the plane A + ive − ive
v2 = 2as B + ive − ive
2
or v = 2 × 0.8 × 1 C − ive + ive
or v = 1.12 ms −1 D − ive + ive
26. Work done by frictional force acting on E zero zero
block
(b) x = 2 m point is of unstable equilibrium
= − µ mgs (U being + ive)
= − 0.25 × 3.5 × 9.8 × 7.8 x = 6 m point is of stable equilibrium
= − 66.88 J (U being lowest − ive)
∴ Increase in thermal energy of block-floor x3
system 29. U = − 4x + 6
3
= 66.88 J For U to be maximum (for unstable
As the block stopped after traversing equilibrium) and minimum (for stable
7.8 m on rough floor the maximum kinetic equilibrium)
energy of the block would be 66.88 J (just dU
before entering the rough surface). =0
dx
Maximum PE of spring d  x3 
i.e.,  − 4 x + 6 = 0
= Maximum KE of block dx  3 
1 2
kxmax = 66.88 or x2 − 4 = 0
2
2 × 66 . 88 or x= ±2
∴ xmax = m d2U d 4
640 = ( x − 4) = 2x
dx2 dx
Maximum compression in the spring
At x = + 2 m,
= 0.457 m
d2U
27. Decrease in PE of mass m2 = Work done = 2 × ( + 2) = + 4
dx2
against friction by mass m1 + Increase KE
of mass m1 + Increase in KE of mass m2 ∴ U is minium.
10 kg m1
At x = − 2 m,
d2U
= 2 × ( − 2) = − 4
dx2
∴ U is maximum.
m2 = 5 kg ∴ x = + 2 m point of stable equilibrium.
x = − 2 m point of unstable equilibrium.
120 | Mechanics-1
dU Thus, the equilibrium of the charge − q
30. F = −
dx is unstable if it is slightly displaced along
i.e., U = − ∫ F dx x-axis.
= − (Area under F - x graph) (b) If charge − q is displaced slightly along
Y axis, the net force on it will be along
The corresponding U Vs x graph will be as
origin O and the particle will return to
shown in figure
its original position. And as such the
equilibrium of the − q is stable.
B
–q
F F' F'
+q +q
Fnet
B A
A C E O
(– a, 0, 0)
x
32. (a) Velocity at t = 0 s is 0 ms −1
U Velocity at t = 2 s is 8 ms −1 (using
D
+ v = 0 + at)
– x ∴vav = 4 m s −1 (as acceleration is constant)
Pav = F × vav
= ma vav
Thus, point C corresponds to stable
equilibrium and points A and E = 1 × 4 × 4 = 16 W
correspond to unstable equilibrium. (b) Velocity at t = 4 s is 16 ms −1 (using
31. − q charge placed at origin is in v = u + at)
equilibrium as is two equal and opposite ∴ Instantaneous power of the net force at
forces act on it. t = 4 s will be
y P = mav
–q
= 1 × 4 × 16
+q +q
x
= 64 W
B F F A
(– a, 0, 0) (+ a, 0, 0) 33. Power = Fmin vmax
vmax
(a) But, if we displace it slightly say
towards +ive x side, the force on it due Fmax Fmin = (r)
to charge to B will decreases while that r r Finally
Initially
due to A will increase.
–q +q
P = r vmax
O F2 F1 > F2
P
∴ vmax =
Due to net force on − q towards right the r
change − q will never come back to original
O, its origin position.

Objective Questions (Level 1)


Single Correct Option
1
1. In KE = mv2 Below reference level (PE = 0) PE is − ive.
2 ∴ Mechanical energy which is sum of KE
m is always +ive and v2 is +ive.
and PE may be − ive.
(even if v is − ive).
∴ Correct option is (a).
∴ KE is always + ive.
Work, Energy and Power | 121
2. Yes, this is work-energy theorem. 80000 + 160000
=
∴ Correct option is (a). 60
3. On a body placed on a rough surface if an = 4000 W
external force is applied the static body Option (c) is correct.
does not move then the work done by t3
frictional force will be zero. 9. x =
→ →
3
4. W = F ⋅ r21 dx
→ → →
∴ v= = t2
= F ⋅ ( r2 − r1) dt
dv
^ ^ ^
= ( ^i + 2 ^j + 3 k) ⋅ [( ^i − ^j + 2 k) − ( ^i + ^j + k)] and a= = 2t
dt
^ ^
= ( ^i + 2 ^j + 3 k) ⋅ ( − 2 ^j + k) F = ma = 2 × 2 t = 4 t
= − 4+3 W = F v = 4 t × t2 = 4 t 3
= −1J ∴ Work done by force in first two seconds
t =2 3
∴ Correct option is (b). = ∫t = 0 4 ⋅ t dt
5. ∴ W = ∫ F dx 2
5  t4 
2 = 4⋅ 
= ∫ 0 7 − 2x + 3 x dx  4 0
= [7 x − x2 + x 3 ] 50 = 135 J = 16 J
∴ Correct option is (d). ∴ Correct option is (c).
→ →
6. P = F ⋅ v 10. Range = 4 × height
u2 sin 2θ u2 sin2 θ
^ ^ ^ ^
= (10 i + 10 j + 20 k) ⋅ (5 i − 3 j + 6 k) ^ ^ or = 4⋅
g 2g
= 50 − 30 + 120 or sin 2θ = 2 sin2 θ
= 140 W or 2 sin θ cos θ − 2 sin2 θ = 0
∴ Correct option is (c). or sin θ (cos θ − sin θ) = 0
7. Work done in displacing the body or cos θ − sin θ = 0 (as sin θ ≠ 0)
= Area under the curve or tan θ = 1
1 × 10 i.e., θ = 45°
= (1 × 10) + (1 × 5) + (1 × − 5) +  
 2  Now, K = KE at highest point
1
= 15 J = m (u cos θ)2
∴ Correct option is (b). 2
1
1
mgh + mv2 = mu2 cos2 θ
W 2 2
8. P = = 1
t t = mu2 cos2 45°
800 kg × 10 ms −2 × 10 m + 2
1 1
1 = mu2 ⋅
× 800 kg × (20 ms −1)2 2 2
= 2 1 2
1 min ∴ mu = 2 K
2
(800 × 10 × 10) + [ 400 × (20)2 ]
= i.e., Initial KE = 2K
60
∴ Correct option is (b).
122 | Mechanics-1
11. P = 3 t2 − 2t + 1 15. x = 2.0 m to 3.5 m
dW dU 6
i.e., = 3 t2 − 2t + 1 =− =−4
dt dx 1.5
dW = (3 t2 − 2 t + 1) dt ∴ F =+4N
4 2
W = ∫ 2 (3 t − 2t + 1) dt x = 3.5 m to 4.5 m
dU 2
4 = =2
 t 3 2t2  dx 1
= 3 ⋅ − + t
 3 2 2 ∴ F = −2N
or = [ t 3 − t2 + t ] 42 x = 4.5 m to 5.0 m
dU
= ( 4 3 − 42 + 4) − (23 − 22 + 2) =0
dx
= 52 − 6
∴ F =0N
= 46
Work done = ( 4 × 1.5) + ( − 2 × 1) + 0
∆K = 46 J
=4J
Correct option is (b). 1 2
1 ∴ mv = 4
12. K i = × 10 × 102 = 500 J 2
2 1
or × 1 × v2 = 4
Work done by retarding force, 2
30
W = ∫ 20 − 0.1 x dx ∴ v = 2 2 ms −1
30 16. KE at highest point
 x2  1
= − 0.1   = m (u cos 45° )2
 2 20 2
2
= − 0.05 × [(30)2 − (20)2 ] 1
= mu2  
1
= − 25 J 2  2
1 1
Final kinetic energy = K i + W =  mu2 
= 500 + ( − 25) 2 2 
E
= 475 J =
2
∴ Correct option is (a).
∴ Correct option is (a).
13. KE of 12 kg mass : KE of 6 kg mass
1 1 17. Work done by person
=m12v2 : m6v2
2 2 = − [Work done by gravitational pull on rope +
gravitational pull on bucket]
[Acceleration being same (equal to g) both
will have same velocities]  h 
=  − mg  + ( − Mgh)
  2  
= m12 : m6
m
= 12 : 6 =  M +  gh
 2
= 2 :1
1 ∴ Correct option is (a).
14. W = k [ x2 − x12 ]
2
1
2 18. mv2 = Fx …(i)
1  10  2  5  2  2
= × 5 × 103   −  1
2    100  mv ′2 = Fx ′ …(ii)
 100  2
5 × 103 x ′ v ′2 (2 v)2
= (100 − 25) = 18.75 Nm ∴ = 2 = 2 = 4 ⇒ x′ = 4 x
2 × 104 x v v
∴ Correct option is (c). ∴ Correct option is (b).
Work, Energy and Power | 123
19. Vertical velocity (initial) = v0 sin θ K
At an altitude h φt02
(vertical velocity)2 = (v0 sin θ)2 + 2 ( − g) h
(horizontal velocity)2 = (v0 cos θ)2
(Net velocity)2 = v20 − 2 gh
Net velocity = v20 − 2 gh O t0 t

20. Maximum power will be at 2 s Fig. 1

velocity (at t = 2 second) = a ⋅ 2 While ball goes up after elastic


F collision with the surface it strikes
= ⋅2
m
2F 2F 2 a = + g
∴ Power (at t = 2 s) = F × =
m m
∴ Correct option is (d). H
21. v = 0 + at
F u = gt
–0
= at = ⋅t
m
Instantaneous power, Using, v = u + at
P = Fv v = ( − gt0 ) + ( + g) ( t − t0 )
F
=F ⋅t v = − g ( t − 2t0 )
m 1
∴ KE = mg2 ( t − 2t0 )2
F3 2
or P= ⋅t
m i.e., K = φ ( t − 2t0 )2
or P = constant t. K
∴ Correct option is (b).
φt02
22. While ball comes down
u=0

+
a=+g
H
O t0 2t0 t
Fig. 2

From the expression for KE


v = gt0
at t = 2t0 , K = 0
at t = t 0 when ball strikes the surface
as shown in Fig. 2.
Using v = u + at
∴ Correct option is (b).
v = 0 + gt
23. ρ (block) = 3000 kgm −3 and σ (water)
i.e., v = gt. = 1000 kgm −3
1 1
∴ KE = mv2 = mg2 t2
2 2
vσg F
K = φ t2
1
where, φ = mg2 = constant.
2
vρg (ρ − )gσ
124 | Mechanics-1
σ 1 3 y + αx = 5
i.e., =
ρ 3 y x
+ =1
(External force applied to move the block 5/3 5/α
upward with constant velocity). From above figure
Work done = F s 15 3
tan θ =
=
= v (ρ − σ) gs 20 4
Work done will be zero, if
= v (ρ − σ) × 10 × 3
5 φ=θ
= (ρ − σ) × 10 × 3 (Qvρ = 5)
ρ i.e., tan φ = tan θ
5/α 3
 σ or =
= 5 1 −  × 10 × 3 5/3 4
 ρ
3 3
1 or =
= 5 1 −  × 10 × 3 = 100 J α 4
 3
⇒ α=4
∴ Correct option is (a). ∴ Correct option is (d).
Net work done
24. pav = 27. Let x be the elongation in the spring.
Time of flight ( T)
Increase in PE of spring = Decrease in PE
mgH cos π + mgH cos 0
= of block
T 1 2
∴ k x = mgx sin θ
H = Maximum height attained by the 2
projectile 2mg sin θ
⇒ x=
(0) k
= = zero
T ∴ Correct option is (a).
1
25. W ( a) = 2 k′ ( x)2 28.

s = 2 t2 ^i − 5 ^j
2
→
l
∴ ds = 4 t ^i dt
→ →
Force Constant
2k' k' W = ∫ F ⋅ ds
l/3 2l/3 2 ^ ^ ^
(a) (b)
= ∫ 0 (3 t i + 5 j) ⋅ 4 t i dt
2
1 2
W ( b) = k′ ( x)2 = ∫ 0 12 t dt
2 2
∴ W ( a) > W ( b)  t3 
= 12 
∴ Correct option is (c).  3 0
26. Y = [ 4 t 3 ] 20
Given = 32 J
Straight line along which
φ work is zero. ∴ Correct option is (b).

s 29. W = mgh + mgd
5/3 →
F = 20i + 15j = mg ( h + d)
or Fav d = mg ( h + d)
θ  h
5 X
∴ Fav = mg 1 + 
 d
α
Work, Energy and Power | 125
1
30. Energy stored in A = E = kA x2A 34. Decrease in PE of mass m
2
kA kB
d
A B
2kA = 2kB
F F k (spring) = 100 N/m
F F θ
A B
2
1 F = Increase in PE of spring
= kA   1
2  kA  i.e., mgd sin θ = kx2
2
2
F kx2
= or d=
2kA 2mg sin θ
F2 100 × 22
Energy stored in B = = = 4 m.
2kB 2 × 10 × 10 × sin 30°
F2
= (Q kB = 2 kA ) ∴ Correct option is (c).
4 kA
35. PE of block will change into its KE and
1 F2 E then the KE gained by the block will
= =
2 2kA 2 change into the PE of the spring.
∴ Correct option is (a). Due to inertia the spring will not start
1 2 1 compressing the moment the block just
31. kx = mv2 touches the spring and as such the block
2 2
will still be in the process as increasing its
m
x=v KE. Thus v of the block will be maximum
k when it compresses the spring by some
0.5 amount.
= 1.5 = 0.15 m
50 ∴ Correct option is (b).
∴ Correct option is (a). 36. Work done by normal force is zero, being
1 perpendicular to the displacement.
32. mv2 = F (2x) …(i)
2 Work done by string is + mgh while that
1
m (2 v)2 = F ( nx) …(ii) due to gravity it is − mgh.
2
Net work done = ( + mgh) + ( − mgh)
Dividing Eq. (i) by Eq. (ii),
=0
(2 v)2 n
= ∴ Correct option is (c).
v2 2
37. Work done on floor = 0 (as displacement is
⇒ n=8
zero).
∴ Correct option is (d). (0 − 20) ms −1
1 1 38. Acceleration =
33. mgh = mv2 + kx2 (10 − 0) s
2 2
2 k 2 = − 2 ms −2
i.e., v = 2 gh − h
m Therefore, net force on particle
10 = 2 kg × − 2 m s −2
= 2 × 10 × 0.15 − × (0.15)2
0.1 =−4N
or v = 0.866 m s −1 i.e., the net force on the particle is opposite
∴ Correct option is (b). to the direction of motion.
∴ Correct option is (a).
126 | Mechanics-1
Net work done = F × s ∴ Correct option is (c).
1
= F × (Area under v-t graph) KE just before bounce = mv2 = mgh
1 2
= − 4 ×  × 20 × 10 = − 400 J
2  KE just after bounce = 80% of mgh
Thus net work done may not be wholly When the ball attains maxmum height
due to frictional force only. after bounce
Further net force − 4 N may not be wholly Gain in PE = mgh′ = Loss of KE
due to friction only. or mgh′ = 80% of mgh
39. Height of bounce = (100 − 20) % of 10 m or h′ = 80% of h = 80 % of 10 m
=8 =8m
JEE Corner
Assertion and Reason
1. P = Fv 4. In circular motion only the work done by
For power to be constant, the velocity The centripetal force is zero. Assertion is
must also be constant. Thus, assertion is false, centripetal force acts towards centre
false. while the velocity acts tangentially.
Reason is true.
According to 2nd low of motion, net
constant force will always produce a ∴ Correct option is (d).
constant acceleration. Reason is true. 5. As the speed is increasing (slope of graph
∴ Correct option is (d). being increasing) there must be net force
in the +ive direction of displacement.
2. As displacement is opposite to force
Thus, work done to all forces will be
(reason) the work done by force will be
positive, Assertion is true and also as
negative. explained above reason besides being true
Thus, Assertion is true. Further, as reason is the correct explanation for the
is the correct explanation of the Assertion. Assertion.
∴ Correct option is (a). ∴ Correct option is (a).

3. Conservative force has nothing to do with 6. Work done by constant force F when the
kinetic energy. (If a non-conservative force body shifts from A to B.
acts on a particles, there would be loss of → →
F F
KE). Thus, assertion is false. →
θ S
Work done by conservative force decreases A B
PE (reason is true). → →
A S' S'
→ →
F → F
mg (a conservative force) S
h D
C
B
→ →
mg W AB = F ⋅ s = F s cos θ
→ →
W = mgh Similarly, W AC = F ⋅ s
If PE at A is zero. = F s′ cos (90° + θ)
The PE at B would be − mgh. = − F s′ sin θ
∴ Correct option is (d). → →
WCD = F ⋅ s = F s cos θ
Work, Energy and Power | 127
→ →
WDB = F ⋅ s cos(90° − θ) greater than 90° and that between N
(normal force on wedge) and its
= F s′ sin θ
displacement will be less than 90° as given
W AC + WCD + WDB = F s cos θ = W AB
in reason.
∴ Work done by a constant force is path N
independent. thus, Assertion is true. A
The Reason is false. Kinetic frictional
force remains constant but is a
N
non-conservative force.
B
∴ Correct option is (c).
7. It is true that work-energy theorem can be
applied to non-inertial frame also as
explained in the answer to question no. 2
of introductory exercise 6.2.
Earth is non-inertial which is also true is
a separate issue and has nothing to do For this the work done byN (or block) will
with the assertion. be negative and that by N (on wedge) will
Thus, option (b) would be the answer. be positive as given in assertion.
8. When block is depressed the excess of Reason also being the correct explanation
of the assertion.
upthrust force will act as restoring force
and will bring the block up. The velocity 11. There will be increase in length of the
elastic cord.
gained by block will take the block above
its equilibrium and the block will oscillate elastic
θ
about its equilibrium position (as given in cord
reason). Thus, the block will be in
equilibrium in the vertical direction. Block
Thus, assertion is also true and the reason
f1
being correct explanation of the assertion. Plank f1
∴ Correct option is (a). s f
f2
9. As displacement ( = s2 − s1) is not equal to Work-done by static force
zero, the work done by all forces may not
f1 (on block) = − f1s
be zero. Therefore, assertion is false.
Work done by static force
Work done by all the forces is equal to
f1 (on plank) = + f1s
change in KE is as per work-energy
theorem. Thus, reason is true. ∴ Work done by static force f1 on the
system (block + plank + cord) = 0
∴ Correct option is (d).
Thus, reason is true.
10. When the block comes down the wedge, The work done by the force F will be used
the wedge will move towards left and the up in doing work against friction f2 and
actually displacement of normal force will also increasing the elastic potential
not be along the wedge (see chapter on energy of the cord. Thus, assertion is true.
CM). It will along AB as shown in figure. Further, as the reason is the correct
Thus, the angle between N (normal force explanation of the assertion.
on block) and its displacement AB will be
128 | Mechanics-1
1 1
12. Decrease in KE = mv2 − 0 PE + KE = mv2
2 2
v Decrease in ME is used up in doing work
f Slope against friction.
Rough Case-I In this case the mechanical energy is
gh being used up in doing work against
ps E=m friction and in increasing the PE of the
Sto P
block.
1
∴ Change ME = mv2 − mgh
2
v' h
Thus, assertion is true.
30°
As explained above the reason is false.
f'
(µ does not change with the increase in
1
mv2 = angle of inclination)
2
∴ Correct option is (c).
Change in mechanical energy
Objective Questions (Level 2)
Single Correct Option
1. Increase in KE of bead = Work done by 3. Loss of PE of block = Gain in PE of spring
gravity + work done by force F
1
∴ mv2 = mgh + FR d
2
(Displacement of force F is R) k
1 A
= × 10 × 5 + 5 × 5 = 50
2
1
100 mg ( d + 2) sin 30° = k × 22
∴ u= 2
m
1 1
= 200 = 14.14 ms −1 or 10 × 10 × ( d + 2) × = × 100 × 22
2 2
∴ Correct option is (a). d = 2m
Let, v A = velocity of block when it just
2. P = Fv touches spring
= mav
 dv   dv dv ds dv  d
= m v  v  a = = ⋅ =v 
 dx   dt ds dt ds 
m 2 2m
∴ ds = v dv
p m A
m 2v 2
i.e., ∫ ds = v dv 30°
p ∫v
1
m v 3 
2v
∴ mv2A = mgd sin 30°
or s=   2
p  3 v 1
v2A = 10 × 2 × 2 ×
m 7 mv 3 2
= [(2v) 3 − (v) 3 ] =
3p 3p v A = 20 ms −1
∴ Correct option is (a). Correct option is (a).
Work, Energy and Power | 129
m ∴ a2 = 0
4. Mass per unit length of chain =
πR/2 ∴ Correct option is (b).
m
dm = R dθ 6. Let x be the expansion in the spring.
πR/2 kx kx T T
A B
∫ y dm
yCM = T
∫ dm
θ T

CM M1g

R (1 – cos θ) y (
R 1–2
π )
Increase in PE of spring = Decrease in PE
of block C
π /2 m 1 2
∫0 R (1 − cos θ) R dθ kx = M1 gx
πR/2 2
=
m i.e., kx = 2M1 g
2R π / 2 For block A to remain at rest
= (1 − cos θ) dθ
π ∫0 kx = µ min Mg
2R  π
= − 1 or 2M1 g = µ min Mg
π  2 
2M1
2 ∴ µ min =
= R 1 −  M
 π 
∴ Correct option is (c).
1
Now, mv2 + mg yCM 7. Ti = mg
2
2mg
or v2 = 2 g yCOM KX i = = mg
2
2
v = 2 gR 1 −  When one spring is cut. It means KX i
 π becomes zero. Downward acceleration,
∴ Correct option is (c). KX i mg g
a= = =
5. The moment string is cut 2m 2m 2
T Now drawing FBD of lower mass :
A mg
T = mg mg − Tf = m. a =
mg 2
mg F (= mg) mg
F ∴ Tf =
2
mg
or ∆T = Tf − Ti =
2
F = kx = mg
mg sin θ − µ mg cos θ
8. a =
F = mg
m
B = g sin θ − µ g cos θ
mg (weight) mg For v to be maximum x
dv a
=0 θ
Net force on A = 2mg (downward) dx os
mg sin θ
mgc
dv µ
∴ a1 = 2 g or v =0
Net force on B = 0 dx θ
130 | Mechanics-1
dx dv 4 µ mg
or ⋅ =0 ⇒ x=
dt dx x
dv ∴ Correct option is (c).
or =0
dt → →
12. Power delivered by man = T ⋅ v
or a=0
i.e., g sin θ = µ g cos θ = T v cos θ →
→ T →
or sin θ = µ cos θ |T|= T v
3 3 4 → m
or = x⋅ and |v|= T
5 10 5
10 13. φ = 3 x + 4 y
⇒ x= = 2.5 m y
4
3N
∴ Correct option is (d). P (6m, 8m)
dU
9. (a) Between points E and F, is − ive. 4N
dr Fnet = 5N
dU
Now, F = − , the force between E and F
dr
will be + ive i.e., repulsive.
R 6m Q x
(b) At point C the potential energy is
minimum. Thus, C is point of stable
equilibrium. ∂φ
∴ Correct option is (c). ∴ Fx = − = −3N
∂x
→ →
10. Power = F ⋅ v ∂φ
and Fy = − =−4N
→ ∂y
v
→ PR 5
u =
→ PQ 4
θ mg
5
→ ⇒ PR = × PQ = 10 m
mg 4
→ → → ∴ Work done by the conservative force on
= m g ⋅ ( u + g t) the particle
→ →
= mg u cos (90° + θ) + m g ⋅ g t = Fnet × PR = 5 N × 10 m
i.e., P = − mg u sin θ + mg2 t = 50 Nm = 50 J
→ → ∴ Correct option is (c).
[|u|= u,|g|= g]
14. Both at x = x1 and x = x2 the force acting
Therefore, the graph between P and t will
on the body is zero i.e., it is in equilibrium.
be as shown in option (c).
Now, if the body (when at x = x1) is moved
11.
towards right (i.e., x > x1) the force acting
x
on it is + ive i.e., the body will not come
2x
back and if the body (when at x = x2 ) is
moved toward rght (i.e., x > x2 ) the force
µmg
acting on it is − ive i.e., the body will
PE of spring due to its compression by x return back. Then,x = x2 is the position of
stable equilibrium.
= | Work done by frictional force when
displaced by 2x| ∴ Correct option is (b).
1 2
i.e., kx = µ mg 2x
2
Work, Energy and Power | 131
−7 −12
15. The man will stop when i.e., 6b x = 12a x
1
2a 6
x =  
x
i.e.,
 b
mg sin θ µmg cos θ
∴ Correct option is (a).
θ
19. For the rotational equilibrium of the rod
µ mg cos θ = mg sin θ
or (µ 0 x) mg cos θ = mg sin θ l kx
tan θ
or x= l/2
µ0
mg
16. Taking moment about A
l
mg⋅ = kx l
2
mg
i.e., x=
2k
1 2
∴ PE stored in the spring = kx
k1x C k2x 2
2
A B 1  mg  ( mg)2
= k  =
F = (k1 + k2)x 2  2k 8k

AC ⋅ F = ( k2 x) l ∴ Correct option is (c).


( k x) l m − m2
∴ AC = 2 20. a = 1 g
F m1 + m2
( k2 x) l Speed (v) with which mass m1 strikes the
=
( k1 + k2 ) x floor
k2  m − m2 
= l = 02 + 2 gh = 2  1  gh
k1 + k2  m1 + m2 
∴ Correct option is (d). ∴ Correct option is (a).
17. mgh + work done by the force of friction 21. F = − ax + bx2
1 dU
= mv2 ∴ − = − ax + bx2
2 dx
∴ Work done by the force of friction or dU = ( ax − bx2 ) dx
1
= mv2 − mgh or U = ∫ ( ax − bx2 ) dx
2
ax2 bx 3
1 or U= − +c
=  × 1 × 22  − (1 × 10 × 1) 2 3
2 
At x = 0, F = 0
= − 8J
∴U = 0
∴ Correct option is (c).
a b and so, c = 0
18. U = 12 − 6 ax2 bx 3
x x Thus, U= −
2 3
U = ax −12 − bx −6
dU ax2 bx 3
For stable equilibrium, =0 U = 0, when =
dx 2 3
3a
i.e., a ( − 12) x −13 − b ( −6) x −7 = 0 i.e., at x =
2b
132 | Mechanics-1
dU 1 2
= 0, when F = 0 ∴ k xmax = mg ( h + xmax ) …(i)
dx 2
i.e., − ax + bx2 = 0 ∴ From above Eq. (i),
a xmax depends upon h and also xmax
i.e., at x=
b depends upon k.
Graph between U and x will be KE of the block will be maximum when it
is just at the point of touching the plank
and at this moment there would no
compression in the spring.
Maximum KE of block = mgh
a 3a ∴ Correct option is (c).
O X
b 2b 25. Gain in KE of chain = Decrease in PE of
chain
∴ Correct option is (c).
W A Fs 1
22. = = CM (i)
WB Fs 1
2r
∴ Correct option is (c). π
1
WA mv2A
= 2 =1
WB 1 4 mv2
B
2
v2A 4
⇒ =
v2B 1
vA 2
⇒ =
vB 1
WA 1
=
WB 1
KA 1 πr/2
∴ =
KB 1 πr CM (f)

23. Ui at (1, 1) = k (1 + 1) = 2k
U f at (2, 3) = k (2 + 3) = 5k
W = U f − Ui
= 5k − 2k = 3 k When the whole chain has justcome out of
the tube.
∴ Correct option is (b).
1 2π πr
24. Gain in PE of spring = Loss of PE of block or mv2 = mg  + 
2  π 2
2 π
∴ v + 2 gr  + 
h  π 2
∴ Correct option is (b).
xmax 26. Acceleration of the block will decrease as
PE = 0 level the block moves to the right and spring
for block
expands the velocity (v) of block will be
maximum, when
1 1
mv2 = kx2
2 2
Work, Energy and Power | 133
At this moment, F = k x 29. Work done on block A in ground frame
F (50 − 30)
i.e., x= = 0.2 × 45 × 10 ×
k 100
2
k  F F2 = 18 J
or v2 =   =
m  k km ∴ Correct option is (b).
F 30. N = mg cos θ
∴ v=
mk N

∴ Correct option is (b). φ = 53° 20 m


→ →
27. Force on each block = k x s s=|s|
4ms–1 6ms–1

A B 10 ms–1
kx kx
θ = 37°
1
= 200 × = 20 N
10 = 10 × 10 × cos 37 °
Power of A = 20 × 4 = 80 W 4
= 10 × 10 ×
5
Power of B = 20 × 6 = 120 W
= 80 N
∴ Total power = 200 W
i.e., rate of energy transfer = 200 Js −1 ∴ Work done by N in 2 s
= N cos φ
∴ Correct option is (c).
= 80 × 20 × cos 53 °
28. From O to x compression in the spring
3
F kx + ive
= 80 × 20 ×
A B 5
m kx 3m
= 960 Nm
Average acceleration of A = 960 J
kx − F ∴ Correct option is (b).
aA =
2m T ′ + mg
31. =5 (given)
Average acceleration of B m
kx Upper spring Lower spring
aB = cut Initially cut
2 (3 m)
As at maximum compression of the spring
both the blocks would be having same T = T' + mg T
velocity. + ive
2 a A x = 2 aB x [using v2 = u2 + 2as] T' T'
i.e., a A = aB mg mg mg
kx − F kx
=
2m 6m
T ′ + m × 10 = 5 m
kx
or kx − F = i.e., T ′ = − 5 m
3 mg − T
2 kx a=
or F= m
3 mg − ( T ′ + mg)
3F =
i.e., x= m
2k T′
=−
∴ Correct option is (c). m
134 | Mechanics-1
( − 5 m) 33. Total ME = − 40 J
=−
m ∴ PE (max) = − 40 J
= 5 ms −2
U (x)
∴ Correct option is (b). 50 J
32. Total ME = 25 J
∴ PE (U) max = 25 J [as KE can’t be −ive.] 25
U (x)
– 10 –5 6 10 15 x (m)
50 J
PE (max) PE (max) – 35
PE (max) PE (max)
25 x (m)
– 10 –5 6 10 15 Particle can't be found in the regions
above PE (max) line.
∴ “It is not possible”. Option (d).
Particle can’t be found in the region above
PE (max) line.
∴ −10 < x < − 5 and 0 < x < 15
∴ Correct option is (a).
More than One Correct Options
→ →
1. (i) Acceleration (iii) As a ⋅ v ≠ 0, the path of the particle
U = 7 x + 24 y can’t be a circle.
∂U → →
∴ Fx = − = −7 i.e., a is not perpendicular to v.
∂x
Options (c) and (d) are incorrect.
∂U
Fy = − = − 24 →
∂y v = 3i + 4j
7
i.e., ax = − – 7i
5
24
i.e., ay = −
5
→ 7 ^ 24 ^
∴ a=− i− j m/s2
5 5

i.e., |a |= 5 ms −2 →
a
– 24j

∴ Correct option is (b).


2. U = 100 − 5x + 100x2
(ii) Velocity at t = 4 s ∂U ∂
→ → → ∴ Fx = − =− (100 − 5x + 100x2 )
v=u+at ∂x ∂x
 7 24 ^ = − [ − 5 + 200 x ]
= ( 8.6 ^i + 23.2 ^j ) + − ^i − j 4
 5 5  = 5 − 200 x
Fx 5 − 200 x
= 8.6 ^i + 23.2 ^j − 5.6 ^i − 10.2 ^j ∴ ax = =
0.1 0.1
= 3 ^i + 4 ^j = 50 − 2000 x
→ (i) At 0.05 m from origin
∴ |v|= 5 ms −1
x = + 0.05 m (first point)
∴ Correct option is (a). ax = 50 − 2000 (0.05)
= 50 − 100
Work, Energy and Power | 135
2
= − 50 m/s Work done by spring = Energy stored in
= 50 m/s 2 towards − ive x the spring
1
∴ Correct option is (a). = kx2
2
(ii) At 0.05 m from origin
∴ Correct option is (b).
x = − 0.05 m (second point)
(iii) If spring is initially its natural length
ax = 50 − 2000( − 0.05) and finally compressed.
= 50 + 100 1
Work done on (not by) the spring  = kx2 
= 150 m/s 2  2 
∴ Correct option is (c). will be stored in the spring.
Mean Position ∴ Option (c) is incorrect.
50 (iv) If spring is initially at its natural
a = 0 at x = m = 0.025 m
2000 length and finally extended.
1
(iii) For point 0.05 m from mean position Work done on (not by) the spring  = kx2 
 2 
x = (0.05 + 0.025) m
will be stored in the spring.
= 0.075 m
∴ Option is (d) is incorrect.
∴ ax ( at x = 0.075 m ) = 50 − 2000 (0.075)
4. (i) Work-Energy theorem states that Wnet
= 50 − 150 = − 100 m/s 2
(Work done by all forces conservative or
= 100 m/s 2 towards − ve x-axis. non-conservative, external or internal)
∴ Correct option is (b). = ∆ (KE)
(iv) For second point 0.05 m from mean Correct option is (d) is incorrect.
position
∴ Correct option is (c).
x = 0.025 m − 0.05 m
(ii) Work done by non-conservative forces
= − 0.025 m (i.e., all forces except conservative
∴ ax = 50 − 2000 ( − 0.025) forces) lead to decrease in KE and thus
= 100 ms −2 change in mechanical energy takes
place.
∴ Option (d) is incorrect.
∴ Correct option is (b).
3. (i) If the spring is compressed by x, elastic
1 2 (iii) Work done by a conservative force may
potential energy equal to kx gets stored be + ve, − ve or zero.
2 →
in the spring. Now, if the compressed s
90°
spring is released the energy stored in the mg
spring will be lost. When the spring mg
h h
attains to natural length.
Work done by spring = Energy stored in
the spring
1
= kx2
2 mg (a conservative force)

∴ Correct option is (a). → →


W = m g⋅h
(ii) If the spring is extended by x, energy
1 = mghcos π
stored in the spring would be kx2 . If the
2 = − mgh
extended spring is released the energy
(PE increases)
stored in the spring will be lost when the
spring attains its natural length.
136 | Mechanics-1
→ →
W = m g⋅h When F is removed, the upper disc
accelerates upwards and when it attains
= mghcos 0 the position as in figure 2, its acceleration
= + mgh reduces to zero and the velocity gained by
(PE increases) it takes it further upwards. Restoring
force on the upper plate now acts
→ → π
W = m g ⋅ h = mghcos =0 downwards and that on the lower plate
2 acts in the upward direction and would lift
(PE remains same) it (lower plate) if
Option (a) is incorrect. k (δ − l) > 3 mg
5. F is the force by hand or upper disc i.e., kδ > 3 mg + kl
or kδ > 3 mg + mg
(δ – l) 4mg
k (l + δ) or δ>
k
k (δ + l)
∴ Correct option is (d).
F
Correct option is (a) obviously being
k (δ + l)
incorrect.
N 6. At maximum extension x :
decrease in potential energy of B=
increase in spring energy
k(l + δ) 1
∴ (2m)( g)( x) = kx2
3mg 3mg 2
4 mg
kl = mg or k=
k
l
mg 7. Total work done by internal forces of a
system, which constitute action and
Artist reaction pairs, is always zero and if it is
to make not so the total work done will not zero.
it 3/2 times
of l ∴ Correct options are (b) and (c).
8. (i) Work done by conservative forces may
be + ive, − ive or zero as explained in the
answer to question no. 4
kl Uncomprssed/
3mg Unstretched ∴ Option (a) is incorrect.
spring Correct option is (b).
N = 3 mg + k ( l + δ) Correct option is (c).
2mg  (ii) In pure rolling work done by frictional
= 3 mg + k l + 
 k  force (a non-conservative force) is always
zero.
= 3 mg + kl + 2mg
∴ Correct option is (d).
= 3 mg + mg + 2mg
2mg  9. In moving from 1 to 2 work done by
= 6mg for δ =  conservative force
 k 
= U1 − U2
∴ Correct option is (b).
= ( − 20) − ( − 10)
N = kl + 3 mg
= − 10 J
= mg + 3 mg = 4mg
∴ Option (a) is incorrect.
∴ Correct option is (c).
Work, Energy and Power | 137
Option (b) is correct. Work done by Normal force ( N ) on block A
Work done by all forces W = Ns cos θ
= ( K 2 + U2 ) − ( K 1 + U1) In motion 2 → + ve as θ < 90°
= [20 + ( − 10)] − [10 + ( − 20)] In motion 1 → + ve as θ < 90°
= 20 J ∴ Correct option is (b).
∴ Correct option is (c). Work done by force of friction (f )
Option (d) is incorrect. In motion 1 → may be − ive if f is directed
10. N N upwards along the plane as shown in
2 figure 1. (Motion 1 being retarded)
1 ∴ Correct option is (c).
mg mg
In motion 1 → may be + ive if f is directed
Work done by gravity downwards along the plane if motion 1 is
In motion 2 → − ive [as θ = π] accelerated.
π ∴ Correct option is (d).
In motion 1 → zero[as θ = ]
2
∴ Correct option is (a).

Match the Columns


1. As body is displaced from x = 4 m to (d) → (p, s).
x =2m 2. W = Fs cos θ
→ ^
s = −2im (a) Work done by N will be + ive as θ < 90°
→ (a) → (p)
(a) F = 4 ^i
→ →
(b) Work done by mg will be − ive as θ = π
∴ W = F ⋅ s = 4 ^i − 2 ^i
= − 8 unit N
T
(a) → (q)
and |W |= 8 unit
mg
(a) → (s)

(b) F = 4 ^i − 4 ^j (b) → (q)
→ → ^ ^ ^
∴ W = F ⋅ s = ( 4 i − 4 j) ⋅ ( − 2 i) = − 8 unit (c) Work done by force of friction (f )
will be zero as f will be zero.
(b) → (q, s)
→ (c) → (r)
(c) F = − 4 ^i
(d) Work done by tension (T) will be + ive
→ → ^
∴ W = F ⋅ s = 8 i unit as θ < 90°
and |W|= 8 unit (d) → (p)
(c) → (p, s) 3. Y

(d) F = − 4 ^i − 4 ^j +q FB FA = FB +q
→ → A +Q B
∴ W = F⋅ s Fnet = 0
(– a, 0) (+ a, 0)

= ( − 4 ^i − 4 ^j) ⋅ ( − 2 ^i) = 8 unit


F A = force on + Q by + q placed at A
and |W |= 8 unit FB = force on + Q by + q placed at B
138 | Mechanics-1
As FA = FB charge +Q will be in 4. (a) From A to B :
equilibrium.
(a)
A FB' FA' +q
+q +Q B A kx
x

B
Mean x
Due to increase in distance between + q (at position mg
A) and+ Q C

FA′ < FA 1 2
Increase in spring PE = kx
Due to decrease in distance between + q 2
(at B ) and + Q Decrease in gravitational. PE of block
FB′ > FB = mgx
Using FA = FB , we have FB′ > FA . As there = ( kx) x = kx2
will net force on + Q which will being + Q to ∴ (a) → (q)
origin. (b) From A to B
Thus, equilibrium will be a stable one. Increase in KE of block
(b) Y = Decrease in gravitational PE of block
Fnet − Increase in spring PE
FB FA 1 2
2
+Q = kx − kx
A B
2
1 2
+q +q = kx < Decrease in gravitational PE of
2
block.
As Fnet will be along the increasing ∴ (b) → (p)
direction of Y , the charge + Q will not (c) From B to C
return to origin. 1 1
Increase in spring PE = k (2x)2 − kx2
Thus, equilibrium will be an unstable one. 2 2
3 2
∴ (b) → (q) = kx
2
(c) As explained in (b) the equilibrium will 1
Decrease in KE of block = kx2
be an unstable on. 2
∴ (c) → (q) (KE of block at C will be zero)
(d) ∴ (c) → (p).
x = y line
Fnet (d) From B to C
FA decrease in gravitational PE = mgx
A = ( kx) x
+q +q = kx2
3 2
Increase in spring PE = kx
2
∴ (d) → (p)
Work, Energy and Power | 139
m − m2 2−1 (c) Work done by string on 2 kg block
5. a = 1 g= g
m1 + m2 2+1 = Ts cos π
40
= × 0.15 × ( −1)
3
= − 20 J
T ∴ (c) → (s).
T
(d) Work done by string on 1 kg block
T
a = Ts cos 0
T a 40
m2 = × 0.15 × 1
m1 3
1g 2g =2J
∴ (d) → (q).
1
= g 6. (a) Work done by friction force ( f )
3
10 (w.r.t. ground)
= m s −2
3
2 m1m2 f
T= g
m1 + m2 f' (= f )
1×2
=2× × 10
1+2
40 = f s cos π
= N
3 =−fs
Displacement of blocks, ∴ (a) → (q).
1
s = ut + at2 (b) Work done by friction force on incline
2 (w.r.t. ground)
1 2
= at = f × 0 × cos 0
2
1 10 =0
= × × (0.3)2
2 3 [There being no displacement of incline
= 0.15 m w.r.t. ground]
(a) Work done by gravity on 2 kg block ∴ (b) → (r).
= mgs cos 0 (c) Work done by a man in lifting a bucket
= 2 × 10 × 0.15 × (1) = Ts cos 0 (T = Tension in rope)
=3J = a + ive quantity
∴ (a) → (r) θ = 0 as T and s both would be in upward
direction.
(b) Work done by gravity on 1 kg block
∴ (c) → (p).
= mgs cos π
(d) Total work done by friction force in (a)
= 1 × 10 × 0.15 × ( − 1) w.r.t. ground
= − 1.5 J = − f s+0= − f s
∴ (b) → (p). ∴ (d) → (q).
7 Circular Motion

Introductory Exercise 7.1


1. In uniform circular motion the magnitude (iii) Magnitude of net acceleration
 v2 
of acceleration  =  does not change = ( ac )2 + ( aT )2
 r
= ( 4 cms −2 )2 + (2 cms −2 )2
while its direction (being always towards
the centre of the circular path) changes. = 2 5 cms −2
→ →
2. If ω 0 and ω are in rad s −1 the value of α 5. |v1|= |v2|= v (say) →
−2 v2 Q
must be in rad s . But, if ω 0 and ω are in 2πr
degree s −1 the value of α must also be in T=
v →
v1
degree s −2 . Thus, it is not necessary to r

express all angles in radian. One way → |PQ|
change rad into degree using π rad = 180°. |v av|= P
tPQ O r
3. During motion of an object along a curved
r 2 r 2
path the speed and magnitude of its radial = =
acceleration may remain constant. Due to T / 4 πr / 2 v
change in direction of motion the velocity 2 2
= v
of the object will change even if its speed π
is constant. Further, the acceleration will →
|v av| 2 2
also change even if the speed is constant. ∴ =
v π
4. (i) Radial acceleration ( ac )
6. ω t = 0 + 4 t
v2 (2 cms −1)2
= = Centripetal acceleration
r 1 cm
= tangential acceleration
= 4 cms −2
r ω2t = r α
(ii) Tangential acceleration (aT )
⇒ ω2t = α
dv d
= = (2t) ( 4 t)2 = 4
dt dt 1
t= s
= 2 cms −2 2
Circular Motion | 141
Introductory Exercise 7.2
1. In uniform circular motion of a body the (a) At rest :
body is never in equilibrium as only one w
Required CPF = w − N =
force (centripetal) acts on the body which 2
forces the perform circular motion. mv2 w
⇒ = …(i)
gra 9.8 × 250 × (1.5 / 2) r 2
2. vmax = =
h 1.5 At dip :
−1
= 35 ms Required CPF = N ′ − w
mv2
= N′ − w …(ii)
r
l
l Comparing Eqs. (i) and (ii),
T sin θ T
w
N′ − w =
θ 2
T cos θ 3w 3
∴ N′ = = × 16 kN
r 2 2
mg
= 24 kN
3. (a) T sin θ = mg (b) At crest on increasing the speed (v), the
T cos θ = mrω2 value of N will decrease and for
g maximum value of v the of N will be just
∴ tan θ = zero.
r ω2
mv2max
g g Thus, = w−0
⇒ ω= = r
r tan θ ( l cos θ) tan θ wr
⇒ vmax =
2π g l sin θ m
or = ⇒ T = 2π
T l sin θ g = gr (as w = mg)
 1 = 10 × 250
2 
 2 = 50 ms −1
= 2π
9.8 (c) At dip :
1 9.8 mv2
∴ f = = rve s −1 N′ = w +
1 2π r
9.8 = w + mg
= × 60 rev min −1
2π = 2w = 32 kN
= 29.9 rev min −1 5. Case I. If the driver turns the vehicle
mg r
(b) T = = 2 mg
sin θ
= 2 × 5 × 9.8 = 69.3N µ mg
4. N=w
2

w'

w
142 | Mechanics-1
m v12 g
≤ µ mg ω=
r l sin φ
[where v1 = maximum speed of vehicle] If θ is the angle made by the string with
⇒ v1 ≤ µ gr the vertical
Case II. If the driver tries to stop the θ + φ = 90°
vehicle by applying breaks. i.e., φ = 90° − θ
Maximum retardation = µ g ⇒ sin φ = cos θ
∴ v22 = 0 2 + 2 (µ g) r g
∴ ω=
⇒ v2 = 2 µ gr l cos θ
= 2 v1 g
⇒ cos θ =
As v2 > v1, driver should apply breaks to l ω2
stop the vehicle rather than taking turn.
6. In the answer to question 3(a) if we
replace θ by φ

Introductory Exercise 7.3


→ →
1. |v1|= u and |v2|= v (say) 2. Ball motion from A to B :
B
→ → →
∆v = v2 + ( − v1)

– v1 →
v2

∆i 2R
C
h v
L
g A

0 2 = u2min + 2 ( − g) (2R)
→ ⇒ u2min
= 4 gR
v1
Ball motion from C to A :
→ →
= v2 − v1 v2 = u2min + 2 ( − g) h
→ → → → → → = 4 gR − 2 gh
∆v ⋅ ∆u = ( u 2 − v 1 ) ⋅ ( v 2 − u 1 )
⇒ v = 2 g (2R − h)
→ → → → → →
= u2 ⋅ v2 + u1 ⋅ v1 − 2 v2 − v1 3. Decrease in KE of bob
→ →
|v2 |2 2
+ |v1| = v + u 2 2 = Increase in PE of bob
2 2
= u − 2 gL + u KE of bob
→ 2
|∆v| = 2 (u2 − gL) = 1/2 mv02
PE of bob

|∆v|= 2 (u2 − gL) v0
= K (say)
Circular Motion | 143
v20= 2 gh
PE of bob = K + mgh v0 = 2 gl (1 − cos θ)
h
= 2 × 9.8 × 5 × (1 − cos 60° )
= 7 ms −1

1
mv20 = mgh
2
AIEEE Corner
Subjective Questions (Level 1)
1. v = 4 t2 v2
∴ = 21.65
dv R
∴ = 8t
dt i.e., v = 21.65 × 2.5 (Q R = 2.5 m)
i.e., aT = 8 × 3 = 24 ms −2 = 7.36 ms −1
v = 4 × 32 = 36 ms −1 (at t = 3 s) 1
(b) aT = a sin 30° = 25 × = 12.5 ms −2
v2 (36)2 2
∴ ac = = = 24 ms −2
4 54 5. (a) v = 2.0t = 2.0 × 1
→ →
Angle between a net and a t = 2.0 cms −1

−1 |a c|
v2 (2.0)2
θ= tan = tan −1 1 = 45° ∴ ac = = = 4 cms −2
→ R 1
|a T|
(b) v = 2.0t
2. v = 16 ms −1 and r = 50 m dv
∴ = 2.0
v2 (16)2 dt
∴ ac = = = 5.12 ms −2
r 50 i.e., aT = 2.0 cms −2
anet = a2c + a2T = (5.12)2 + 82
(c) anet = a2C + a2T
−2
(given aT = 8 ms )
= 4.47 cms −2
= 9.5 ms −2
2h
3. Speed (v) at the highest point ( P) 6. R = uT = u
v = u cos θ
g
u
ac = g r
θ u cos θ R
P
Centre of curvature u
v = u cos θ h = 2.9 m
h
Now, ac = g
v2
∴ =g Range (R) = 10 m
R
v2 u2 cos2 θ g
i.e., R= = ∴ u=R
g g 2h
3 R2 g
4. (a) ac = a cos 30° = 25 × i.e., u2 =
2 2h
= 21.65 ms −2
144 | Mechanics-1
Thus, centripetal acceleration of the stone mrω2 − T1 cos θ
⇒ cos θ =
(at point P) while in circular motion T1 sin θ − mg
u2 R2 g 1
= = ⋅ or T1 cos θ − mg cos θ = mrω2 − T1 cos θ
r 2h r
2
10 × 9.8 2 T1 cos θ − mg cos θ
= or ω =
2 × 2.9 × 1.5 mr
= 112.6 ms −2 3 3
2 × 200 ×   − 4 × 10 ×  
18.5  5  4
7. v = 18 km / h = ms −1 = 5 ms −1 =
18 4×3
v2
Angle of banking (θ) = tan −1 = 8.37 rad s −1
rg 1
8.37 × rev
(5)2 2π
= tan −1 =
10 × 10 1
min
1 60
= tan −1
40 = 39.94 rev min −1.
mv2 (b) From Eq. (ii),
8. = µ mg mg
r T2 = T1 −
2
v (5) 12 sin θ
i.e., µ= = = = 0.25 4 × 100
gr 10 × 10 4 = 200 −
 4
9. v = r g tan θ  
 5
= 50 × 10 × tan 30° = 150 N
= 17 ms −1 12. (a) For the block not to slip
10. mr ω2min = N …(i) N
L
and mg = µ N …(ii)
Solving Eqs. (i) and (ii),
µN µN
g
ω min = mg
µr N mg
10 mL ω2max = µ N = µ mg
=
0.15 × 3 µg
∴ ω max =
= 4.7 rads −1 L
11. (a) ( T1 + T2 ) cos θ = mrω2 …(i) (b) If the angular speed is gradually
increased, the block will also have
translational acceleration ( = α L)
T1
4m T1 sin θ besides centripetal acceleration
(= L ω2 ).
T1 cos θ T2 cos θ αL
4m T2 sin θ anet
T2
mg At the t) time of (
2 just slipping

At t = 0
and ( T1 − T2 ) sin θ = mg …(ii)
i.e., T2 cos θ = mrω2 − T1 cos θ
and T2 sin θ = T1 sin θ − mg
Circular Motion | 145
The block will be at he point of slipping, g g
i.e., < 1 or ω >
when Rω2 R
manet = µ mg g
∴ if ω≤ : θ = 0°
2 2 2
R
i.e., ( L ω ) + (α L) = µ g
i.e., bead will remain at the lowermost
or L2ω4 + α2 L2 = µ 2 g2 position.
µ 2 g2 2g g 1
or ω 4 + α2 = 2 For ω= : cos θ = =
R 2 g
R  2
L  
1  R
µ 2 g 2 4
or ω =  2 − α2  i.e., θ = 60°
 L 
15. (a) T = 2 ⋅ 2 ⋅ ω2
60°
13. F ′ = 2F cos =F 3
2 T T
2 1
PQ = QR = RP = a = 2 PO cos 30° 2m Ff

∴ a=r 3 = 4 ( 4)2
For the circular motion of P, Q or R = 64 N
mv2
= F′ = F 3 T − Ff = 1 ⋅ 1 ⋅ ω2
r
64 − Ff = ( 4)2
mv2 G mm
or = − 3 ⇒ Ff = 48 N
r ( r 3 )2
1 1 (b) T = 2 ⋅ 2 ⋅ ω2max and
 Gm  2 GM  2 T − Ff max = 1 ⋅ 1 ⋅ ω2max
or v=  = a 
r 3    ∴ 4ω2max − Ff max = 1 ⋅ ω2max
2πr i.e., 3ω2max = µ mg
Now, v =
T µ mg
∴ ω max =
2πr a/ 3 a3 3
∴ T= = 2π 1
= 2π
v 3 Gm 0.8 × 1 × 10
GM  2 =
 a  3
 
= 1.63 rad s −1
14. N sin θ = mrω2
(c) 100 = 2 ⋅ 2 ⋅ ω2max
O'P = r ∴ ω max = 5 rad s −1
Therefore, if the string can sustain a
tension of 1100 N, the angular speed of
R O N cos θ the block system will be 5 rad s −1.
N
θ In this case the frictional force ( Ff ) on the
O' P block of mass 1 kg will be given by the
N sin θ relation
mg 100 − Ff = 1 ⋅ 1 ⋅ 52
and N cos θ = mg ⇒ Ff = 75 N
sin θ rω 2
sin θ ( R sin θ) ω2 16. vmax =
gr l
∴ = or =
cos θ g cos θ g h
g 10 × 200 × 0.75
i.e., cos θ = = (2 r = 1.5 m)
Rω 2 1.5
For θ > 0° : cos θ < 1 = 31.6 ms −1
146 | Mechanics-1
17. If the block just leaves the surface of 19. At point P
sphere at point C l
θ
mv2
= mg cos θ …(i) h = l sin θ
r v0 T

[v = velocity of block at point C] P


2 mg sin θ
⇒ v = gr cos θ
mg Tmax = 2W
v0 = 2 mg

mv2
h
T − mg sin θ =
vel = v l
m 2
= [v0 + 2 gh]
l
r
θ m 2
or 2mg − mg sin θ = [v0 + 2 gl sin θ ]
l
52 + 2 g × 2 sin θ
Further, v2 = v20 + 2 gh or 2 g − g sin θ =
2
= v20 + 2 gr (1 − cos θ) …(ii) 1
Comparing Eqs. (i) and (ii), Solving, sin θ =
4
v20 + 2 gr (1 − cos θ) = gr cos θ  1
i.e,. θ = sin −1  
or (0.5 gr )2 + 2 gr (1 − cos θ) = gr cos θ  4
1
+ 2 (1 − cos θ) = cos θ 20. 8 = 5 + 5 cos θ
4
3 v
i.e., θ = cos −1   H
 4
2
mv θ 5 cos θ
18. T − mg cos θ = T=0
r
θ = 30° 8m 5
θ 4
5m
θ
3
T
3
∴ cos θ =
θ 5
mg cos θ
mg mv2
T + mg cos θ =
mv2 r
∴ 2.5 mg − mg cos θ = 3
r ∆t θ = cos −1
5
v2
∴ ac = = 25 − 5 3 ms −2 T is just zero.
r
mv2
at = g sin θ = 5 ms −2 ∴ mg cos θ =
r
Thus, anet = ( ac )2 + ( at )2 i.e,. 2
v = 5 g cos θ
= (25 − 5 3 )2 + (5)2 = 30
∴ v = 5.42 ms −1
= 625 + 75 + 25 − 10 3 The height (H) to which the particle will
= 26.60 m/s2 rise further.
Circular Motion | 147
2 2
v sin θ 30 16 ∴ F= 2
N + ( mg) 2
H= = ×
2g 2 × 10 25
= 64 m2 g2 + m2 g2
24
= = 0.96 m
25 = 65 mg
mv2 8 gR mv ′2
21. (a) N = =m = 8 mg (b) N ′ + mg =
R R R
m
v2 = 02 + 2. g.4 R mg + mg = 2 g ( h′ − 2R)
R
= 8 gR
or 2R = 2 ( h′ − 2R)
i.e., h′ = 3 R

4R
h = 5R
N

mg
F

Objective Questions (Level 1)


Single Correct Option
→ →
1. (a) As the speed (v) is increasing uniformly 2. |p1|= |p2 |= mv
the magnitude of centripetal acceleration →
 v2  →
p2 →
 =  will also keep on increasing besides –p1 p2
 r →
→ –p1
its direction as usual. ∆p →
∆p →
(b) v = kt (k = constant) p1
dv
∴ =k θ = 45°
dt →
p2
i.e., at = k →
∆p →
Although the magnitude of the tangential –p1
acceleration will remain constant but its

direction will keep on changing as the p2
direction of velocity would be changing. →
→ –p1
(c) v = kt ∆p
Rω = kt → 1°
ω = k′ t θ = 45°, |∆p|= 2 mv sin 22
k 2
(k′ = = constant) = 0.765 mv
R

dω θ = 90°, |∆p|= 2 mv sin 45°
= k′
dt
= 1.414 mv
α = constant. →
θ = 180° , |∆p|= 2 mv sin 90°
As the direction of α (angular acceleration)
is perpendicular to the plane of rotation of = 2 mv (max)
the body, it will remain constant both in →
θ = 270°, |∆p|= 2 mv sin 135°
magnitude and direction.
Option (c) is correct. = 1.414 mv
148 | Mechanics-1

θ = 360°, |∆p|= 2 mv sin 180° 6. Normal acceleration
= Tangential acceleration
= 0 (min)
v2
Option (c) is correct. = 5 (cms −2 )
R
3. ∴ v = 10 cms −1 (Q R = 20 cm)
Using, v = 0 + 5t
θ
10 = 5t
⇒ t =2s
Option (b) is correct.
PEmax = mgh = mgl (1 – cosθ)
h 7. Mass = 2π kg
KE = 0 2π kg
∴ Mass per unit length =
2π (0.25) m
∴KE max = mgl (1 − cos θ) [as here PE = 0]
Option (c) is correct. T
∆θ
4.
B
v2 = u2 + 2(– g) (2l) ∆θ
OB = OA
l= = 4gl – 4gl
=0
T
Light O
1
rigid ∆m = r ∆θ = ∆θ (Q r = 0.25)
rod 0.25
1
= kgm −1
u = √4gl 0.25
A rev
ω = 300
= 4 gl − 4 gl min
2π rad
=0 = 300 ×
60 s
If the mass m is given velocity u ( = 4 gl ) at
point A it will complete circle to reach point = 10π rad s −1
B (the highest point) with zero velocity. From figure
[If in place of light rod these is light string  180° − ∆θ  2
2T cos   = ( ∆m) rω
the minimum value of u at A for the mass  2 
to reach point B will be 5gl and the ∆θ
minimum velocity at B will be g l.] or 2T sin = ( ∆m) rω2
2
5. As explained in question 4 speed at lowest ∆θ
point or 2T = ( ∆m) rω2
2
u = 5 gl or T ∆θ = ( ∆θ) rω2
At the lowest point or T = r ω2
mu2
T − mg = = 0.25 × (10π)2
l
≈ 250 N (Q π2 ≈ 10)
m 5 gl
= Option (d) is correct.
l
= 5 mg 8. Maximum speed of car = µgr

⇒ T = 6 mg = 0.3 × 10 × 300
Option (d) is correct. = 30 m/s
Circular Motion | 149
18 Frictional force = manet
= 30 × km/h
5 725
= 0.36 × 10−3 kg × × 10−2 ms −2
= 108 km/h 3
Option (c) is correct. = 32.4 µN
9. T sin θ = mrω2 = m ( l sin θ) ω2 Option (a) is correct.
12. At the lowermost point
θ θ

l
T T cos θ
θ l

r T u=0
T sin θ
h
v2 = 2gh
and T cos θ = mg mg
g
∴ cos θ = mv2
l ω2 T − mg =
l
2
ω l = revs −1 m ⋅ 2 gh
π or T= + mg
l
2
= (2π rad) s −1  2h
π = mg 1 + 
 l 
= 4 rads −1
10 5 Option (d) is correct.
∴ cos θ = = 2π
1 ( 4)2 8 13. ω A = rad min −1
3
 5
i.e., θ = cos −1  
8
Option (d) is correct.
A B
10. From above question no. 9.
2
t = mlω
100 8
= × 1 × 42 = N
1000 5
and ω B = 2π rad min −1
Option (b) is correct.
1 ∴ ω BA = ω B − ω A
11. α = rads −1 and R = 25 cm  2π 
3 = 2π −  rad min
−1
25  3
∴ at = Rα = cms −2 4π
3 = rad min −1
Thus, ωt = 0 + α t 3
1 Time required for B to complete one
= ×2 [Q t = 2 s]
3 revolution w.r.t. A
2 2π 3
 2 = = min
∴ aN = R ω2t = 25 ×   cms −2 4π / 3 2
3
725 = 1.5 min
Thus, anet = a2N + a2t = cms −2
3 Option (c) is correct.
150 | Mechanics-1
14. Let all the particles meet at time t
∴ Distance travelled by B in t second :
(seconds)
B 2.5 ms–1 Distance travelled by C in t second
= 2.5t : 2t
= 5:4
Option (c) is correct.
A C
1 ms–1 2 ms–1

JEE Corner
Assertion and Reason
1. For stopping car : →
→ |BD| v 2
Maximum retardation i.e., |a av |= =
t AB t AB
Maximum frictional force
= →
m → |AB| R 2
µ N µ mg Now, |v av|= =
= = =µ g t AB t AB
m m

v2 = u2 + 2as ∴
|a av|
=
v 2
= ω (angular velocity)
02 = v2 + (2)( − µ g) d [Q Initial velocity = v] →
|v av| R 2
v2
∴ d= Thus, assertion is correct.
2µ g
In circular motion, when speed is
For circular turn of car : constant, the angular velocity will
Centripetal force obviously be constant; but this reason does
not lead to the result as explained.
= Maximum frictional force = µ mg
Option (b) is correct.
mv2
∴ = µ mg 3. A frame moving in a circle with constant
d′
v2 speed can never be an inertial frame as
i.e., safe radius = d′ = ⇒ d′ = 2d the frame is not moving with constant
µg
velocity (due to change in direction).
Thus, Assertion and Reason are both Reason that the frame is having constant
correct and Reason is the correct acceleration is false.
explanation of the Assertion.
Option (c) is correct.
Option (a) is correct. → →
→ → 4. If speed is constant angle between v and a
→ vB− v A will always be 90°
2. a av =
t AB →
ω
θ
→ → →
vB B a = ac
→ → →
vA vA
D BD
=
t AB A
→ →
∴ v⋅a = 0
Circular Motion | 151
2
If speed is increasing angle θ between v
→ v
⇒ R=

and a will be less than 90°. ac
→ → (2)2
v aT = =2m
2

θ
a T = − ^i ms −2
→ → →
a ac Speed is decreasing (as a T is −ive) at a
rate of 1 ms −1 per second i.e., 1 ms −2 .
Both Assertion and the Reason are correct
→ → but reason has nothing to do with the
∴ v ⋅ a will be positive.
assertion.
→ →
v aT Option (b) is correct.
→ → →
θ 6. a = a T + a c (Reason)
→ →
ac a

ac
A
→ → →
If speed is decreasing angle between v and → aT = g
a

a will be greater than 90°.
→ →
∴ v ⋅ a will be negative. → v2
∴ |a |= + g2
Assertion is correct. r
→ → →
Reason ω⋅v =0 as both are ⇒ |a |> g (Assertion)
perpendicular to each other. We see that both Assertion and the
Reason is also true but not the correct Reason are correct and the reason is the
explanation of the assertion. correct explanation of assertion.
Option (b) is correct. Option (a) is correct.
→ ^ 7. At points A and C : The bob is
5. v = 2 i ms −1
momentarily at rest.

→ x
anet
2j
v = 2z
–i A C

→ B
a = − ^i + 2 ^j ms −2 →
g

g
→ ^ −2
∴ a c = 2 j ms i.e., v = 0 (Reason)
→ → −2 → v2
ac = |a c |= 2 ms ∴ |a c |=
=0

R
v = |v|= 2 ms −1 but net acceleration is not zero (see figure)
v2 i.e., Assertion is false.
ac =
R ∴ Option (d) is correct.
152 | Mechanics-1
8. v (speed) = 4 t − 12 N cos θ is balanced by mg (weight of
For t < 3 (time unit) speed is negative, particle).
which can’t describe a motion. Thus, Acceleration is not along the surface of the
assertion is correct. funnel. It is along the centre O of the
As speed can be changed linearly with circle. Thus, reason is true.
time, the reason is false. Option (d) is correct.
Option (c) is correct.  mv2 
11. Centripetal force   = N + mg
9. In circular motion the acceleration  r 
changes regularly where as in projectile
motion it is constant. Thus, in circular N
→ → →
motion we can't apply v = u + a t directly,
whereas in projectile motion we can say mg

reason that in circular motion gravity has


no role is wrong.
Option (c) is correct.
i.e., Centripetal force (reason ) ≥ wt ( mg) of
10. N = mg cos θ
water
N
for N = 0, v = gr
O
N sinθ θ mg cos θ If at the top of the circular path v ≥ gr
mg i.e., if bucket moved fast, the water will
not fall (Assertion).
As assertion and reason both are true and
reason is the correct explanation of the
Therefore, assertion is wrong. assertion of the option would be (a).
Particle performs circular motion due to
N sin θ
mv2
∴ = N sin θ
r
Objective Questions (Level 2)
Single Correct Option
1  1
1. KE  mv2  = Change in PE  k( ∆x)2  2. The particle will remain in equilibrium till
2  2  ω is constant. Any change in the value of ω
∆x = Length of spring (Collar at B) will displace the particle up (if ω increase)
and down (if ω decreases). Thus, the
− Length of spring (Collar at A)
equilibrium is unstable.
= (7 + 5)2 + 52 m − 7m = 6 m Option (b) is correct.
1 1 3. Centripetal force = µ mg
Thus, × 2 × v2 = × 200 × 62
2 2 or mrω2 = µ mg
2
i.e., v = 3600 5a 2
mv2 or ω =µ g
∴ Normal reaction = 4
r 4g  1
2 × 3600 or ω2 = as µ = 
= = 1440 N 15a  3
5
Option (d) is correct.
Option (a) is correct.
Circular Motion | 153
2
4. Acceleration at B = Acceleration at A v 2R
or R1 = =
v2 g cos θ2 5 cos θ2
∴ = g sin θ
r 2R
=
2 gr (1 − cos θ) 5 ⋅ cos 37 °
or = g sin θ
r 2R
=
or 2 (1 − cos θ) = sin θ  4
5⋅  
 5
or [2 (1 − cos θ)]2 = 1 − cos2 θ
R
or (5 cos θ − 3) (cos θ − 1) = 0 =
2
As cos θ = 1, i.e., θ = 0° is not possible.
3 Option (c) is correct.
cos θ = mv2
5 7. mg cos θ =
−1 3 a
i.e., θ = cos a
5 42 + 2 ⋅ g ⋅
Option (c) is correct. or g cos θ = 4
a
5. At point P (for the circular motion) a a
mv2 g− u2 + 2 ⋅ g ⋅
mg cos θ − N = or g 4 = 4
R a a
If at point P skier leaves the hemisphere. 3 a
or g ⋅ a = u2 + 2 g
N =0 4 4
mv2 ag
∴ mg cos θ = ⇒ u=
R 2
m  R Option (c) is correct.
or mg cos θ = 2g h + 
R  4 v2 4
8. = 2
m  R r r
or mg cos θ = 2 g R (1 − cos θ) +  2
R  4 ⇒ v=
1 r
⇒ cos θ = 2 (1 − cos θ) +  ∴ Momentum = mv
 4
2m
−1 5 =
i.e., θ = cos r
6
Option (c) is correct. 9. N cos θ = mrω2
6. Velocity at B and N sin θ = mg
v = 2 gh ∴ N = m g2 + r2ω4
4
= 2 gR (cos θ2 − cos θ1)  2π 
= m g2 + r2  
T
= 2 gR (cos 37 ° − cos 53 ° )
4
2 22π 
 4 3 = 10 10 + (0.5)  
= 2 gR  −   1.5 a
 3 5
2 gR = 128 N
=
5 Option (b) is correct.
1
For the circular motion at B, when block 10. θ = ω 0 t + αt2
just leaves the track 2
 1 
v2 = ω 0 + α t t
= g cos θ2  2 
R1
154 | Mechanics-1
 v 1  1 v2   4 πR   t=2s
=  + − ⋅   t R
 R 2  R 4 πR   v  
v v  4πR
=  − vB
 R 2R  v vB

= AD 30° 60° B'


= 4π
30 m
= 2 rev.
1 PR
11. H = gT2 Further, = cos 30°
2 PQ
vA ⋅ 2 3
=
30 2
⇒ v A = 7.5 3 ms −1
H Substituting value of v A in Eq. (i),
vB = 7.5 ms −1
R
mv2
14. mg cos θ − N =
r
2H
⇒ T= When breaks off N = 0
g
mv2
2π g ∴ mg cos θ =
ω= = 2π r
T 2H N
2g

H Acceleration
= g sin θ
2π θ θ
12. ω (minute hand) = rad s −1 sin
3600 mg
O mg

ω (second hand) = rads −1
60
For second hand to meet minute hand for
the first time.
2 gr (1 − cos θ)
2π + Angle moved by minute hand in t or g cos θ =
second r
= Angle moved by second hand in t second
3
2π 2π √5
or 2π + t= t
3600 60 θ
t t 2
1= −
60 3600
t 59 or cos θ = 2 (1 − cos θ)
1= × 2
60 60 or cos θ =
3600 3
⇒ t= s
59 Acceleration of particle when it leaves
sphere
Option (d) is correct.
= g sin θ
13. ( PR)2 + (QR)2 = ( PQ)2
g 5
(v A ⋅ 2)2 + (vB ⋅ 2)2 = 302 =
3
v2A + v2B = 225 …(i)
Option (b) is correct.
Circular Motion | 155
15. For minimum velocity (v) : aT = g
tan θ = tan 45° = 1 ∴ anet = a2c + a2T
µ =1 (given) = (3 g)2 + g2 = g 10
As µ = tan θ
Option (a) s correct.
θ is the angle of repose.
17. N = mrω2
N cos θ
N
and µN = mg
g
N sin θ ∴ µ= µN
rω2
µN cos θ 10 N
= = 0.2 mg
µN µN sin θ 0.2(5)2
θ
Option (c) is correct.
Therefore, the automobile will be at the gra
point of slipping when its velocity is zero 18. vmax =
h
For maximum velocity (v′ ) 10 × 200 × 075.
mv ′2 =
N sin θ + µ N cos θ = 1.5
r = 31.62 m/s
Also N cos θ = mg + µ N sin θ
19. At point A : Velocity is zero and as such its
i.e., N (cos θ − µ sin θ) = mg …(ii) vertical component will also be zero.
Dividing Eq. (i) by Eq. (ii), At point B : Velocity is completely
sin θ + µ cos θ v ′2 horizontal and as such its vertical
=
cos θ − µ sin θ gr component will again be zero.
A T
Now, as θ = 45° and µ = 1
v′ = ∞ θ
R
Option (d) is correct.
16. For the particle to just complete the circle P
the value of
v
u = 5 gR
v1
Particle's velocity (v) when it is at B i.e., B
when its velocity is vertical would be given In figure, TP = R cos θ
by the relation
At point P :
v2 = 2 g ( R cos θ)
v
v = 2 gR cos1/ 2 θ
R
∴ v⊥ = v sin θ = 2 gR cos1/ 2 sin θ
For v⊥ to be maximum
d
u = √5gR v⊥ = 0

v2 = u2 + 2 ( − g) R d
i.e., 2 gR cos1/ 2 θ sin θ = 0
= 5 gR − 2 gR dθ
1
= 3 gR or cos1/ 2 θ cos θ + cos −1/ 2 θ( − sin θ) sin θ = 0
At B : 2
v2 sin2 θ
ac = = 3g or cos θ cos θ − =0
R 2 cosθ
156 | Mechanics-1
or 2 cos2 θ − sin2 θ = 0 21. At highest point B
or 3 cos2 θ = 1 B
1
or θ = cos −1 T
3 mg
Option (b) is correct.
20. At any time R
at = ac
u
dv v2 A
i.e., =
dt R mv2
−2 dt T + mg =
∴ ∫ v dv = ∫ R + k l
(where v = velocity at point B)
v −2 + 1 t
i.e., = +k Thus, T =0
−2 + 1 R
mv2
1 1 if mg =
or − = +k l
v R 2
i.e., v = gl
At t = 0, v = v0 (given)
1 0 If u = velocity at the lowest point A
∴ − = +k
v0 R v2 = u2 + 2 ( − g) (2l)

i.e.,
1 or gl = u2 − 4 gh
k= −
v0 ⇒ u2 = 5 gl
1 t 1
Thus, − = − i.e., u = 5 gl
v R v0 → →
1 tv − R (If T = 0, T ⋅ a = 0)
or − = 0
v Rv0 Option (b) is correct.
Rv0 22. For any value of u at the lowest point both
or u= → →
R − tv0 T and a will be towards the centre of the
v0 circle and thus
or ω= → →
R − tv0 T ⋅ a will be positive.
dθ v0
i.e., = Option (d) is correct.
dt R − tv0
v0 23. Change in PE of the system
or dθ = dt m
R − tv0
2π T v
∫ 0 dθ = ∫0 R −0tv0 dt l M
l/2 M m
or 2π = − [log e ( R − tv0 )]T0
= log e R − log e ( R − Tv0 )
R − Tv0
or log e = = − 2π
R
T
or 1 − v0 = e−2 π l
r = Mg + mgl
R 2
or T= (1 − e−2 π )
v0 M 
= + m gl
 2 
Option (c) is correct.
Circular Motion | 157
2
24. Decrease in KE = Increase in PE = (2 v) − 4 gL (Qu = 2 v)
1 M  or 3v2 = 4 gL
i.e., mu2 =  + m gl
2  2  4 gL
i.e., v = gL = 2
∴ Initial speed given to ball, 3 3
 M + 2m  Option (b) is correct.
u=   gl
 m  gL
26. u = 2 v = 4
3
25. Maximum and minimum velocities will be
1
respectively at the lowest and the highest ∴ KE at the lowermost position = mu2
points. 2
(Min. velocity)
8 mgL
v
=
3
27. Let ω = velocity of the particle when
moving downwards.
∴ ω2 = v2 + 2 gL
4 gl
ω = + 2 gl
u
3
(Max. velocity) 10 gl
or ω=
3
v2 = u2 + 2 ( − g) 2L
= u2 − 4 gl Option (a) is correct.

More than One Correct Options


1. At point C :
v' B
vmin A aT = 0
T
g B mg
ac(min) aT = g sin θ
θ
l
C
aT = g

ac(max) g v
2
mv ′
vmax or 2mg + mg =
D aT = 0 l
or v ′2 = 3 gl
aT is maximum and ac is somewhere i.e., v ′ = 3 gl
between maximum value (at D) and Option (d) is correct.
minimum value (at A).
Now, v2 = v ′2 + 2 g (2l)
∴ Option (b) is correct.
∴ v2 = 3 gl + 4 gl
At point D :
ac is maximum while be aT is minimum. = 7 gl
∴ Option (d) is correct. i.e., v = 7 gl
2. At point B : Option (b) is correct.
mv ′2
T + mg =
l
158 | Mechanics-1
3. N sin α = mg 4. Particle can’t have uniform motion
N sin θ because of change in direction of motion
N
i.e., its velocity value. [Option (a)]
r Particle can’t have uniformly accelerated
N cos θ
motion as acceleration changes direction
mg even if speed is constant. [Option (b)]
h Particle can’t have not force equal to zero
α as centripetal force would be required for
the circular motion. [Option (c)]
5. For this see figure in answer 3.
N cos α = mrω2
If ω is increased N cos α will increase.
g g
⇒ tan α = = Thus, N will increase (as α is constant)
rω2 r tan θ
[Option (b)]
2π g
i.e., = And as such net force
T r tan α
= N 2 + m2 g2 + 2 N cos (90° + α)
r tan α
or T = 2π
g on the block will increase.
If α is increase r will also increase and as ∴ Option (a) is correct.
such T will increase. As N increases, the value of N sin α
Option (c) is correct. (acting opposite to mg) will increase and
the block will upwards i.e., h will increase.
h tan2 α
T = 2π
g
Thus, if h is increase, T will increase.
Option (a) is correct.
Match the Columns
1. At point B : v (c) → (r).
2 2
v = u − 2 gl B Tangential acceleration of bob :
= 12 gl − 2 gl aT = g = 10 ms −2
i.e., v = 10 gl (d) → (p).
= 10 × 10 × 1 2. v = 2 t
= 10 ms −1 dv
∴ =2
(a) → (p). b dt
A
Acceleration of bob : i.e., a = 2 ms −2
v2 10 g At t = 1 s : v = 2 ms −1
ac = = = 100 ms −2
l 1 v2 22
∴ ac = = = 2 ms −2
aT = g = 10 ms −2 r 2
anet = a2c + a2T aT = 2 ms −2

= 1002 + 102 ms −2 ∴ anet = 22 + 22 = 2 2 ms −2


→ →
(b) → (s). Angle between a net and v = 45°
Tension in string :
[As ac = aT ]
mv2 1 × 100
T= = = 100 N
l 1
Circular Motion | 159
→ → → →
(a) a ⋅ v = |a ||v|cos 45° F=f
F
→ 1 i.e., =1
= a net ⋅ v f
2
1 ∴ (c) → (r).
= 2 2 ⋅2
2 If v is increased F will increase which will
= 4 unit. automatically increase the value of f .
Thus, (a) → (r). ∴ (d) → (p).
→ → → →
(b) |a × ω|= |v||ω|sin 90° 4. Speed of particle is constant

→ → |v|2
[ω will be perpendicular to the plane of ∴ |a |=

circle] |r |
= anet ω ( 4)2 + ( − a)2
v ( −6)2 + ( b)2 =
= anet (3)2 + ( − 4)2
r
=2 2⋅
2 16 + a2
36 + b2 = …(i)
r 5
= 2 2 unit → → → →
(A) r ⋅ v = |r ||v|cos 90° = 0
Thus, (b) → (p)
→ → → → i.e., (3 ^i − 4 ^j) ⋅ ( 4 ^i − a ^j) = 0
(c) v⋅ ω = |v||ω|sin 90°
v 12 + 4 a = 0
= vω = v ⋅
r a= −3
2
(2) ∴ (a) → (s).
=
2 (B) Substituting value of a ( = −3) in Eq. (i)
= 2 unit 36 + b2 = 5
Thus, (c) → (q)
→ → → → 36 + b2 = 25
(d) |v × a |= |v||a |sin 45°
1 b = 25 − 36
= v anet ⋅
2 ∴ (b) → (s).

= 4 unit (C) r = 3 ^i − 4 ^j
Thus, (d) → (r) →
∴ r = |r |= 5
M v2 h
3. N1 = g −  ∴ (c) → (r).
2  ra 
→ → → → ^
On increase v, the value of N1 will (D) ( v × a ) = |v||a |sin 90° k
decrease. → → ^
= |v||a | k
Thus, (a) → (q). → → →
M v2 h Thus, r ⋅ ( v × a )
N2 = g + 
2  ra  → → → ^
= r|
⋅ v||a |k
On increasing v, the value of N2 will → → ^
increase. = (3 ^i − 4 ^j) ⋅|v||a|k = 0
∴ (b) → (q). ^ ^
[as ^i ⋅ k = 0, ^j ⋅ k = 0]
As the centripetal force ( F ) would be
provided by the frictional force (f ) ∴ (d) → (s).
160 | Mechanics-1
5. (A) As speed is constant R 2 R 2 2 2
= = ⋅1 =
−1 AB arc πR π
Average speed = 1 ms
→ speed 2
v2 B
∴ (b) → (q)
→ →
v1 → v1 (C) Modulus of average acceleration
∆v mg

= |a c | [as speed is constant]
A → → →
|v − v1| |∆v| v 2
= 2 = =
t AB t AB R
π
2
2
∴ (a) → (s) = 2 [ Q v = 1 ms −1 and R = ]
AB π
(B) Modulus of average velocity = ∴ (c) → (r)
t AB
Centre of Mass, Conservation
8 of Linear Momentum, Impulse
and Collision

Introductory Exercise 8.1


1. If a body is placed in a uniform 2. The centre of mass of a rigid body may lie
gravitational field, the CG of the body inside, on the surface and even outside the
coincides with the CM of the body. body. The CM of a solid uniform sphere is
n
Σ mi ri
→ at its centre. The CM of a solid ring is at
→ i =1 the centre of the ring which lies outside
rCM = n the mass of the body thus, the statement
Σ mi
i =1 is false. (For further details see answer to
n → 1 Assertion and Reason JEE corner).
Σ mi g i ri
→ i =1 3. Centre of mass always lies on the axis of
while r CM = n symmetry of the body, if it exists. The
Σ mi g i statement is thus true.
i =1
4. Statement is true.
If a body is placed in a uniformly
→ 5. As more mass is towards base.
increasing gravitational field ( g ) in the
r
upward direction the CG of the body will Distance < .
4
be higher level than the CM. And, if the
body is placed in a uniformly decreasing 6. If two equal masses are kept at
gravitational field in the upward direction co-ordinates (R, 0) and ( −R, 0), then their
the CG will be at a lower level the CM. centre of mass will lie at origin.
7. 3 kg (1/2, 3/2)

CG
CM CM
CG
g g
increasing decreasing
1 kg 2 kg
CG shifts from CM according to the (0, 0) (1, 0)
magnitude and direction of the m1 X1 + m2 X2
gravitational field (by some other agency X CM =
m1 + m2
eg, earth) in which the body is placed.
m1 y1 + m2 y2
In zero gravitational field CG has no YCM =
meaning while CM still exists, as usual. m1 + m2
r = X 2CM + YCM
2

= Distance of centre of mass from A


162 | Mechanics-1
8. y A1 y1 − A2 y2
YCM =
A1 − A2
9. A1 = 4 a2 , x1 = a, y1 = a
3a 3a
A2 = a2 , x2 = , y2 =
2 2
x
O

Introductory Exercise 8.2


1. Method 1
1 kg t=0s 2 kg
A x (10 – x) 2.
CM (i)
t=0s a

e
+v
1x = 2 (10 – x) T T a
At t = 0s 20

°
⇒ B mg

60
x= cm A
3 s in

in
30
°

s
1 kg 2 kg 60°

mg
30°
A y 7–y
2m
CM (f) 1m
t=1s t=1s

1 y = 2 (7 − y)
14
⇒ y= cm
3
Displacement of CM
mg sin 60° − T = ma …(i)
= Position of CM ( f ) − Position of CM ( i)
T − mg sin 30° = ma …(ii)
= ( y + 2) − ( x)
Adding above equations,
14 20
=  + 2 −   mg (sin 60° − sin 30° ) = 2 ma
3  3
 3 − 1
=0 ⇒ a= g 
 4 
Method 2
→ →
M vCM = m1v1 + m2v2 a1 and a2 are at right angle
= 1 ( 2 ms −1) + 2 ( − 1 ms −1) →

m a1 + m a2 1

∴ a CM = = ( a1 + a2 )
=0 2m 2
As velocity of CM is zero, there will not be → a g( 3 − 1)
or |a CM|= =
any change in the position of the CM. 2 4 2
Centre of Mass, Conservation of Linear Momentum, Impulse and Collision | 163
Introductory Exercise 8.3
20 v + 60 ( + 3) $
20 × 20 $i + 30 × 20 $i + 40 × 20 k
1. vCM = =0 vCM =
20 + 60 20 + 30 + 40
60 kg + ve
y
(10i + 20k) cms–1
20 kg
30 g

v
3 m/s j Rest
x
20 kg → i 20 g
60 kg v k

[As there is no force along horizontal


z
direction].
⇒ Velocity of trolley (v) = − 9 ms −1 20 × 0 + 30 (10 ^i + 20 k) + 40 v
^ →
vCM =
Total energy produced by man 20 + 30 + 40
= KE of man + KE of trolley ^ ^
1 1 300 ^i + 600 ^j+ 40 k = 400 ^i + 600 ^j+ 800 k
= × 60 × 32 + × 20 × ( − 9)2 = 1.08 kJ
2 2 → ^
⇒ 40 v = 100 ^i + 600 ^j + 200 k
2. On streching and then releasing the
→ ^
spring the restoring force on each block at i.e., v = ( 2.5 ^i + 15 ^j + 5 k ) cms −1
instant will be same (according to
4. Velocity of projectile at the highest point
Newton’s 3rd law of motion). Now, as force
is same momentum p of each block will before explosion = 10 2 ^i ms −1
also be same (∆t being same) [As according → ^
v = 10 √2 i
to Newton’s second law of motion rate of 20
change of momentum of a body is directly
proportional to the net force applied on 45°
20 cos 45° x
the body.] R/2
= 10√2 ms–1
p2
Now, as KE = ,
2m As no extra external force would be acting
KE of blocks at any instant will be during explosion, the velocity of CM will
inversely proportional to their respective not change
masses. m → m→
⋅0+ v
2 2 = 10 2 ^i
3. As no external force acts on the system of
m
particles, the velocity of CM shall not →
change. Thus, ⇒ v = 20 2 ^i
y R
20 cms–1 Range of rest half part = ×2
2
30 g
(as the velocity of the projectile has
20 cms–1 doubled at the highest point)
j u2
x = R= (as α = 45°)
20 cms–1 i g
20 g
k
(20)2
= = 40 m
10
z
164 | Mechanics-1
Therefore, the rest half part will land at a = Height of point P + Extra maximum
40 height attained
distance of  40 +  m i.e., 60 m from the
 2 by second half part
point of projection. 1 ( 2000)2 sin2 α
20 √2 i = 20 × 1 − g × 12  +
 2  2g
u 2000 1
= 15 + ×
2 × 10 5
R = 15 + 20
5. At point P = 35 m
6. Momentum of platform + boy + stone
along x-axis after throwing stone = before
u = √(40)2 + (20)2
throwing stone
–1
P α = √2000 ms  1
–1
∴ (60 + 40) v + 110  = 0
20√2 ms t = 15  2
20 ms–1
15 m 5 2
⇒ v=− ms −1
t=0s 45° 100
60 kg 10 ms–1
Horizontal velocity = Initial horizontal 40 kg 45°
X
velocity 1 kg
−1
= 20 2 cos 45° ms
= 20 ms −1
Vertical velocity
10 ms–1
= 20 2 sin 45° − g ⋅ 1 (as t = 1s)
= 70 ms −1 10 cm 1 kg
→ X
T = √2 s
∴ v (velocity of projectile at point P i.e., at
t = 15 just before explosion)
2 × 10 sin 45°
= 20 ^i + 10 ^j Time of flight of stone = 2s
g
Now, as the projectile breaks up into two ∴ Horizontal displacement of platform
equal parts and one part comes to rest, the (+ boy)
velocity of other half part after explosion
=vT
will be
5 2 1
→ →
u = 2 v = 40 ^i + 20 ^j ms −1 =− × 2=− m = − 10 cm
100 10
Angle of projection (α) of 2nd half part 7. Thrust due to the upward component of
after explosion at point P. the velocity of the bullet will rotate the
20 movable end of the barrel and thus the
α = tan −1  
 40 √5
1
bullet leaving the barrel will travelling at
−1 1
an angle greater than 45° when it comes
= tan α
2 out of the barrel.
2
1 Bullet
sin α =
5
Maximum height attained by second half Bullet θ = 45° θ' > θ
part
Centre of Mass, Conservation of Linear Momentum, Impulse and Collision | 165
Introductory Exercise 8.4
1. To just lift rocket off the launching pad 180 kg
∴ t= =9s
Thrust force = Weight 20 kgs −1
 dm  200
or v − i.e., v = − 9.8 × 9 + 1.6 × 103 ln
 = mg 20
 dt 
= 3.6 kms −1
 dm  mg
or −  = 2. Mass at time t, m = m0 − µ t
 dt  v
dm
(20 + 180) × 9.8 ∴ = −µ
= dt
1.6 × 103
ma = thrust force − mg
= 1.225 kgs −1
 dm 
(i) Rate of consumption of fuel = 2 kgs −1 or ma = v  −  − mg
 dt 
∴ Time required for the consumption of fuel
180 kg or ma = µ v − mg
t= = 90 s d2 x
2 kgs −1 or ( m0 − µt) 2 = µ u − ( m0 − µ t) g (Qv = u)
dt
Ultimate speed gained by rocket m
m 3. v = u − gt + v ln  0 
v = u − gt + v log e  0  …(i)  m
 m m0
= 0 − gt + u ln
Substituting u = 0, v = 1.6 kms −1, t
m0 1 − 

m0 = (20 + 180) kg and m = 20 kg in Eq. (i).  3
200 3
v = − 9.8 × 90 + 1.6 × 103 ln   = − g 1 + u ln (at t = 1 s)
 20  2
−1 3
= 2.8 kms
= u ln − g
(iii) Rate of consumption of fuel = 20 kgs −1 2

Introductory Exercise 8.5


→ ^ ^ →
1. u (at t = 0 s) = ( 10 3 i + 10 j ) ms −1 2. Impulse ( J ) imparted
20 m/s = Change in momentum in the time
interval
60° t = 0 s to t = 2 s
= m [(Velocity at t = 2 s)
At t = 1 s − (Velocity at t = 0 s)]
= 2 kg [( 4 i + 4 j) ms − ( 4 ^j ) ms −1 ]
^ ^ −1
Horizontal velocity = 10 3 ^i ms −1
= 8 ^i Ns (Q 1 kg ms −1 = 1 Ns)
Vertical velocity = (10 − g ⋅ 1) ^j ms −1
3. Spring will become taut when the ball
= 0 ^j ms −1 would go down by 2 m.
→ ^ −1
∴ v = 10 3 i ms v2 = u2 + 2 × g × 2
→ → →
Change in velocity, ∆ v = v − u or v = 2 10 ms −1
= ( 10 3 ) ^i − (10 3 ^i + 10 ^j) ∆p = m ∆v
= 1(2 10 − 0) kg ms −1
= − 10 ^j ms −1

= 2 10 kg ms −1 2m
i.e., ∆ v = 10 ms −1, downwards. ∴ Impulse imparted = 2 10 Ns.
166 | Mechanics-1
Introductory Exercise 8.6
1. At maximum extension of the spring both 3. Let us consider the following case
blocks will move with same velocity v. v v1 = 0 v2' v1'
+ ive
m2 m1 m2 m1
1.0 ms–1 2.0 ms–1
 m − m2   2 m2 
3 kg 6 kg v2 ′ =  1  v2 +   v1
 m1 + m2   m1 + m2 
∴ ( − 1.0) × 3 + ( + 2.0) × 6 = (3 + 6) v m − m2
= 1 v2 (as v1 = 0)
⇒ v = + 1 m/s m1 + m2
Applying law of conservation of ∆ K (Transfer of KE of particle of mass
mechanical energy m2 )
1 1 1 1 1 1
3 ( − 1)2 + 6 (2)2 = 9 × 12 + x2 = m2 v22 − m2 v ′22
2 2 2 k 2 2
−1 2
Substituting k = 200 Nm 1 1  m − m2 
= m2 v22 − m2  1 v2 
x = 0.3 m = 30 cm 2 2  m1 + m2 
2. Kinetic energy of particle = K  2
1 1  m − m2  
⇒ mv2 = K = m2 v22 1 −  1  
2 2   m1 + m2  
2K From above we conclude that ∆ K will be
∴ v=
m maximum when
v Rest V m1 − m2 = 0
m2
m m m m i.e., =1
m1
During collision the EPE of the system
would be at its maximum value when both 4. Continuing from the previous answer
the particles move with same velocity V 2
given by the relation ∆K  m1 − m2 
=1−  
mv + m × 0 = ( m + m) V K  m1 + m2 
(Law of conservation of momentum) ( m1 + m2 )2 − ( m1 − m2 )2
=
v ( m1 + m2 )2
⇒ V =
2 4 m1m2
=
1 2K ( m1 + m2 )2
=
2 m
5. Substituting m2 = m and m1 = 2m in the
Applying law of conservation of result of the previous question no. 4.
mechanical energy
u Rest
1
K + 0 = ( m + m) V 2 + EPE max m
2 2m

or K = mV 2 + EPE max
∆ K 4 (2m) ( m)
1 2 K  =
or K = m⋅   + EPE max K (2m + m)2
4 m  8
K =
or EPE max = 9
2
Centre of Mass, Conservation of Linear Momentum, Impulse and Collision | 167
6. As the collision is elastic ∴ Total number collisions between the
u Rest v1 v2 balls = 2.
m − m1 2m1
m1 m2 m1 m2 7. v ′2 = 2 v2 + v1
m1 + m2 m1 + m2
m1 − m2 v2 v1 v2' v1'
v1 = u
m1 + m2
m2 Before m1 m2 After m1
and v2 = v1 + u
Elastic collision
m − m2
= 1 u+u m − m2 2m2
m1 + m2 v ′1 = 1 v1 + v2
2 m1 m1 + m2 m1 + m2
= u
( m1 + m2 ) We see that v ′2 ≠ v1 and v ′1 ≠ v2 .
Before first collision of B with C However, if m1 = m2
A B C v2′ = v1 and v ′1 = v2
v
4m 4m 4m Ans. No.
Rest 8. In elastic collision of two bodies of equal
masses, the velocities get interchanged.
After collision of B with C
m − 4m Before collision of A with B
vB = v
1 m + 4m velocity of A = + v
3 velocity of B = zero
=− v
5 Therefore, after collision of A with B
3 2
and vC = − v + v = v velocity of A = zero
1 5 5
Before second collision of B with A velocity of B = + v
A B C After collision of B with wall
2v velocity of B = − v
4m m 4m
3v 5
Rest 5 Before collision of B with A
m − 4m  3  velocity of A = zero
vB =  − v
2 m + 4m  5  velocity of B = − v
9 Therefore, after collision of B with A
= v
25 velocity of A = − v
9 3 6
vA = v +  − v = − v velocity of B = zero
2 25  5  25
Now, no more collision will take place as A
After second collision : of B with A
will move towards left with speed v
A B C
leaving B at rest to its right side.
6v 9v 2v
4m 4m 4m
25 25 5 Thus, we see that speeds of the balls
remains unchanged after all the possible
As velocity of C is greater than that of B,
collisions have taken place.
no further collision will take place.
168 | Mechanics-1
Introductory Exercise 8.7
1. Applying law of conservation of ∴ vmin = 2 m / s 
momentum vmax = 2 2 m / s
2v
v Rest v' 3 3. A ( m) hitting B (2m)
B
m m m m
A B A B 2m
v
2v A
mv = mv ′ + m ⋅ m
3
v r
i.e., v′ =
3
velocity of separation
e=
velocity of approach
2v 2v v A collision is elastic
− v′ −
= 3 = 3 3 =1 vB =
2 ( m) 2
v= v
v−0 v 3 2m + m 3
 1 + e  1 − e m − 2m v
2. v ′1 =   v and v ′2 =  v and vA = v=−
 2   2  2m + m 3
v Rest v2' v1' ∴ Velocity of B w.r.t. A = vB − v A
2 v
m m m m
= v −  −  = v
A B A B 3  3
Before collision After collision
∴ Next collision between the balls will
KE after collision = 0.2 J (given) take place after time
1 2 1 2 2π r
∴ mv2′ + mv ′1 = 0.2 t=
2 2 v
or v ′22 + v ′21 = 4 (Q m = 0.1 kg) 4. As collision is elastic
2 2 v2'
 1 − e 2  1 + e 2 p
Rest
v'1
or   v +  v =4
 2   2  m m m m
v2
or (2 + 2e2 ) = 4 A B A B
4 Before collision After collision
Extreme cases  1 − e p
v 2′ =  
If collision is perfectly in elastic i.e., e = 0  2  m
v2 Now, Momentum of A before impact
(2) = 4
4 − Impulse given by B on A
i.e., v = 2 2 m/s = Momentum of A after impact.
If collision is perfectly elastic i.e., e = 1 ∴ p − J = m v ′2
v2  1 − e p
(2 + 2) = 4 or p− J = m 
4  2  m
2J
i.e., v = 2 m/s or e= −1
p
Centre of Mass, Conservation of Linear Momentum, Impulse and Collision | 169
Introductory Exercise 8.8
1. v0 sin α = v cos (α + β) V −0
=
v0 v−0
sin
α
v0 v0 s
V
co
s α v0 in ( =
α+
β)
v
m
=
α M
β 4.
v co α A
s (α Rest
+ β) u
α P ∠PRX = 30°
∠QRX = 30°
R
X
v0 cos α = sin (α + β) (Impact being elastic) C
∴ tan α = cot(α + β) Rest
π Q
or cot − α = cot(α + β) B
2
π As the balls are of same size, the centres
or −α =α +β
2 of the balls P, Q and R will be at vertices
π
or β = − 2α of an equilateral triangle when ball C just
2 strikes balls A and B symmetrically and
2. Speed after n impacts as such the balls A and B will follow the
Speed before first impact = u ( = 2 gh) path as shown below
Speed before one impact = e u v
A
Speed after 2 impacts = e ( eu) = e2u
........ ......... ........ ........ ........ ......... Rest
Speed after n impacts = enu
30°
Height ( H ) upto which the ball rebounds X Angle OA
after nth rebound 30°
( enu)2 C
H=
2g B
2n 2
e u
= v
2g
= e2 n ⋅ h Applying law of conservation of
momentum
3. mv + 0 = 0 + MV mu = Resultant momentum of A and B
v V balls along the axis of X.
Rest Rest
m M m M or mu = 2 mv cos 30°
A B A B 3
or u = 2v
Before collision After collision 2
m or u=v 3
⇒ V = v
M
Velocity of separation As the ball C will strike ball A (and as well
e= as ball B) with velocity ucos 30°
Velocity of approach
170 | Mechanics-1
Velocity of approach of ball C towards ball A ∴ after impact = 2 ^j
= ucos 30° − 0
= ucos 30° ∴ Velocity after impact = − ^i + 2 ^j
=u 3 6. At A, u|| = u cos θ
Velocity of separation of ball A away from u⊥ = u sin θ
ball C = v u sin θ
v v
∴ e= e u sin θ
3 H1
u A θ
2 u cos θ T1 B u sin θ H2
T2
v
= R1 R2
3
(v 3 )
2 ∴ At B
2
= u|| = u cos θ
3
u⊥ = e u sin θ
5. x-component of velocity 2u sin θ
T1 g 1
∴ = =
45° T2 2 e u sin θ e
45° g
 2u⊥ 
Using T = g 
 
2 (u sin θ) (u cos θ)
2 ^j 45° v = 2i^ + 2j^
R1 2  2 u⊥ u||
= Using R =
X R2 2 ( eu sin θ) u cos θ  g 
2 ^i
g
before impact = 2 ^i 1
=
e
∴ after impact = − e 2 ^i (u sin θ)2
1 u2 
= − × 2 ^i H1
=
2g 
Using H = ⊥ 
2 H2 ( e u sin θ) 2  2g 

= − ^i 2g
y-component of velocity 1
= 2
before impact = 2 ^j e

AIEEE Corner
Subjective Questions (Level 1)
1. 1⋅0 + 2⋅1 + 3 ⋅1 + 4 ⋅0
xCM =
y 1+2+3 + 4
D 4kg 1m C 5
3kg = m
P 10
(CM) 1⋅0 + 2⋅0 + 3 ⋅1 + 4 ⋅1
yCM =
1m 1m 1+2+3 + 4
7
= m
B 2kg 10
x
A 1kg 1m
Centre of Mass, Conservation of Linear Momentum, Impulse and Collision | 171
2 2
5 7 4
AP2 = x2CM + y2CM =   +   V ⋅ 0 +  − πa 3 b
 10  10  3 
4. xCM =
4 3
= 0.74 m 2 V − πa
A x + A2 x2 3
2. xCM = 1 1 4
A1 + A2 − π a 3b
= 3
4 4
A1 = a2 πR 3 − πa 3
a2 3 3
A2 = π
a
4  a3 
O =− 3 3
b
a R − a 
2
By symmetry yCM = 0
a a
5. In Fig. 1, C is CM.
πa2 l
a2 ⋅ 0 + ⋅ ( a) Fig. 1
a
= 4 Fig. 2 l2
2 m1 C m2
πa
a2 + l1 b
4
π m1 m2
= a C'
4+π
∴ m1a = m2 ( l − a) …(i)
3. Let A = area of rectangle
Y In Fig. 2, C′ is CM.
m1( a + b − l1) = m2 ( l − a + l2 − b) …(ii)
Substituting Eq. (i) in Eq. (ii),
(a4 , b4 ) m1( b − l1) = m2 ( l2 − b)
b O or m1b − m1l1 = m2 l2 − m2 b
X
or ( m1 + m2 ) b = m1l1 + m2 l2
m1l1 + m2 l2
or b=
a
m1 + m2

 A a
A ⋅ 0 + −  6. xCM =
∫ x dm
xCM =
 4 4 ∫ dm
 A dx
A + − 
 4 O x
A 4 a
=− ⋅ ⋅ where, dm = mass of element of length dx.
4 3A 4
=−
a
=
∫ x ρ dx A
12 ∫ ρ dx A
 A b  x2 
A ⋅ 0 + −  l
 4 4 x ρ0 2  dx
∫0
yCM =  l  x2
 A = (Qρ = ρ0 )
A + −  l  x2  l2
 4
∫ 0 ρ0 l2  ax
b
=− l
12 ∫0 x 3dx
 a b = l
Centre of mass  − ,−  x2 dx
 12 12 ∫0
172 | Mechanics-1
l
 3 x4  New position of CM
= 3⋅  → →
x 4 0 → r1 (1) + r2 (2)
R=
3 1+2
= l
4
10 ^i + 15 ^j + 60 ^i + 90 ^j
1 × 10 + 2 × 12 =
7. xCM = 3
1+2
m 10 m 10 m 70 ^i + 105 ^j
= m
6ms–1 1 kg 2 kg 4ms–1 + ive 3

34 10.
= m 6 ^j ms–1
3 m kg 0.10 kg
1 × ( − 6) + 2 × ( + 4) 2 O 3 m CM
vCM = = + ms −1
1+2 3 12 m
m ⋅ 0 + 0 ⋅ 1 × 12
x′ CM (new position of CM) (a) 3 =
34 2 m + 0. 1
= m + 2 s  ms −1
3 3  ⇒ m = 0.3 kg
38 (b) Momentum of system = Momentum of CM
= m = 12.67 m
3
1 × 2 ( + 2) + 2 × ( − 1) = ( m + 0. 1) kg × 6 ^j ms −1
8. vCM =
1+2 = 2.4 ^j kg ms −1
1 kg 2 kg → →
→ v (0.3) + 0 (0.10)
2 ms –1
1 ms –1 + ive (c) v CM = m
0.3 + 0.1
= 0 ms −2 → 4→
v m = v CM
∴ Displacement of CM in 1 s = 0 m. 4
→ ^ 4
9. Acceleration ( a ) = − 10 j ms −2 = × 6 j^ ms −1
3
→ → →
v=u+at
= 8 ^j m s −1
(1)(0) + (2)(10$i + 10 $j) m
= − 10 $j 11. A • t = 0s
3
2m
20 10 $
=  $i − j m/s B • ( t = 100 ms)
3 3 
→ Position of 1st particle ( A) at t = 300 ms
New position vector ( r1) of particle A 1
s1 = × 10 × (300 × 10−3)2
→ → → 1→ 2
( r1) = s 0 + u t + a t2
2 = 0.45 m
→ 1
= (10 ^i + 20 ^j) + 0 + ( − 10 ^j) ⋅ 12 Position of 2nd particle ( B) at t = 300 ms
2
(B is at the position of A at t = 100 ms)
= 10 ^i + 15 ^j m 1

s2 = × 100 × (200 × 10−3)2
New position vector ( r2 ) of particle B 2
→ 1 = 0.20 m
r2 = (20 ^i + 40 ^j) + (10 ^i + 10 ^j) ⋅ 1 + ( − 10 ^j) ⋅ 12 2 m × 0.2 + m × 0.45
2 ∴ Position of CM =
^ ^ 2m + m
= 30 i + 45 j
= 28.3 cm
Centre of Mass, Conservation of Linear Momentum, Impulse and Collision | 173
Velocity of 1st particle ( A) at t = 300 ms 14. Let x = displacement of wedge (30 kg)
v1 = 10 × 300 × 10−3 towards right.
= 3 ms −1 ∴ Displacement of block A towards right
(along x-axis) when it arrives at the
Velocity of 2nd particle at t = 300 ms
bottom of the wedge
v2 = 10 × 200 × 10−3 ms −1
A
= 2 ms −1 5 kg
2m × 2 + m × 3
∴ vCM =
2m + m
7
= = 2.33 ms −1
3 30° B (50 kg)
X
m A ⋅ 0 + mB ⋅ 80 Q R x
12. 24 =
m A + mB = QR − x
or 24 ( m A + 0.6) = 80 × 0.6 = 0.5 − x
or m A = 1.4 kg Now, as net force on the system (wedge +
∴ Total mass of system = 1.4 kg + 0.6 kg block) along x-axis is zero, the position of
CM of the system, along x-axis, will not
= 2.0 kg change
vCM = 6.0 t2 ^j 1

∴ aCM = 12 t ^j ms −2
Net force acting on system (at t = 3 s)
A
= Total mass of system × (aCOM at t = 3 s)
= 2.0 kg × 36 ^j ms −2 ∴ 5 (0.5 − x) = 30x
^ 0.5
= 72 N j i.e., x= m
7
13. = 71.4 mm
L
15. As no external force acts on the system,
O x the velocity of CM will be zero.
dx
A B
xCM =
∫ x dm
FA FB
∫ dm A B
L
∫ 0 x ( Ax dx) i.e.,
m Av A + mBvB
=0
= L
Ax dx m A + mB
∫o vA m 2
L i.e., =− B =−
∫0 x2 dx vB mA 1
= L
x dx (a) ∴ Ratio of speeds = 2
∫0
p m v m  m 
L3 / 3 (b) A = A A = A  − B  = − 1
= pB mBvB mB  m A 
L2 / 2
2 KA p2 / 2 m A p2 m 2
= L (c) = 2A = 2A × B =
3 KB pB / 2 mB pB m A 1
174 | Mechanics-1
16. While man travels from P to Q
m
or V =− v
( M + m)
CM Negative sign shows that the balloon will
P m Q move downwards.
3 v 
m − v + mv
 2  3v
vCM = =
m+m 4 M
v
L + ive
Displacement of CM (along horizontal) =
.
2
∴ Time taken by man to reach point Q
starting from point P
L /2 2L v m
tPQ = =
3v/ 4 3v
(b) If the man slops climbing i.e., v = 0,
While man travels from Q to P then according to the above relation
3v the value of v will also be zero. Thus,
m  − − v + mv the balloon will also slop.
 2 
vCM =
m+m 18.
3v m vi ⇒
=− M v2 v3
4
1
Displacement of CM = − mv1 = Mv1 + mv3 …(i)
2
∴ Time taken by man to reach point P m = 4 × 10−3 kg
starting from point Q M = 1 kg
−L / 2 2 L v3 = 100 m/s
t QP = =
− 3v / 4 3 v v1 = 500 m/s
∴ Total time = tPQ + t QP Find v2
2L 2L (a) 0 = v2 − 2as = v22 − 2(µg) s
2

= + v2
3v 3v ∴ µ= 2
2 gs
4L
= Here s = 0.30 m
3v
4L (b) Decrease in kinetic energy of bullet
∴ Net displacement of trolley = ×v 1
3v = m(v12 − v23 )
2
4L 1
= (c) KE of block = mv22
3 2
17. (a) While the man climbs in the rope, no 19. m v1 v2
m1 µ = 0.5
extra external net force acts on the system
m2 ⇒
(balloon along with rope + man). Force
applied by man to gain velocity to climb v3
up is an internal force and as such the
velocity of the CM of the system will
remain stationary. ⇒

mv + ( M + m) V = 0
( m + m1)v2 = mv1
Centre of Mass, Conservation of Linear Momentum, Impulse and Collision | 175
mv1 2
2 m gh
∴ v2 = …(i) or V2 =
m + m1 M ( M + m)
mv1
Common velocity v3 = …(ii) 2 m2 g l (1 − cos α)
m + m1 + m2 or V2 =
M ( M + m)
For m1 : v23 = v22 − 2a1s1 and Find s1 α
2 m2 g l 2 sin2
For m2 : v23 = 2a2 s2 and Find s2 or 2
V = 2
µm1 g M ( M + m)
Here : a1 = = µg
m1 α gl
or V = 2 m sin
µm1 g 2 M ( M + m)
and a2 =
m2
21. Let the track shift by x (to the right) when
20. Let v = Velocity of pendulum bob when the the cylinder reaches the bottom of the
pendulum is vertical track.
∴ x′ + x = R − r
α or x ′ = ( R − r) − x
(x′ = horizontal displacement of cylinder
w.r.t. ground)
PE = mgh As no force would be acting along
h horizontal direction, for no shift in CM
along horizontal. We would have
M mx ′ − Mx = 0
or m [( R − r) − x ] − Mx = 0
or ( M + m) x = m ( R − r)
m ( R − r)
⇒ x=
M+m
KE = 1 mv
2
2 Now, as the PE of the cylinder would
v change into the kinetic energies of the
KE = 1 (R – r)
2
m
V

V = Velocity of wagon when pendulum is


vertical
x x' M
∴ mv = MV x x
M
or v= V
m
KE of wagon + KE of bob = PE of bob
1 1
MV 2 + mv2 = mgh O
2 2
2 cylinder and the track we have,
1 1 M  1 1
or MV 2 + m  V  = mgh mg ( R − r) = mv2 + MV 2 …(i)
2 2 m  2 2
1  M
or MV 2 1 + = mgh where, v = velocity of cylinder
2  m  and V = velocity of track.
1
or MV 2 ( M + m) = m2 gh (when cylinder just reaches bottom of the
2 track)
176 | Mechanics-1

Applying law of conservation of p f = − 50 × 10−3 × 2 kg ms −1
momentum

mv + MV = 0 ∴ |p f |= 50 × 10−3 × 2 kg ms −1
M
or v=− V ∴ Change in the magnitude of the
m → →
momentum of the wall = |p f |− |p i |
Thus, Eq. (i) becomes
1  M 
2
1 =0
mg ( R − r) = m  − V  + MV 2
2  m  2 23. m0 = 40 kg
1  M  m = ( 40 + 160) kg
or mg( R − r) = MV 2  + 1
2 m  = 200 kg
2 m2
g ( R − r) vi = 2 km/s = 2 × 103 kg −1
or V2 =
M ( M + m) Rate of consumption of fuel = 4 kgs −1
2 g ( R − r) ∴ Time (t) required to completely brunt out
or V =m 160 kg
M ( M + m) of the fuel = = 40 s.
4 kgs −1
22. No change in momentum of ball and also
Ultimate vertical speed gained by the
that of wall along horizontal direction.
rocket
√2 m/s m
= − g t + vi ln
2 m/s m0
50 g √2 m/s
√2 m/s 200
2 m/s = − 10 × 40 + 2 × 103 × ln
40
45

+ ive
°

45° 45° = − 400 + 3218 = 2818 m/s


√2 m/s
= 2.82 kms −1
Wall
24. When y length of rope has fallen on table
Along perpendicular direction : top
Momentum of ball after reflection A

→ y
( p f ) = − 50 × 10−3 2 kg ms −1
Momentum of ball before reflection L A v = √2gy

( p i ) = + 50 × 10−3 2 kg ms −1
→ → →
∴ ∆p = p f − p i
= − 100 × 10−3 2 kg ms −1
= − 0.14 kg ms −1 dm
Fth = vrel
→ dt
i.e., |∆p|= 0.14 kg ms −1 dm dy
=v ⋅
Momentum of wall (when ball strikes dy dt
wall) dm M
→ = v2 = v2
p i = 50 × 10−3 × 2 kg ms −1 dy L
→ M
∴ |p i|= 50 × 10−3 × 2 kg ms −1 W = g
L
Momentum of wall (when ball rebounds Fnet = W + Fth
from the wall)
Centre of Mass, Conservation of Linear Momentum, Impulse and Collision | 177
M M ma = Fth − mg
∴ Fnet = yg + v2 26.
L L or ma = Fth
M M
= yg + 2 gy (neglecting mg as compare to Fth )
L L
M  dm 
or ma = u  − 
=3 yg  dt 
L
dm a
= Weight due to 3 y length of the rope. or = − dt
m u
25. Horizontal velocity of incoming sand dm a
y or ∫ m = ∫ − u dt
a
or log e m = − t + K
Fth v = 2 ms–1 x u
O i
Now, as t = 0, m = m0 (given)
log e m0 = K
a
Thus, log e m = − t + log e m0
u
→ → m a
v=0 or log e =− t
m0 u
Horizontal velocity of the conveyer belt −
a
t
→ ^ or m = m0 e u
= ( v) (say) = v i
→ → → 27. u = 100 ms −1, v = 0 ms −1, s = 6 cm
∴ u rel = v1 − v2
v2 = u2 + 2as
→ ^
= 0 − (v i) u2
⇒ a=−
2s
= − v ^i
(a) Now, v = u + at
→ dm u u 2s
∴ Fth = v rel ∴ t=− =− 2 =
dt a u u
 dm  ^ 2s
= − v ⋅ i
 dt  2 × 6 × 10−2
dm = = 1.2 ms
As > 0, the falling sand particles exert 100
dt (b) Impulse on log = − Change in
thrust force which decelerates the
momentum of bullet
conveyer belt.
= − m (0 − u)
Force required to keep the belt moving
→ →
= mu
F = − F th = 5 × 10−3 × 100
dm ^ = 0.5 Ns
=v i
dt (c) Average force experienced by the log
= (2 ms −1 × 5 kgs −1) ^i Impulse 0.5
= =
time 1.2 × 10−3
= 10N ^i
= 416.67 N
Power delivered by motor to drive belt at
28. Let us consider right direction as positive.
2 m/s
→ → Impulse of the spring on the block
= |F||v|= 10 N × 2 ms −1
= Change in momentum of block
= 20 W = m (v − u)
178 | Mechanics-1
= 3 [( + 40) − ( − 50)] 32. m1 = 1 kg and m2 = 1 kg (given)
= + 270 Ns v2 = + 4ms–1 v1 = – 6ms–1 v2' v1'
= 270 Ns (to the right)
Impulse m2 1 kg m1 2 kg m2 m1
Average force on block = + ive
∆t A B A B
270 Before collision After collision
=
0.02
 m − m2   2m2 
= 13500 N v1 ′ =  1  v1 +   v2
 m1 + m2   m1 + m2 
= 13.5 kN (to the right)
 2 − 1  2 × 1
29.
→ →
p = m v = 2 (2t i − 4 j)^ ^ =  ( − 6) +   ( 4)
 2 + 1  2 + 1
dp
→ 8
∴ = 4 ^i = −2+
dt 3
2
→ → = + ms −1
or dI = 4 ^i dt (where, I = Impulse) 3
2
→ 2 = ms −1 (in + ive x direction)
or ∫ dI = ∫0 4 ^i dt 3
 m − m1   2m1 
→ v2 ′ =  2  v2 +   v1
or I = 4 ^i (2 − 0) m
 1 + m 2  m1 + m2 
 1 − 2  2 × 2
= 8 ^i kg-ms −1 =  ( +4) +   ( − 6)
 2 + 1  2 + 1
30. m ∆v = F ∆t
4
or m ∆v = Area under F-t graph = − −8
3
16 + 8 28 28
or m ∆v = × 20000 =− ms −1 = ms −1
2 3 3
or m ∆v = 240000 (in − ive x direction).
240000
∴ ∆v = = 200 m/s
1200 1+ e 1− e
33. v1 ′ = v2 and v2 ′ = v2
v − u = 200 2 2
v2 v1 = 0 v2' v1' = 2v2'
∴ v = 200 ms −1 as u = 0
31. v1 = v2 ′ = − 2 ms −1 m 1 m 2 m 1 m 2
−1
v2 = v1 ′ = + 3 ms Rest

v2 v1 v2' = – 2ms–1 v1' = + 3ms–1 v1 ′ 1 + e


⇒ =
v2 ′ 1 − e
m m m m + ive 1+ e
A B A B or 2= (Qv1 ′ = 2 v2 ′)
1− e
(When two bodies of equal masses collide or 2 − 2e = 1 + e
elastically they interchange their 1
or e=
velocities) 3
Centre of Mass, Conservation of Linear Momentum, Impulse and Collision | 179
34. mv = mv1 + mv2 1
or e2 =
v v1 v2 2
1
m m m m or e=
Rest
2
Before collision After collision  2m2 
35. v1 ′ =   v2
 m1 + m2 
⇒ v = v1 + v2
1 1 v2 = 2 ms–1 v1 = 0 ms–1 v2' v1'
mv12 + mv22
2 2 3
Also, = A B A B
1 2 4
mv m2 = 3 kg m1 = 2 kg
2
3 2×3
or (v12 + v22 ) = v2 …(i) = ×2
4 2+3
3
or (v1 + v2 ) − 2v1v2 = v2
2
12
4 = = 2.4 ms −1
2 3 2 5
or v − 2 v1v2 = v
4  m − m1 
1 2 and v2 ′ =  2  v2
or 2 v1v2 = v  m1 + m2 
4
3−2
Velocity of separation = ×2
e= 2+3
Velocity of approach
v2 − v1 2
or e= = = 0.4 ms −1
v−0 5
v2 − v1 Distance between blocks when they stop
e=
v sliding
2
 v − v1  v ′21 v ′22
e2 =  2  = −
 v  2µ k g 2µ k g
v22 + v12 − 2 v1v2 1
or e2 = = (v ′21 − v ′22 )
v2 2µ k g
(v + v2 )2 − 4 v1v2 1
or e2 = 1 = [(2.4)2 − (0.4)2 ]
v2 2 × 0.3 × 10
v2
v2 − 2.8
2 = = 0.933 m
or e2 = 3
v2
180 | Mechanics-1
Objective Questions (Level 1)
Single Correct Option
1. Momentum remains conserved. Decrease Velocity of separation
6. =e
in momentum of the ball is transferred to Velocity of approach
sand while KE does not remain conserved As in an elastic collision e < 1
as it gets used up in doing work against
Velocity of separation < velocity of
friction.
net approach
2. Fext = M × aCM
net (when e = 0, the velocity of separation in
∴ If Fext = 0, aCM = 0
zero and the colliding bodies do not
d
i.e., v =0 separate from each other.)
dt CM
Further, whether the collision in elastic or
or vCM = constant inelastic the law of conservation of
Option (a) is correct. momentum always hold gord.
3. The forces acting on the blocks would be ∴ Option (d) is correct.
→ →
equal and opposite as per Newton's 3rd 7. p = M v CM , Option (a) is correct.
law of motion. Acceleration of the blocks → → → →
will depend upon their masses as per p = p1 + p2 + p3 + ....,
Newton’s 2nd law of motion. Accelerations Option (b) is correct.
being different velocities will be unequal.
Further, we define momentum for every
M1 M2 type of motion.
F F ∴ Option (d) is correct.
Option (c) is correct. 8. Let us consider a system of two masses as
4. While colliding the balls will apply equal shown in figure.
→ → →
and opposite impulsive force on each v1 vCM v2
other. Impulsive forces will change the
momentum of the balls but the total m1 CM m2
momentum of the system of 2 balls will
Momentum of system about CM
remain conserved impulsive forces being → → → →
internal ones. Change in momentum of = m1 ( v1 − v CM ) + m2 ( v2 − v CM )
→ → →
the system will definitely be due to = m1 v1 + m2 v2 − ( m1 + m2 ) v CM
external gravitational forces on the balls → →
but as the time of impact shall be very less = ( m1 + m2 ) v CM − ( m1 + m2 ) v CM

the impulsive force will over shadow the =0
weak gravitational force.
Option (c) is correct.
5. External force acting on the cannon shell
9. Option (a) If collision is inelastic.
before explosion is the gravitational force. A
m
Now, as no extra net external force would
be act on the shell during collision the v
momentum of the system shall remain
conserved and the CM of the system (now
broken in pieces) will also keep on v
following the path which the shell would m
v
m
have followed had the explosion not taken B C
place. Further, as the explosion would Option (b) If collision is perfectly inelastic
never be super-elastic, the KE of the
Option (c) If the dimensions of the
system can’t increase after explosion. particles → 0
Option (d) is correct. ∴ Option (d) would be the answer.
Centre of Mass, Conservation of Linear Momentum, Impulse and Collision | 181
10. F ∆t = m ∆v 14. Horizontal velocity of the leaving coal :
m ∆v
⇒ F= system
v
∆t
5 × (65 − 15)
= x
2 O ^i
= 125 N

11. mc × d1 = mo × ( d − d1) v1 = + v ^i
d
mc CM mo Horizontal velocity of the system
O →
C d1 v2 = + v ^i
mo → → → →
⇒ d1 = d ∴ U rel = v1 − v2 = 01
mo + mc
→ → dm →
8 ∴ Fth = U rel =0
= d dt
8+6
4 As, the leaving coal does not exert any
= d thrust force on the wagon, the speed of the
7 wagon won’t change.
4
= × (1.2 × 10−10 m) Option (a) is correct.
7
= 0.64 × 10−10 m 15. If n be the number of bullet shots per
second
12. M × 0 = mv + ( M − m) V
v V
n × [change in momentum per second] ≤ F
Rest
40
i.e., n × (1200 − 0) ≤ 144
M m M–m 100 
144
9 kg 3 kg 6 kg or n≤ or n ≤ 3
48
m
⇒ V =− v ∴ Option (a) is correct.
( m − m)
16. Change in momentum along x-axis
3 v sin θ v
=− × 16
9−3
– θ Q v cos θ
x
= − 8 ms −1 P v cos θ θ
1 +
∴ KE of 6 kg mass = × 6 × ( − 8)2 y
v
2 v cos θ

= 192 J = m (v cos θ − v cos θ) = 0


2 m2
13. v1 ′ = v2 ∴ Net change in momentum
m1 + m2
= Change in momentum along y-axis
v2 v1 = 0 v2' v1'
= m[( + v sin θ) − ( − v sin θ)]
m2 m1 m2 m1 = 2 mv sin θ
Before collision After collision = mv 2 (as θ = 45°)
= 2 v2 (as m1 < < m2 ) Option (a) is correct.
Option (b) is correct. 17. Velocity of ball before first impact i.e.,
when it reaches point Q of the horizontal
plane
182 | Mechanics-1
P u=0 50 kg
450 kg
O X
CM (boat)
10 m
50 kg
+ eg
2nd 450 kg
Q 1st x X
– eg CM (boat)

50 × 10 + 450 × 5 50 × ( x) + 450 × ( x + 5)
v = 0 + ( g) ⋅ 1 = g ∴ =
50 + 450 50 + 450
∴ Velocity of ball after 1st impact
(Initially) (Finally)
= ev = eg
⇒ x = 1m
time elapsed between 1st and 2nd impact
with the horizontal plane Option (b) is correct.
( + e g) = ( + e g) + ( − g) t 20. As discussed in the answer to previous
question no. 19.
⇒ t = 2e
M
4 2
= s (as, e = L)
3 3 O
M/3
Option (c) is correct. X
L Smooth

18. xCM =
∫ x dm = ∫ x m dx M

∫ dm ∫ m dx O
x M/3
dx X
L Smooth

0 x
x M L M  L
L Mx +  x +  ML +  
3 2 3  2
=
(where m = mass per unit length) M M
M+ M+
Ax2 3 3
∫ x L dx  Ax2  (Initially) (Finally)
= 2 Q m =   M
Ax  L  ⇒ x M + = ML
∫ L dx  3 
3
L x= L
∫0 x 3dx 4
= L 3
x2 dx i.e., the distance that plank moves = L
∫0 4
L4 3 ∴ The distance that the man moves
= × 3 3
4 L =L− L
3 4
= L L
4 =
4
Option (a) is correct.
Option (b) is correct.
19. As there is no net external force acting on
the system in the horizontal direction, the 21.
v u
CM of the system shall not shift along Rest Rest
x-axis. m 3m m 3m
Before collision After collision

mv + 3 m ⋅ 0 = m ⋅ 0 + 3 mu
Centre of Mass, Conservation of Linear Momentum, Impulse and Collision | 183
v
∴ u= Impulse on ball
3
Velocity of separation = Change in momentum of ball
e=
9 4
Velocity of approach =  − mv0  −  + mv0 
u−0 u  20   5 
= =
v−0 v 5
= − mv0
1 4
=
3 25. If a ball dropped from height h rebounds
Option (d) is correct. to a height h′, then speed of ball just
before 1st impact, u = 2 gh
22. Change in momentum of A or B = mu
u u Just after 1st impact u′ = 2 gh′
Rest Rest
u′ h′ 64
m m m m ∴ e= = =
u h 100
Before collision After collision
= 0.8
(As collision is elastic)
i.e., h′ = e2 h
Impulse = Change in momentum
Ft0 Height attained after nth impact
= mu
2 = e2 n h
2 mu
⇒ F= = (0.8)2 n ⋅ 1 (as h = 1 m)
t0 2n
= (0.8)
Option (b) is correct.
Option (d) is correct.
23. Acceleration of block A
m m 26. Momentum of car (+ block) before
F' F' F
A B throwing block
= Momentum of car after throwing block
F′
a0 = + Momentum of block
m
→ ^
⇒ F ′ = ma0 500 × 1 ^i = (500 − 25) v + 25 (20 k)
Acceleration of block B : → ^
F − F′ or 475 v = 500 ( ^i − k)
aB =
m → 20 ^ ^
or v= ( i − k)
F − ma0 F 19
= = − a0
m m Option (c) is correct.
24. 27. While force is increasing with time
v0 cos 37° = v0 4
v0 5 F = kt (where k is + ive constant)
or ma = kt
+
37° X 5 dv k
53° or = t
3 dt m
53°
Y 37° k t2
3 v cos 53° ∴ v= +C
4 m 2
4 0
= 9 v0 k t2
20 3v or v= ⋅
4 0 m 2
(If at t = 0, v = 0)
Option (c) is correct.
184 | Mechanics-1
Thus, graph between v and t would be 30. As no external force is acting along the
horizontal direction on the system (wedge
v0 + block). The CM of the system shall not
change along horizontal when the block
Graph would be moves over the wedge but would change
parabolic in along vertical as net force (= gravitational
nature
v force) is acting on the block.
Further, as no non-conservative force is
acting on the system, its total energy will
t not change.
t0
Option (d) is correct.
While force is decreasing with time. 31.
F = − kt (where k is + ive constant)
k t2
⇒ v=− + C′
m 2
At t = 0, v = v0
k t20 v'
∴ C ′ = v0 +
m 2 m m m
k 2 M M Rest
Thus, v = v0 + ( t0 − t2 ) v h
2m Rest
Before collision Just after Final position
Thus, graph between v and t would be collision

mv = ( M + m) v ′ …(i)
Parabola
v From final position, v ′ = 2 gh
From Eq. (i),
v0 m
or v = 2 gh
M+m
 M
or v = 1 +  2 gh
t0 t  m

∴ Option (a) is correct. Option (c) is correct.


A x + A2 x2 32. As no net extra external force is acting on
28. xCM = 1 1
A1 + A2 the system the CM of the gun and the
[ π (3 R)2 − π R2 ] x1 + [ − πR2 ] 2R bullet system remains at rest. The force
0= exerted by the trigger of the gun on the
π (3 R)2 − π R2 bullet is an interval one.
or 8 π R2 x1 = 2πR2 R or 8 x1 = 2 R 33. m1 g − T = m1a
R
or x1 =
4
Option (c) is correct.
A x + A2 x2 + A3x3
29. xCM = 1 1
A1 + A2 + A3
T T
∴ 0 = [ π ( 4 R)2 − π R2 − π R2 ] x1
+ [ − π R2 ] 3 R + [ − π R2 ] ⋅ 0 T T
or 14 x1 = 3 R
a
3
or x1 = R
14
m2g m1g
Option (d) is correct.
Centre of Mass, Conservation of Linear Momentum, Impulse and Collision | 185
T − m2 g = m2 a 36. Velocity of separation would be zero as the
∴ ( m1 − m2 ) g = ( m1 + m2 ) a collision is completely inelastic.
m − m2
or a= 1 g A
m1 + m2 v cos θ
2
m a + m2 ( − a)
aCM = 1 v
m1 + m2 θ
v sin
m1 − m2 2 θ/2 2m
= a θ
V
m1 + m2 v sin θ/2
2
2
 m − m2  v
= 1  g
 m1 + m2  v cos θ
B 2
Option (b) is correct.
m
34. v = u − gt + vi ln 0 Velocity of approach = Velocity of A w.r.t.
m
m0 B or velocity of B w.r.t. A
or v = u + v2 ln θ θ
m =  + v sin  −  − v sin 

 2  2
(neglecting gravity as given)
θ
m = 2 v sin
or v = v2 ln 0 (Taking u = 0) 2
m
m0 Common velocity ( V ) after collision
or v = v ln (as vi = v) θ θ
m mv cos + mv cos = 2 mV
m 2 2
∴ log e 0 = 1 θ
m ∴ V = v cos
m0 1 2
⇒ = e = 2.718
m Option (d) is correct.
Option (a) is correct. 37. v sin 53 ° = u sin 37 °
35. Velocity of water after striking the plate u sin 37°
v cos 53°
would be almost zero as then it flows u cos 37° u v
parallel to the plate.
v sin 53°
53° 37°
Floor
4 3
Jet of water v× =u×
5 5
Water flow 3
i.e., v= u
4
Impulse exerted on floor
Force exerted on plate
= − [Change in momentum of ball]
= − [Rate of change of momentum of
water] = − [( − mv cos 53 ° ) − ( + mu cos 37 ° )]
 m ⋅ 0 − mu = m [v cos 53 ° + u cos 37 ° ]
=− 
 t 3 3 4
mu m = m u× +u× 
= = ⋅u  4 5 5 
t t 5
kg = mu
= 0.5 × 1ms −1 4
s 5
= × 1 × 5 = 6.25 Ns
= 0.5 N 4
Option (c) is correct.
186 | Mechanics-1
38. As the CM will not change along x-axis, x = Displacement of boat
for no net force acting on system (wedge + x ⋅ 40 + (1 + x) 15 = 2 × 40 + 1 × 15
block) along x-axis. i.e., x = 1.46 m
m
(The frictional forces on the boat by the
boy and that by the boy on the boat are
h internal forces).
M a = cot θ
→ → →
h → m v1 + m v2 + m v 3
x' θ 41. v=
X
x a m+m+m
Mg 1 → → →
= [ v1 + v2 + v 3 ]
3
h M 1 ^
= [v0 ^i + ( − 3v0 ^j ) + (5v0 k)]
3
m
v ^
X = 0 [ ^i − 3 ^j + 5 k ]
Mg 3
a
Option (d) is correct.
mx + M ( x + x ′ ) = Ma + Mx ′ 42. The shell explodes at A, the highest point.
m M
∴ mx + Mx = ma or x = a As piece of mass falls very close to
M+m 5
m h cotθ mortar, its velocity after explosion must
=
M+m be − u.
Option (b) is correct. Shell A
u (say)
2(4M ) + 3 (− M ) + 3 (+ M ) M
39. xCM = = 2 cm
4M + (− M ) + (+ M )
r
ta
or

Y Y
M

R/2 R/2
After explosion
M M M 2M path of M/5 part (Velocity of 4M/5 man part
2 cm

2 cm

A D A D+C of shell after explosion)


Shell 3/2 u
After explosion path of
4M 4M/5 part
M M M 5
2 cm

2 cm

B C B
R/2
O X O X D
2 cm 2 cm 2 cm 2 cm
3R/4
2(4M ) + 1(− M ) + 3 (+ M )
yCM = = 2.5 cms
4M + (− M ) + (+ M ) M 4M
Thus, Mu = ( − u) + (v)
5 5
Option (c) is correct. 3
i.e., v= u
40. As explained in the answer to question no. 2
20 and 19. 3
As the velocity has increased times the
40 kg 2
range of this part will be
15 kg
3 R 3R
2m of i.e.,
2 2 4
R 3 R 5R
15 kg ∴ D= + =
2 4 4
x 2m
Option (c) is correct.
Centre of Mass, Conservation of Linear Momentum, Impulse and Collision | 187
m − m1 1 1 1  2
43. v2 ′ = 2 v2 mv22 −  m  − v2  
m2 + m1 2  
= 2 3 
v2 v1 = 0 v2' 1 2
mv2
A m B 2m m 2m 2
m2 = m m1 = 2m
1 1

m − 2m = 2 18
1
= v2
m + 2m 2
1 8
=− v2 =
3 9

Fraction of KE lost by colliding particle A


KE lost
=
Initial KE

JEE Corner
Assertion and Reason
1. To answer this question, let us find the The centre of mass of a uniform plate lies
centre of mass of an L-shaped rigid body at its centre. The CM of a uniform sphere
(as shown in figure) of uniform thickness. is at its centre.
Y Y
L

4L A2 L
L

P
2L A1
X
X L
O
2L 4L
These examples show that assertion is
A x + A2 x2 false.
xCM = 1 1
A1 + A2 It is correct that centre of mass and centre
of gravity of a body coincide if the body is
(6L × 2L) 3 L + ( 4 L × 2L) L
= placed in a uniform gravitational field.
(6L × 2L) + ( 4 L × 2L)
Option (d) is correct.
36L3 + 8 L3 F F − F1
= 2. a1 = 1 , a2 =
12L2 + 8 L2 m 2m
a1 a2
44 L3
= F
20L2 m 2m
F1 F
= 2.2 L
ma1 + 2ma2
Similarly yCM = 2.2 L ∴ aCM =
m + 2m
Thus, we see that CM is lying outside. F1 + ( F − F1)
=
Centre of mass of the rigid body of 3m
uniform thickness as shown in figure F
=
would be at point P which is neither 3m
outside nor inside.
188 | Mechanics-1
As F is constant, the value of aCM will (In magnitude)
remain constant (Reason). (KE) gun Mass of bullet
∴ =
As CM is accelerated the velocity of CM (KE) bullet Mass of gun
will obviously increase (Assertion).
(as given under Reason)
Option (a) is correct.
This is what is given in Assertion.
3. As per assertion if force is applied on a
Thus, both Assertion and Reason are true
system the linear momentum of the and also reason is the correct explanation
system must not remain conserved. But, it of the assertion.
will not be true if we apply two equal and
Option (a) is correct.
opposite external forces on the system as
then net external force on the system will 7. As no net external force is acting there on
be zero and the linear momentum will the system along horizontal direction the
remain conserved as given under Reason momentum of the system remains
which is correct. conserved along horizontal direction but
Option (d) is correct. as gravitational force (a net external force)
acts on block in the vertical direction
4. A rocket moves forward due to the thrust
(downwards) the momentum of the system
force produced on it as per Newton’s 3rd
does not remain constant along vertical
law of motion (as given under Reason)
direction. As overall momentum of the
when gas inside (not the surrounding air
system does not remain constant,
as given under assertion) it is pushed
Assertion is true.
backwards.
Option (d) is correct. As wedge is at rest, Reason is false.
Option (c) is correct.
5. Linear momentum of a system remains
conserved when no net external force acts 8. In any collision, there is not change in the
on the system i.e., only a net external force momentum of the system as given in
on a system can change its linear reason, which is true.
→ → →
momentum. Inside a system internal ∴ ∆p1 + ∆p2 = 0
forces are always in pairs and as such → →
can't change linear momentum. i.e., ∆p1 = − ∆p2 …(i)
If two blocks connected by a spring placed ∴ Assertion is false.
on a smooth surface are stretched apart
Option (d) is correct.
the internal restoring forces acting on
blocks will definitely increase the KE of 9. Reason is true as explained in the answer
the system but this argument under to question no. 6 and also in Eq. (i) in the
Reason has nothing to do with the nothing answer to question no. 8.
under Assertion that internal forces can't As KE is inversely proportional to mass,
change linear momentum. K
the KE of the block of man 2m will be
Option (b) is correct. 2
p2 when the KE of the block of mass m is K .
6. KE =
2m ∴ Assertion is true.
1 Further, as Reason is the correct
i.e., KE ∝ (if momentum p is constant).
m explanation of the Assertion. Option
would be (a).
∴ Nothing under reason is correct.
10. Assertion is false as for example heat
When bullet is fired and comes out of the energy can be given to a system without
gun. any increase in momentum of the system
Linear momentum of gun while KE given to a system increases its
= Linear momentum of bullet = p momentum.
Centre of Mass, Conservation of Linear Momentum, Impulse and Collision | 189
2
 p  x1 x2
 KE =  as given is Reason which is
 2m  m1 CM m2
true.
m1x1 = m2 x2
Option (d) is correct. x2 m1
11. As no external force would be acting on ∴ =
x1 m2
the system of electron and proton along
the line joining electron and proton the ∴ x2 > x1 as m2 > m1
CM of electron and proton will remain at Reason is true.
rest. Therefore, Assertion is false. Option (d) is correct.
Further, as proton is heavier than →
electron the reason is true.
→ dp
14. F = (Newton's second law of motion)
dt
Option (d) is correct. →
m − m2 2m2 → F
12. v1 ′ = 1 v1 + v2 a= (outcome of the above)
m1 + m2 m1 + m2 m
v2 v1 v2' v1' ∴ Reason is true.

m2 m1 m2 m1 First equation tells that if same force F is
A B A B
Before collision After collision applied on different masses the rate of

m2 − m1 2m1 p
and v2 ′ = v2 + v1 change of momentum i.e., d of each mass
m1 + m2 m1 + m2 dt
v2 ′ − v1 ′ will be same. Second equation tells that if

m − m1 − 2m2 2m1 − m1 + m2 same force F is applied on different
= 2 v2 + v1 →
m1 + m2 m1 + m2 masses the a produced in each will be
= − v2 + v1 different.
= − (v2 − v1) ∴ Assertion is true.
i.e., v ′21 = − v21 Further, as Reason is the correct
explanation of the Assertion.
i.e., relative velocity of A w.r.t. B after
collision Option (a) is correct.
= − (relative velocity of A w.r.t. B before 15. Assertion is false as explained in the
collision) answer to question no. 12.
∴ Reason is true and Assertion is false. In every type of collision the linear
Option (d) is correct. momentum of the system remains
conserved. Therefore, Reason is true.
13. As explained in the answer to question no.
Option (d) is correct.
11, the CM of the objects will remain at
rest. Therefore, assertion is false.
190 | Mechanics-1
Objective Questions (Level 2)
Single Correct Option
v1 3
1. m1v1 + m × 0 = m 5 g l + m1 or α = cos −1
3 4
Option (c) is correct.
→ →
3. Here |v2 |= |v1|= v say

→ dp
l v2

dm v1
dm
m1 dθ
m m 2θ
v1 Rest u = √5gl v1/3 θ
R
θ
2 v1 →
v1
m1 ⋅ = m 5g l dθ
3
dl
3 m
∴ v1 = 5g l
2 m1 Net momentum of the two elements as

Option (b) is correct. shown in figure = dp = |dp|
2. mu = 2mv = 2v dm sin θ
l  M
m = 2v   ( R dθ) sin θ
α  π R
2M v
= sin θ dθ
π
2m π /2 2Mv
H p= ∫ sin θ dθ
0 π
2M v
m
=− [cos θ ] π0 / 2
v π
u = √2gl Rest 2M v 2Mv
=− [0 − 1] =
m m π π
u Option (b) is correct.
⇒ v= 1 1
2 4. (2 m) v2 = kx20
2 2
0 = v2 + 2 ( − g) H
2

v2
⇒ H=
2g
(u / 2)2 kx0
= kx0
2g
u2 or v=
k
x0
=
8g 2m
2g l or 2mv = 2 mk x0
or H=
8g
or Fav × ∆t = 2 mk x0
l
or l (1 − cos α) = 2 mk
4 or Fav = x0
1 ∆t
or 1 − cos α =
4 Option (b) is correct.
Centre of Mass, Conservation of Linear Momentum, Impulse and Collision | 191
5. As the collisions of the striker and the 4 p = 1(1 − p)
walls of the carrom are perfectly elastic the 1
⇒ p= m
striker will follow the path OPQROP ... 5
Q Displacement ( x) of bar when pendulum
becomes vertical
x
= sin θ
p
R P
45°
x = p sin θ
1 1
45° = sin 30°= m = 0.1 m
5 10
O A
When the ball reaches the other extreme
Change in KE = work done against friction end the bar will further shift to the left by
1 distance x and as such the net
mv2 − 0 = µ mgs
2 displacement of the bar will be 2x i.e.,
(m = mass of striker, s = displacement of 0.2 m.
the striker) Option (b) is correct.
v2 7.
⇒ s=
2µ g p ep ep e2p e 2p e 3p
(2)2
= = 1m Floor During During During Floor
2 × 0.2 × 10 first Second third
1  1 collision collision collision
PQ = OP = OA 2 =   2= m
2 2 2 Momentum imparted to the floor in
1 1 1st collision = p − ( − ep) = p (1 + e)
OP + PQ = + = 1 m
2 2
2nd collision = ep − ( − e2 p) = ep (1 + e)
∴ Striker will stop at point Q where
1 1 3rd collision = e2 p − ( − e3 p) = e2 p (1 + e)
co-ordinates are  , .
2 2 2 As theoretically there will be infinite
collision, total momentum imparted to
Option (a) is correct. floor
6. As no force is acting on the system along = p (1 + e) + ep (1 + e) + e2 p (1 + e) +.... ∞
horizontal, the CM of the system will not
= p (1 + e) [1 + e + e2 + K ∞ ]
shift horizontally.
1  1 + e
4 kg = p (1 + e) = p 
1− e  1 − e
p x
θ Option (d) is correct.
CM
8. Let F be the frictional force applied by
p
1–

plate when bullet enters into it


1
1 kg ∴ mu2 = Fh …(i)
2
x
If plate was free to move
mu + 0 = ( M + m) u′
x m
∴ u′ = u
M+m
4 kg
1 1
New KE of bullet = mu2 − ( M + m) u′2
2 2
CM
(entering plate)
192 | Mechanics-1
2
1 1  m  ∴ 3 m1 = m2
= mu2 − ( M + m)  u m1 1
2 2 M + m  or =
m2 3
1  m  1 2  M 
= mu2 1 −  = mu   12. As the resultant of the velocities of 1st
2  M + m  2  M + m
and 2nd are just opposite to that of 3rd,
= Fh′ …(ii) the 4th particle will travel in the line in
Dividing Eq. (ii) by Eq. (i), which 3rd is travelling.
h′ M v
=
h M+m
2nd
 M 
i.e., h′ =  h
 M + m
Option (a) is correct.
45° v
9. v2 = 16 ms–1 2
v1 = 2gs = 10 ms–1 4th 1st
v2'
Rest u
m m 3rd
v
A B A B
1+ e Let the velocity of 4th particle is u as
Now, v1 ′ = v2 (entering plase) shown in figure.
2
1+ e ∴ u cos 45° + v cos 45° = v
∴ 10 = ⋅ 16
2 i.e., u = v ( 2 − 1)
1 Total energy released
i.e., e=
4 1 1 1 1
= mv2 + mv2 + mv2 + m [v( 2 − 1)] 2
Option (b) is correct. 2 2 2 2
l 1  L 1
( l ⋅ b) +  L ⋅ b  l +  = mv [3 + ( 2 − 1) ] = mv2 [3 − 2 ]
2 2
2  2   3 2
10. yCM =
 1  Option (a) is correct.
( l ⋅ b) +  L ⋅ b
2  13.
b l2 bL l L2 b  yCM = l 
+ +
l= 2 2 6  according 
bL  
bl + to question
2
 L  l2 L l L2 mg mg 2mg 2mg
or l l +  = + +
 2 2 2 6 2mg 2mg

l2 L2 mg C u C u
or = mg
2 6
A B A B A B
L
or l=
3 mg
mg C u mg 2 mg mg 2 mg
11. m1 v m1
1
A B ⇒ v2 A B v2
mu = 2mv1 C will increase C will also increase the
u the tension in weight of B when collision
m1v1 = m2v2 − m1v2 …(i) i.e., v1 =
2 the string. takes place. Thus,
e=1 mu = 2mv 2 mu = (2m + m ) v 3
u u
∴ 2 v2 = v1 i.e., v 2 = i.e., v 3 =
2 3
∴ m1(2 v2 ) = m2v2 − m1v2
Centre of Mass, Conservation of Linear Momentum, Impulse and Collision | 193
∴ v1 : v2 : v3 m A
u u u
= : : = 3 :3 :2
2 2 3
Option (b) is correct.
Place of
14. time = t (say) collision g 3m
h w
CM
h/2 h/2
v
v 30° 2m
B
v cos 30° Motion of B :
vCM = v cos 30° h h 1 h
=v − g
3 2 g 2 g
=v
2 h
i.e., h=v
Option (a) is correct. g
→ ^
15. r CM = i ∴ v= gh
→ h
r1 (position vector of lighter piece) vB = gh − g ⋅ =0
g
^
= 3 ^i + 2 ^j − 4 k Collision of A and B at time t :
→ →
→ m1 r1 + m2 r2 gh Rest ω
r CM =
m1 + m2
A B A B
→ →
→ ( m1 + m2 ) r CM − m1 r1 m 2m m + 2m
r2 =
m2
m gh = 3 m ω
2 ^ gh
2 i − (3 ^i + 2 ^j − 4 k)
^
∴ ω=
= 3 3
4
Velocity of the combined mass when it
3 reach ground
1 ^ ^ v ′2 = ω2 + 2 gh
= [6 i − 2 (3 ^i + 2 ^j − 4 k)]
4 gh
= + 2 gh
1 ^ g
= [ − 4 ^j + 8 k ]
4 19 gh
i.e., v′ =
^ 3
= − ^j + 2 k
Option (d) is correct.
∴ The heavier part will be at (0, − 1, 2). 17. u = velocity of man w.r.t. cart
Option (d) is correct. Let v = velocity of cart w.r.t. ground
16. Motion of A : ∴ Velocity of man w.r.t. ground = u + v
h 1 2 m (u + v) + 2 mv = 3 m ⋅ 0
= gt , v A (at time t) = g t = gh u
2 2 ∴ v=−
3
h
i.e., t= Work done = KE gained by man and cart
g
1 1
= m (u + v)2 + 2 mv2
2 2
194 | Mechanics-1
1  u 1
2
u
2 1
= m u −  + 2 m  −  = 2 mv2 − mv2
2  3 2  3 2
3
1 4u2 1 u2 mv2=
= m⋅ + 2m 2
2 3 2 9
3 p2
2mu2 mu2 =
= + 2m
3 9
7 Option (c) is correct.
= mu2
9 20. According to question
P
Option (d) is correct. 4m
30 m + 50 m
18. vCM =
m+m
4m 4m
= 40 m/s upwards. CM CM
m
30 m/s x x R
a a
m
a
( 4 m) x + ( 4 m)
2 = ( 4 m) x + ( m) a
40 m Initial position of CM 4m + 4m 4m + m
x a 4x + a
20 m 50 m/s or + =
2 4 5
m a
i.e., x=
6
If the velocity of CM becomes zero at
displacements Option (b) is correct.
02 = 402 + 2 ( − 10) s 21. xCM of Fig. 1 will as that of Fig. 2.
⇒ s = 80 m y

∴ Maximum height attained by CM a 40


= 20 m + 80 m
a 20 10 20
= 100 m x
O
Option (c) is correct. a a a
Fig. 1
19. As the masses are equal and the collision y
is elastic, the particles will exchange their
a 40
velocities as shown in figure.
p – 2p 50 a
In terms of momentum x
O
a a
A B
Fig. 2
Before collision
v 2v 2v v a 3a
40 ⋅ + 50 ⋅
xCM = 2 2
In terms of velocity 40 + 50
A B A B 20a + 75a
Before collision After collision =
90
Gain in KE of 1st particle 95a 19 a
= =
1 1 90 18
= m ( − 2 v)2 − mu2
2 2
Centre of Mass, Conservation of Linear Momentum, Impulse and Collision | 195
3a a There is no  24. Velocity of 2nd ball when 1st with velocity
40 ⋅ + 50 ⋅
yCM = 2 2 need to find  v strikes 2nd at rest
40 + 50  
the value of yCM 
17 a
=
18
r
Option (a) is correct. m m/2 m/22 m/2n – 1
22. Conservation of momentum along y-axis 1 2 3 n
v
m0 0 sin 45° = 2 m0 v sin θ 1st 2nd 3rd nth
2
v v2 = v Rest v2' v1'
i.e., 2 v sin θ = 0 …(i)
2 1 2 1 2
Conservation of momentum along x-axis.
m2 = m m1 = m/2
v0/√2 2m2 2m 4
v1 ′ = v2 = v= v
m0 m1 + m2 m 3
v0 +m
Rest 45° 2
x x
θ Velocity of 3rd ball when 2nd with velocity
m0 2m0
4
v strikes 3rd at rest
2m0 3
v
v2 = 4 v v2' v1'
v 3 Rest
m0 0 cos 45° + 2 m0 v cos θ = m0v0
2 2 3 3
v
i.e., 2 v cos θ = 0 …(ii) m2 = m m1 = m
2 2 4
Squaring and adding Eqs. (i) and (ii), m
2⋅
v 2m2 2 4v
2v = 0 2 v1 ′ = v2 =
2 m1 + m2 m m 3
v +
∴ v= 0 4 2
2 2 4 4
= ⋅ v
Option (b) is correct. 3 3
1 1 4
2
23. kx20 = m2v22 =   v
2 2 3
k 4
3−1
i.e., v2 = x0 Velocity of 3rd ball =   v
m2 3
m × 0 + m2v2 m2 4
vCM = 1 As in every collision =
m1 + m2 m1 + m2 3
n−1
4
(When wall just breaks off the velocity of The velocity of nth ball =   v
mass m1 would be zero) 3
m2
= v2 Now, this must be equal to 5 gr
m1 + m2 for it to complete the circle
n−1
m2 k  4
= x0 ∴   v = 5 gr
m1 + m2 m2 3
x0 n−1
3
= k m2 i.e., v =   5 gr
m1 + m2  4
Option (b) is correct. Option (a) is correct.
196 | Mechanics-1
25. Impulse given to the block will also
release it from abstraction besides giving 5cm
and then imparting the restoring force on
it due to velocity to it 5cm of the spring 6.7 cm
expansion will accelerate it.
F
Impulse = 4 kg ms −1 F 6.7 cm
4 kg ms –1
Initial velocity (u) = = 2 ms −1 3 m/s
2 kg 6.7 cm
Average Acceleration ( a)
5 
4000 ×  
kx  100
= = KE of the block moves blocks ahead and
2m 2×2
the spring stretches by 6.7 cm but the
= 50 m/s2 block does not return due to same region.
5 Block is now at rest for the second time.
Displacement ( s) = x = m
100 ∴ Distance travelled by block when it
∴ v2 = u2 + 2as comes to rest for the second time
5 = (5 + 6.7 + 6.7 + 6.7) cm
= 22 + 2 × 50 × = 25 cm approx.
100
Option (b) is correct.
= 4+5
=9 27. M1= 8 m M2= 16 m
2v (M + s) 2v
∴ v = 3 ms −1
A 48 m P B
Option (b) is correct. 6L
26. Compression in spring Velocity gained 12 L
by block when the spring is at its natural 8 m × AP = 16m × (12L − AP)
length will compress the spring.
AP = 24 L − 2 AP
1 2 1
kx = mv2 3 AP = 24 L
2 2
m AP = 8 L
x=v
k As the CM of M and S does not change,
2 the CM of the bar shall also not change
=3 i.e., the displacement of bar will be zero.
4000
2×9 ×5 Let x be the displacement of rod.
= ∴ x × 8 m + ( x + 6 L) 48 m + ( x + 12 L) 16m
4000 × 5
45 = (6L × 48 m) + (8 L × 24 m)
= i.e., x = 0 m
10000
6.7 i.e., no displacement of bar.
= m
100 Option (d) is correct.
= 6.7 cm 8 m (2 v) + 16 m ( − v)
28. vCM of M and S = =0
8 m + 16 m
Spring at its natural length.
There CM of M and S will not change
Block at rest first time due to some while they move i.e.,the point P (where
reason. they meet) is at the edge of the table
Restoring force brings the spring to its supporting the end B.
natural length and block attains a velocity Option (b) is correct.
of 3 ms −1.
Centre of Mass, Conservation of Linear Momentum, Impulse and Collision | 197
29. When the spider eats up the moth and 20L 20L
= = = 20T
v v L/ T
travels towards A with velocity relative
2
= 20 × 4 = 80 s
to rod.
v 31. Form CM not to shift
24 m  + vR  + 48 m vR = 0 8L M+S
2 
x'
48 m
[vR = velocity (absolute) of rod]
v
⇒ vR = − 48 m
6
Option (c) is correct. ⇒ ( x ′ + 8 L) 24 m + ( x ′ + 6L) 48 m
30. Time taken by spider to reach point A = 64 × 48 m
starting from point B 8L
i.e., x ′ = −
4L 8L 3
= +
v v/2 Option (a) is correct.

More than One Correct Options


v 1
1. Along vertical : 2 mV sin θ = m ⋅ sin 45° = < 1
2 2 2−1
v
i.e., 2 V sin θ = …(i) ∴ θ < 45°
2 2
Thus, the divergence angle between the
v/2 π
m particles will be less than .
2
v Rest 45° Option (b) is correct.
1
θ Initial KE = mv2
m 2m 2
2
1  v 1
2m Final KE = m   + 2 mV 2
2  2 2
L
1 1 5 1 
Along horizontal : = mv2  + 2 ⋅ − 2⋅
2 
4 32 8 2 
v
mv = m cos 45° + 2mV cos θ
2 As Final KE < Initial KE
1 
i.e., 2 V cos θ = v 1 −  …(ii) Collision is inelastic.
 2 2 Option (d) is correct.
Squaring and adding Eq. (i)and (ii), m − m1
2 2 2. v2 ′ = 2 v2
v  v 
8 V 2 =  
 + v −  m1 + m2
2 2  2 2 v2 v1 = 0 v2' v1'
v2 v2
v
= + v2 + − 2⋅v ⋅
8 7 2 2
5v2 v2 m2 = m m1 = 5m
= −
4 2 m − 5m
Dividing Eq. (ii) by Eq. (i) = v2
5m + m
1 2
2 2 = − v2
tan θ = 3
2 2−1 2
=− 2g l
2 2 3
198 | Mechanics-1
mv ′22 u
= mv sin θ  e +
u
T− = mg  v
l v 
mv ′2 = mu sin θ (1 + e)
or T= + mg
l Option (d) is correct.
m8g
= + mg u 1 − (1 − e2 ) sin2 θ
9
17 mg = u 1 − sin2 θ + e2 sin2 θ
=
9
= u cos2 θ + e2 sin2 θ
Option (a) is correct.
Velocity of block v2
= u cos2 θ + sin2 φ
2m2 u2
v1 ′ = v2
m1 + m2 = u2 cos2 θ + v2 sin2 φ
2m
= ⋅ 2g l
5m + m = v2 cos2 φ + v2 sin2 φ
1 =v
= 2g l
3 Option (c) is correct.
Option (c) is correct. cos2 θ + e2 sin2 θ
Maximum height attained by pendulum tan2 φ
bob = cos2 θ + sin2 θ
tan2 θ
v ′2 8 g l / 9 4 l
= 2 = = = cos2 θ (1 + tan2 φ)
2g 2g g
= cos2 θ sec2 φ
Option (d) is correct.
v cos θ cos2 θ
3. v cos φ = u cos θ ⇒ = =
u cos φ cos2 φ
v sin φ v2 Final KE
= 2
=
v
u Initial KE
u cos θ
v cos φ Option (d) is correct.
+ u → ^ ^
u sin θ
4. u = (3 i + 2 j) ms −1

v = (–2 ^i + ^j) m/s
θ φ
u
v sin φ mM m
and e=
u sin θ
Impulse received by particle of mass m
cos θ sin φ
= × → →
cos φ sin θ = − mu + mv
tan φ
= = − m (3 ^i + 2 ^j) + m( − 2 ^i + ^j)
tan θ
Option (b) is correct. = − m (5 ^i + ^j) unit
Change in momentum of particle Option (b) is correct.
= ( − mv sin φ) − ( + mu sin θ) Impulse received by particle of mass M
∴ Impulse delivered by floor to the particle = − (impulse received by particle of
= mv sin φ + mu sin θ mass m)
sin φ u = m (5 ^i + ^j)
= mv sin θ  +
 sin θ v 
Option (d) is correct.
Centre of Mass, Conservation of Linear Momentum, Impulse and Collision | 199
5. T = m1a  1 + e
v1 ′ =  v
 2  2
and m2 g − T = m2 a
m2 3 1
Solving, a= g = v [as e = and v2 = v (given)]
m1 + m2 4 2
v
a ≠
2
m1 Impulse given by A to B
T T
T a = change in momentum of B
3
T = m  v − m ⋅ 0
x 4 
m2 3
= mv
y 4
m2g
Option (b) is correct.
m1a + m2 0 Velocity of A after collision
( aCM ) x =
m1 + m2  1 − e
v2 ′ =  v
m1  2  2
= a v
m1 + m2 =
m1m2 4
= g
( m1 + m2 )2 Loss of KE during collision
1 1
Option (b) is correct. = mv22 − m(v ′21 + v ′22 )
2 2
m 0 + m2 a
( aCM ) y = 1 1  v
2 2
3v 
( m1 + m2 ) = m v2 −   −   
2   4  4 
m2
= a 3
m1 + m2 = mv2
2 16
 m 
=  g Option (c) is correct.
 m1 + m2  8. As the mass of the system keeps on
Option (c) is correct. decreasing momentum of the system does
6. As the block comes down, the CM of the not remain constant.
system will also come down i.e., it does not Thrust force is developed on the rocket
remain stationary. due to Newton’s 3rd law of motion.
mg Option (b) is correct.
aCM = ≠g
m+M dv vi  dm 
As, a= = −  −g
aCM is downwards and also aCM < g. dt m  dt 
Option (d) is correct. The value of a will remain constant if vi
As no force acts along horizontal direction, dm
and − are constant.
the momentum of the system will remain dt
conserved along horizontal direction. Option (c) is correct.
Option (c) is correct. Fnet = Ft (Thrust force due to gas ejection)
7. Velocity of B after collision : − W (weight of rocket)
v2 v1 = 0 v2' v1' F
a = net
m2 m1 m1 m2 m
A B A B Thus, Newton’s 2nd law is applied.
Before collision After collision Option (d) is correct.
200 | Mechanics-1
Match the Columns
1. If x0 is the compression made in the m ( + g) + m ( + g)
2. Initial aCM =
spring, the restoring force on B will m+m
decrease from kx0 to zero as the spring u=0
regains its original length. Thus, the 2nd particle
acceleration of B will also decrease from
kx0
to zero.
mB 180 m

+
mA mB 20 ms–1
k
A B 1st particle
m
x0
=+g
= + 10 SI unit
A B kx0
kx0 ∴ (a) → (q)
m ( − 20) + m × 0
So, the aCM will also decrease from Initial vCM =
m+m
kx0
to zero. = − 10
m A + mB
∴ |vCM|= 10 SI unit
∴ (a) → (r) ∴ (b) → (q)
When spring is released after compressing For the time taken by the first particle to
it, the restoring on B will accelerate it return to ground
towards right while the reaction force on 1
A will apply a force on the wall which in s = ut + at2
2
turn will apply equal and opposite force on
0 = ( − 20) t + 5t2
A and consequently A will travel towards
right. As both travel towards right the ⇒ t=4s
velocity of CM will be maximum in the Now, as the collision of the first particle
beginning. with the ground is perfectly inelastic, the
After this A will start compressing the first particle will remain on ground at
rest.
spring and at a certain instant when the
spring is compressed to maximum value Now, let us find the position of 2nd
both the blocks will travel towards right particle at t = 5 s
1
with a constant velocity and then the s = (0) 5 + (10) 52
velocity of CM will become constant. 2
∴ (b) → (q) = 125 m
As the blocks will never move along The particle (2nd) will still be in space
y-axis, the y-component of the CM of the moving downwards.
m ⋅0 + m ⋅ g
two blocks will not change. aCM =
m+m
∴ (d) → (p) g
As the two blocks will keep on moving = =5 (SI unit)
2
towards right (surface below being
∴ (c) → (p)
smooth) the x-coordinate of the CM of the
blocks will keep on increasing. Velocity of 2nd particle at t = 5 s
∴ (c) → (s) v = 0 + 10 × 5
= 50 ms −1
Centre of Mass, Conservation of Linear Momentum, Impulse and Collision | 201
∴ At t = 5 s ∴ (a) → (r)
m ⋅ 0 + m ⋅ 50 If collision is perfectly inelastic, the two
vCM =
m+m balls will move together (with velocities V).
= 25 (SI unit) ∴ mv = ( m + m) V
v
∴ (d) → (s) ⇒ V =
2
3. Initial KE of block B = 4 J
∴ (b) → (s)
1
A B If collision is inelastic with e = ,
2
m m = 0.5 kg
1+ e
1 v1 ′ = ⋅ v2
∴ × 0.5 × u2 = 4 2
2 1
1+
⇒ u = 4 ms −1 = 2 ⋅ v [Qv = v (given)]
2
∴ Initial momentum of B = 0.5 × 4 2
3
= 2 kg ms −1 = v
4
∴ (a) → (r)
∴ (c) → (p)
Initial momentum 1
If collision is inelastic with e = ,
pCM = pA + pB 4
=0+2 1 + 1
 
= 2 kg ms −1 v1 ′ =
 4 v = 5 v
∴ (b) → (r) 2 8
Velocity given to block B will compress the ∴ (d) → (q).
spring and this will gradually increase the 5. If A moves x towards right
velocity of A. When the spring gets A B
compressed to its maximum both the 50 kg 60 kg
blocks will have the same velocities i.e.,
same momentum as both have same mass. C 30 kg
Smooth
pA = pB
(at maximum compression of the spring) Let plank (along with B) move by x′ to the
But, pA + pB = initial momentum of B. right.
x × 30 + x ′ (60 + 30)
∴ pA + pA = 2 ∴ =0
30 + (60 + 30)
i.e., pA = 1 kgms −1
x
∴ (c) → (q) i.e., x′ = −
3
After the maximum compression in the
x
spring, the spring will gradually expand = , towards left.
but now the velocity of block A will 3
increase and that of B will decrease and ∴ (a) → (r)
when the spring attains maximum If B moves x towards left
expansion the velocity of B will be zero
Let plank (along with A) move x′ to the
and so will be its momentum.
left
∴ (d) → (p) x ⋅ 60 + x ′ (30 + 30)
∴ =0
4. If collision is elastic, the two blocks will 60 + (30 + 30)
interchange there velocities (mass of both
balls being equal). i.e., x′ = − x
Thus, velocity of A after collision = v = x, towards right
202 | Mechanics-1
∴ (b) → (p) = (60 − 10) 10
If A moves x towards right and B moves x = 500 N
towards left. ∴ (a) → (r)
Let plank moves x′ towards right For the equilibrium of man
30 × x + 60 ( − x) + 30 ( x ′ )
∴ =0 N = W − T1
30 + (60) + (30)
= W − 2T2
i.e., x ′ = x = W − 2 ( N + m1 g)
= x, towards right
i.e., 3 N = W − 2m1 g
∴ (c) → (p)
N = 100 N
If A and B both move x towards right.
∴ (d) → (s)
Let plank moves x′ towards right
Force exerted by man on string to
(30 + 60) x + 30x ′
∴ =0 accelerate the centre of mass of the system
(30 + 60) + 30 upwards
i.e., x′ = − 3 x Centre of mass of the system will move
= 3x, towards right upwards if man move upward.
∴ (d) → (s) i.e., when T1 > W − N
6. For man to be in equilibrium > 500 − 10
> 400 N
Options are 500 N and 600 N.
∴ (b) → (r) and (s).
Force ( T1) exerted by man on string to
1 accelerate the centre of mass of the
T1 system.
T1
T1 Centre of mass of the system will move
N downward if man moves downward.
T1 2 i.e., when T1 < W − N
T2 T2
T2 < 400 N
T2
Options are 100 N and 150 N.
m1 m2g ∴ (c) → (q).
2×3 +0×6
N 7. vCM =
m1g 3+6
N + T1 = W …(i) 2ms–1
3 kg 6 kg
For the block of mass m1 to be in
quilibrium 2
= ms −1
T2 = N + m1 g …(ii) 3
For the block of mass m2 to be in When both the blocks move with same
quilibrium velocity (say v) deformation in the spring
T2 = m2 g …(iii) will be maximum
For the equilibrium of pulley 2 2 × 3 = 3v + 6 v
T1 = 2T2 …(iv) 2
i.e., v = ms −1
Solving Eqs. (i), (ii), (iii) and (iv) 3
W = 3 m2 g − m1 g = velocity of A
= (3 m2 − m1) g = velocity of B
Centre of Mass, Conservation of Linear Momentum, Impulse and Collision | 203
2 2 2 × 5 + 1 ( − 10)
3 × +6× 8. vCM =
vCM = 3 3 2+1
3+6
= 0 ms −1
2
= ms −1 + ive
3
When both the blocks move with same Rough
10 ms–1
velocities, each will be at rest w.r.t. the 1 kg
5 ms–1
other 2 kg
∴ (a) → (p), (r), (s) Smooth
(b) → (p), (r), (s)
∴ (a) → (r)
Minimum speed of 3 kg block will be
2 Momentum of CM = 2 ( + 5) + 1 ( −10)
ms −1 and at that moment velocity of CM
3 = 0 kg ms −1
2 ∴ (b) → (r)
will be ms −1.
3 Velocity and so the momentum of 1 kg
∴ (c) → (p) block will decrease to zero as the surface
Initial velocity of 6 kg block is zero. below is rough.
∴ (c) → (q)
When the spring is compressed to
maximum value the velocity of 6 kg will Velocity and so the KE of 2 kg block will
be maximum and the velocity of CM will decrease to zero when the velocity of 1 kg
2 block becomes zero (according to law of
be ms −1 (as explained above).
3 conservation of momentum).
∴ (c) → (p). ∴ (d) → (q).

You might also like